Π‘ΠΎΠ΄Π΅Ρ€ΠΆΠ°Π½ΠΈΠ΅

УдСльноС сопротивлСниС. РСостаты β€” ΡƒΡ€ΠΎΠΊ. Π€ΠΈΠ·ΠΈΠΊΠ°, 8 класс.

Π‘ΠΎΠ±Π΅Ρ€Ρ‘ΠΌ Ρ†Π΅ΠΏΡŒ, ΠΈΠ·ΠΎΠ±Ρ€Π°ΠΆΡ‘Π½Π½ΡƒΡŽ Π½Π° рисункС. Π‘ΠΈΠ»Ρƒ Ρ‚ΠΎΠΊΠ° Π² Ρ†Π΅ΠΏΠΈ ΠΈΠ·ΠΌΠ΅Ρ€ΡΡŽΡ‚ Π°ΠΌΠΏΠ΅Ρ€ΠΌΠ΅Ρ‚Ρ€ΠΎΠΌ, напряТСниС β€” Π²ΠΎΠ»ΡŒΡ‚ΠΌΠ΅Ρ‚Ρ€ΠΎΠΌ. Зная напряТСниС Π½Π° ΠΊΠΎΠ½Ρ†Π°Ρ… ΠΏΡ€ΠΎΠ²ΠΎΠ΄Π½ΠΈΠΊΠ° ΠΈ силу Ρ‚ΠΎΠΊΠ° Π² Π½Ρ‘ΠΌ, ΠΏΠΎ Π·Π°ΠΊΠΎΠ½Ρƒ Ома ΠΌΠΎΠΆΠ½ΠΎ ΠΎΠΏΡ€Π΅Π΄Π΅Π»ΠΈΡ‚ΡŒ сопротивлСниС ΠΊΠ°ΠΆΠ΄ΠΎΠ³ΠΎ ΠΈΠ· ΠΏΡ€ΠΎΠ²ΠΎΠ΄Π½ΠΈΠΊΠΎΠ².

Β 

Β 

Π’ Ρ†Π΅ΠΏΡŒ источника Ρ‚ΠΎΠΊΠ° ΠΏΠΎ ΠΎΡ‡Π΅Ρ€Π΅Π΄ΠΈ Π±ΡƒΠ΄Π΅ΠΌ Π²ΠΊΠ»ΡŽΡ‡Π°Ρ‚ΡŒ Ρ€Π°Π·Π»ΠΈΡ‡Π½Ρ‹Π΅ ΠΏΡ€ΠΎΠ²ΠΎΠ΄Π½ΠΈΠΊΠΈ, Π½Π°ΠΏΡ€ΠΈΠΌΠ΅Ρ€, Π½ΠΈΠΊΠ΅Π»ΠΈΠ½ΠΎΠ²Ρ‹Π΅ ΠΏΡ€ΠΎΠ²ΠΎΠ»ΠΎΠΊΠΈ ΠΎΠ΄ΠΈΠ½Π°ΠΊΠΎΠ²ΠΎΠΉ Ρ‚ΠΎΠ»Ρ‰ΠΈΠ½Ρ‹, Π½ΠΎ Ρ€Π°Π·Π½ΠΎΠΉ Π΄Π»ΠΈΠ½Ρ‹. Π’Ρ‹ΠΏΠΎΠ»Π½ΠΈΠ² ΡƒΠΊΠ°Π·Π°Π½Π½Ρ‹Π΅ ΠΎΠΏΡ‹Ρ‚Ρ‹, ΠΌΡ‹ установим, Ρ‡Ρ‚ΠΎ ΠΈΠ· Π΄Π²ΡƒΡ… Π½ΠΈΠΊΠ΅Π»ΠΈΠ½ΠΎΠ²Ρ‹Ρ… ΠΏΡ€ΠΎΠ²ΠΎΠ»ΠΎΠΊ ΠΎΠ΄ΠΈΠ½Π°ΠΊΠΎΠ²ΠΎΠΉ Ρ‚ΠΎΠ»Ρ‰ΠΈΠ½Ρ‹ Π±ΠΎΠ»Π΅Π΅ длинная ΠΏΡ€ΠΎΠ²ΠΎΠ»ΠΎΠΊΠ° ΠΈΠΌΠ΅Π΅Ρ‚ большСС сопротивлСниС.
Π’ ΡΠ»Π΅Π΄ΡƒΡŽΡ‰Π΅ΠΌ экспСримСнтС ΠΏΠΎ ΠΎΡ‡Π΅Ρ€Π΅Π΄ΠΈ Π±ΡƒΠ΄Π΅ΠΌ Π²ΠΊΠ»ΡŽΡ‡Π°Ρ‚ΡŒ Π½ΠΈΠΊΠ΅Π»ΠΈΠ½ΠΎΠ²Ρ‹Π΅ ΠΏΡ€ΠΎΠ²ΠΎΠ»ΠΎΠΊΠΈ ΠΎΠ΄ΠΈΠ½Π°ΠΊΠΎΠ²ΠΎΠΉ Π΄Π»ΠΈΠ½Ρ‹, Π½ΠΎ Ρ€Π°Π·Π½ΠΎΠΉ Ρ‚ΠΎΠ»Ρ‰ΠΈΠ½Ρ‹ (Ρ€Π°Π·Π½ΠΎΠΉ ΠΏΠ»ΠΎΡ‰Π°Π΄ΠΈ ΠΏΠΎΠΏΠ΅Ρ€Π΅Ρ‡Π½ΠΎΠ³ΠΎ сСчСния). Установим, Ρ‡Ρ‚ΠΎ ΠΈΠ· Π΄Π²ΡƒΡ… Π½ΠΈΠΊΠ΅Π»ΠΈΠ½ΠΎΠ²Ρ‹Ρ… ΠΏΡ€ΠΎΠ²ΠΎΠ»ΠΎΠΊ ΠΎΠ΄ΠΈΠ½Π°ΠΊΠΎΠ²ΠΎΠΉ Π΄Π»ΠΈΠ½Ρ‹ большСС сопротивлСниС ΠΈΠΌΠ΅Π΅Ρ‚ ΠΏΡ€ΠΎΠ²ΠΎΠ»ΠΎΠΊΠ°, ΠΏΠΎΠΏΠ΅Ρ€Π΅Ρ‡Π½ΠΎΠ΅ сСчСниС ΠΊΠΎΡ‚ΠΎΡ€ΠΎΠΉ мСньшС.
Π’ Ρ‚Ρ€Π΅Ρ‚ΡŒΠ΅ΠΌ экспСримСнтС ΠΏΠΎ ΠΎΡ‡Π΅Ρ€Π΅Π΄ΠΈ Π±ΡƒΠ΄Π΅ΠΌ Π²ΠΊΠ»ΡŽΡ‡Π°Ρ‚ΡŒ Π½ΠΈΠΊΠ΅Π»ΠΈΠ½ΠΎΠ²ΡƒΡŽ ΠΈ Π½ΠΈΡ…Ρ€ΠΎΠΌΠΎΠ²ΡƒΡŽ ΠΏΡ€ΠΎΠ²ΠΎΠ»ΠΎΠΊΠΈ ΠΎΠ΄ΠΈΠ½Π°ΠΊΠΎΠ²ΠΎΠΉ Π΄Π»ΠΈΠ½Ρ‹ ΠΈ Ρ‚ΠΎΠ»Ρ‰ΠΈΠ½Ρ‹. Установим, Ρ‡Ρ‚ΠΎ никСлиновая ΠΈ нихромовая ΠΏΡ€ΠΎΠ²ΠΎΠ»ΠΎΠΊΠΈ ΠΎΠ΄ΠΈΠ½Π°ΠΊΠΎΠ²Ρ‹Ρ… Ρ€Π°Π·ΠΌΠ΅Ρ€ΠΎΠ² ΠΈΠΌΠ΅ΡŽΡ‚ Ρ€Π°Π·Π½ΠΎΠ΅ сопротивлСниС.
Π—Π°Π²ΠΈΡΠΈΠΌΠΎΡΡ‚ΡŒ сопротивлСния ΠΏΡ€ΠΎΠ²ΠΎΠ΄Π½ΠΈΠΊΠ° ΠΎΡ‚ Π΅Π³ΠΎ Ρ€Π°Π·ΠΌΠ΅Ρ€ΠΎΠ² ΠΈ вСщСства, ΠΈΠ· ΠΊΠΎΡ‚ΠΎΡ€ΠΎΠ³ΠΎ ΠΈΠ·Π³ΠΎΡ‚ΠΎΠ²Π»Π΅Π½ ΠΏΡ€ΠΎΠ²ΠΎΠ΄Π½ΠΈΠΊ, Π²ΠΏΠ΅Ρ€Π²Ρ‹Π΅ Π½Π° ΠΎΠΏΡ‹Ρ‚Π°Ρ… ΠΈΠ·ΡƒΡ‡ΠΈΠ» Ом. Он установил:

Π‘ΠΎΠΏΡ€ΠΎΡ‚ΠΈΠ²Π»Π΅Π½ΠΈΠ΅ прямо ΠΏΡ€ΠΎΠΏΠΎΡ€Ρ†ΠΈΠΎΠ½Π°Π»ΡŒΠ½ΠΎ Π΄Π»ΠΈΠ½Π΅ ΠΏΡ€ΠΎΠ²ΠΎΠ΄Π½ΠΈΠΊΠ°, ΠΎΠ±Ρ€Π°Ρ‚Π½ΠΎ ΠΏΡ€ΠΎΠΏΠΎΡ€Ρ†ΠΈΠΎΠ½Π°Π»ΡŒΠ½ΠΎ ΠΏΠ»ΠΎΡ‰Π°Π΄ΠΈ Π΅Π³ΠΎ ΠΏΠΎΠΏΠ΅Ρ€Π΅Ρ‡Π½ΠΎΠ³ΠΎ сСчСния ΠΈ зависит ΠΎΡ‚ вСщСства ΠΏΡ€ΠΎΠ²ΠΎΠ΄Π½ΠΈΠΊΠ°.

Β 

ΠžΠ±Ρ€Π°Ρ‚ΠΈ Π²Π½ΠΈΠΌΠ°Π½ΠΈΠ΅!

Π‘ΠΎΠΏΡ€ΠΎΡ‚ΠΈΠ²Π»Π΅Π½ΠΈΠ΅ ΠΏΡ€ΠΎΠ²ΠΎΠ΄Π½ΠΈΠΊΠ° прямо ΠΏΡ€ΠΎΠΏΠΎΡ€Ρ†ΠΈΠΎΠ½Π°Π»ΡŒΠ½ΠΎ Π΅Π³ΠΎ Π΄Π»ΠΈΠ½Π΅, Ρ‚.Π΅. Ρ‡Π΅ΠΌ Π΄Π»ΠΈΠ½Π½Π΅Π΅ ΠΏΡ€ΠΎΠ²ΠΎΠ΄Π½ΠΈΠΊ, Ρ‚Π΅ΠΌ большС Π΅Π³ΠΎ элСктричСскоС сопротивлСниС.
Π‘ΠΎΠΏΡ€ΠΎΡ‚ΠΈΠ²Π»Π΅Π½ΠΈΠ΅ ΠΏΡ€ΠΎΠ²ΠΎΠ΄Π½ΠΈΠΊΠ° ΠΎΠ±Ρ€Π°Ρ‚Π½ΠΎ ΠΏΡ€ΠΎΠΏΠΎΡ€Ρ†ΠΈΠΎΠ½Π°Π»ΡŒΠ½ΠΎ ΠΏΠ»ΠΎΡ‰Π°Π΄ΠΈ Π΅Π³ΠΎ ΠΏΠΎΠΏΠ΅Ρ€Π΅Ρ‡Π½ΠΎΠ³ΠΎ сСчСния, Ρ‚.Π΅. Ρ‡Π΅ΠΌ Ρ‚ΠΎΠ»Ρ‰Π΅ ΠΏΡ€ΠΎΠ²ΠΎΠ΄Π½ΠΈΠΊ, Ρ‚Π΅ΠΌ Π΅Π³ΠΎ сопротивлСниС мСньшС, ΠΈ, Π½Π°ΠΎΠ±ΠΎΡ€ΠΎΡ‚, Ρ‡Π΅ΠΌ Ρ‚ΠΎΠ½ΡŒΡˆΠ΅ ΠΏΡ€ΠΎΠ²ΠΎΠ΄Π½ΠΈΠΊ, Ρ‚Π΅ΠΌ Π΅Π³ΠΎ сопротивлСниС большС.

Π§Ρ‚ΠΎΠ±Ρ‹ Π»ΡƒΡ‡ΡˆΠ΅ ΠΏΠΎΠ½ΡΡ‚ΡŒ эту Π·Π°Π²ΠΈΡΠΈΠΌΠΎΡΡ‚ΡŒ, ΠΏΡ€Π΅Π΄ΡΡ‚Π°Π²ΡŒΡ‚Π΅ сСбС Π΄Π²Π΅ ΠΏΠ°Ρ€Ρ‹ ΡΠΎΠΎΠ±Ρ‰Π°ΡŽΡ‰ΠΈΡ…ΡΡ сосудов, ΠΏΡ€ΠΈΡ‡Ρ‘ΠΌ Ρƒ ΠΎΠ΄Π½ΠΎΠΉ ΠΏΠ°Ρ€Ρ‹ сосудов ΡΠΎΠ΅Π΄ΠΈΠ½ΡΡŽΡ‰Π°Ρ Ρ‚Ρ€ΡƒΠ±ΠΊΠ° тонкая, Π° Ρƒ Π΄Ρ€ΡƒΠ³ΠΎΠΉ β€” толстая. Ясно, Ρ‡Ρ‚ΠΎ ΠΏΡ€ΠΈ Π·Π°ΠΏΠΎΠ»Π½Π΅Π½ΠΈΠΈ Π²ΠΎΠ΄ΠΎΠΉ ΠΎΠ΄Π½ΠΎΠ³ΠΎ ΠΈΠ· сосудов (ΠΊΠ°ΠΆΠ΄ΠΎΠΉ ΠΏΠ°Ρ€Ρ‹) ΠΏΠ΅Ρ€Π΅Ρ…ΠΎΠ΄ Π΅Ρ‘ Π² Π΄Ρ€ΡƒΠ³ΠΎΠΉ сосуд ΠΏΠΎ толстой Ρ‚Ρ€ΡƒΠ±ΠΊΠ΅ ΠΏΡ€ΠΎΠΈΠ·ΠΎΠΉΠ΄Ρ‘Ρ‚ Π³ΠΎΡ€Π°Π·Π΄ΠΎ быстрСС, Ρ‡Π΅ΠΌ ΠΏΠΎ Ρ‚ΠΎΠ½ΠΊΠΎΠΉ, Ρ‚.Π΅. толстая Ρ‚Ρ€ΡƒΠ±ΠΊΠ° ΠΎΠΊΠ°ΠΆΠ΅Ρ‚ мСньшСС сопротивлСниС Ρ‚Π΅Ρ‡Π΅Π½ΠΈΡŽ Π²ΠΎΠ΄Ρ‹. Π’ΠΎΡ‡Π½ΠΎ Ρ‚Π°ΠΊ ΠΆΠ΅ ΠΈ элСктричСскому Ρ‚ΠΎΠΊΡƒ Π»Π΅Π³Ρ‡Π΅ ΠΏΡ€ΠΎΠΉΡ‚ΠΈ ΠΏΠΎ толстому ΠΏΡ€ΠΎΠ²ΠΎΠ΄Π½ΠΈΠΊΡƒ, Ρ‡Π΅ΠΌ ΠΏΠΎ Ρ‚ΠΎΠ½ΠΊΠΎΠΌΡƒ, Ρ‚.Π΅. ΠΏΠ΅Ρ€Π²Ρ‹ΠΉ ΠΎΠΊΠ°Π·Ρ‹Π²Π°Π΅Ρ‚ Π΅ΠΌΡƒ мСньшСС сопротивлСниС, Ρ‡Π΅ΠΌ Π²Ρ‚ΠΎΡ€ΠΎΠΉ.

ΠŸΡ€ΠΈΡ‡ΠΈΠ½ΠΎΠΉ наличия сопротивлСния Ρƒ ΠΏΡ€ΠΎΠ²ΠΎΠ΄Π½ΠΈΠΊΠ° являСтся взаимодСйствиС двиТущихся элСктронов с ΠΈΠΎΠ½Π°ΠΌΠΈ кристалличСской Ρ€Π΅ΡˆΡ‘Ρ‚ΠΊΠΈ ΠΏΡ€ΠΎΠ²ΠΎΠ΄Π½ΠΈΠΊΠ°. Из-Π·Π° различия Π² строСнии кристалличСской Ρ€Π΅ΡˆΡ‘Ρ‚ΠΊΠΈ Ρƒ ΠΏΡ€ΠΎΠ²ΠΎΠ΄Π½ΠΈΠΊΠΎΠ², Π²Ρ‹ΠΏΠΎΠ»Π½Π΅Π½Π½Ρ‹Ρ… ΠΈΠ· Ρ€Π°Π·Π»ΠΈΡ‡Π½Ρ‹Ρ… вСщСств, сопротивлСния ΠΈΡ… ΠΎΡ‚Π»ΠΈΡ‡Π°ΡŽΡ‚ΡΡ Π΄Ρ€ΡƒΠ³ ΠΎΡ‚ Π΄Ρ€ΡƒΠ³Π°. Для характСристики ΠΌΠ°Ρ‚Π΅Ρ€ΠΈΠ°Π»Π° вводят Π²Π΅Π»ΠΈΡ‡ΠΈΠ½Ρƒ, ΠΊΠΎΡ‚ΠΎΡ€ΡƒΡŽ Π½Π°Π·Ρ‹Π²Π°ΡŽΡ‚ ΡƒΠ΄Π΅Π»ΡŒΠ½Ρ‹ΠΌ сопротивлСниСм.

УдСльноС сопротивлСниС β€” это физичСская Π²Π΅Π»ΠΈΡ‡ΠΈΠ½Π°, которая опрСдСляСт сопротивлСниС ΠΏΡ€ΠΎΠ²ΠΎΠ΄Π½ΠΈΠΊΠ° ΠΈΠ· Π΄Π°Π½Π½ΠΎΠ³ΠΎ вСщСства Π΄Π»ΠΈΠ½ΠΎΠΉ \(1\) ΠΌ ΠΈΒ ΠΏΠ»ΠΎΡ‰Π°Π΄ΡŒΡŽ ΠΏΠΎΠΏΠ΅Ρ€Π΅Ρ‡Π½ΠΎΠ³ΠΎ сСчСния \(1\) ΠΌΒ².

Π’Π²Π΅Π΄Ρ‘ΠΌ Π±ΡƒΠΊΠ²Π΅Π½Π½Ρ‹Π΅ обозначСния: \(ρ\) β€” ΡƒΠ΄Π΅Π»ΡŒΠ½ΠΎΠ΅ сопротивлСниС ΠΏΡ€ΠΎΠ²ΠΎΠ΄Π½ΠΈΠΊΠ°, \(l\) β€” Π΄Π»ΠΈΠ½Π° ΠΏΡ€ΠΎΠ²ΠΎΠ΄Π½ΠΈΠΊΠ°, \(S\) β€” ΠΏΠ»ΠΎΡ‰Π°Π΄ΡŒ Π΅Π³ΠΎ ΠΏΠΎΠΏΠ΅Ρ€Π΅Ρ‡Π½ΠΎΠ³ΠΎ сСчСния. Π’ΠΎΠ³Π΄Π° сопротивлСниС ΠΏΡ€ΠΎΠ²ΠΎΠ΄Π½ΠΈΠΊΠ° \(R\) выразится Ρ„ΠΎΡ€ΠΌΡƒΠ»ΠΎΠΉ:


R=ρ⋅lS.

Β 

Из этой Ρ„ΠΎΡ€ΠΌΡƒΠ»Ρ‹ ΠΌΠΎΠΆΠ½ΠΎ Π²Ρ‹Ρ€Π°Π·ΠΈΡ‚ΡŒ ΠΈ Π΄Ρ€ΡƒΠ³ΠΈΠ΅ Π²Π΅Π»ΠΈΡ‡ΠΈΠ½Ρ‹:

Β 

l=Rβ‹…Sρ, S=ρ⋅lR, ρ=Rβ‹…Sl.

Β 

Из послСднСй Ρ„ΠΎΡ€ΠΌΡƒΠ»Ρ‹ ΠΌΠΎΠΆΠ½ΠΎ ΠΎΠΏΡ€Π΅Π΄Π΅Π»ΠΈΡ‚ΡŒ Π΅Π΄ΠΈΠ½ΠΈΡ†Ρƒ ΡƒΠ΄Π΅Π»ΡŒΠ½ΠΎΠ³ΠΎ сопротивлСния. Π’Π°ΠΊ ΠΊΠ°ΠΊ Π΅Π΄ΠΈΠ½ΠΈΡ†Π΅ΠΉ сопротивлСния являСтся \(1\) Ом, Π΅Π΄ΠΈΠ½ΠΈΡ†Π΅ΠΉ ΠΏΠ»ΠΎΡ‰Π°Π΄ΠΈ ΠΏΠΎΠΏΠ΅Ρ€Π΅Ρ‡Π½ΠΎΠ³ΠΎ сСчСния β€” \(1\) ΠΌΒ², Π° Π΅Π΄ΠΈΠ½ΠΈΡ†Π΅ΠΉ Π΄Π»ΠΈΠ½Ρ‹ β€” \(1\) ΠΌ, Ρ‚ΠΎ Π΅Π΄ΠΈΠ½ΠΈΡ†Π΅ΠΉ ΡƒΠ΄Π΅Π»ΡŒΠ½ΠΎΠ³ΠΎ сопротивлСния Π±ΡƒΠ΄Π΅Ρ‚:

Β 

1Β ΠžΠΌΒ β‹…1ΠΌ21Β ΠΌ=1Β ΠžΠΌΒ β‹…1Β ΠΌ, Ρ‚.Π΅. ΠžΠΌβ‹…ΠΌ.

Β 

Π£Π΄ΠΎΠ±Π½Π΅Π΅ Π²Ρ‹Ρ€Π°ΠΆΠ°Ρ‚ΡŒ ΠΏΠ»ΠΎΡ‰Π°Π΄ΡŒ ΠΏΠΎΠΏΠ΅Ρ€Π΅Ρ‡Π½ΠΎΠ³ΠΎ сСчСния ΠΏΡ€ΠΎΠ²ΠΎΠ΄Π½ΠΈΠΊΠ° Π² ΠΊΠ²Π°Π΄Ρ€Π°Ρ‚Π½Ρ‹Ρ… ΠΌΠΈΠ»Π»ΠΈΠΌΠ΅Ρ‚Ρ€Π°Ρ…, Ρ‚Π°ΠΊ ΠΊΠ°ΠΊ ΠΎΠ½Π° Ρ‡Π°Ρ‰Π΅ всСго Π±Ρ‹Π²Π°Π΅Ρ‚ нСбольшой. Π’ΠΎΠ³Π΄Π° Π΅Π΄ΠΈΠ½ΠΈΡ†Π΅ΠΉ ΡƒΠ΄Π΅Π»ΡŒΠ½ΠΎΠ³ΠΎ сопротивлСния Π±ΡƒΠ΄Π΅Ρ‚:

Β 

1Β ΠžΠΌΒ β‹…1ΠΌΠΌ21Β ΠΌ, Ρ‚.Π΅. ΠžΠΌβ‹…ΠΌΠΌ2ΠΌ.

Β 

Π’ Ρ‚Π°Π±Π»ΠΈΡ†Π΅ ΠΏΡ€ΠΈΠ²Π΅Π΄Π΅Π½Ρ‹ значСния ΡƒΠ΄Π΅Π»ΡŒΠ½ΠΎΠ³ΠΎ сопротивлСния Π½Π΅ΠΊΠΎΡ‚ΠΎΡ€Ρ‹Ρ… вСщСств ΠΏΡ€ΠΈ \(20\) Β°Π‘.

Β 

Β 

ΠžΠ±Ρ€Π°Ρ‚ΠΈ Π²Π½ΠΈΠΌΠ°Π½ΠΈΠ΅!

УдСльноС сопротивлСниС с ΠΈΠ·ΠΌΠ΅Π½Π΅Π½ΠΈΠ΅ΠΌ Ρ‚Π΅ΠΌΠΏΠ΅Ρ€Π°Ρ‚ΡƒΡ€Ρ‹ мСняСтся.

ΠžΠΏΡ‹Ρ‚Π½Ρ‹ΠΌ ΠΏΡƒΡ‚Ρ‘ΠΌ Π±Ρ‹Π»ΠΎ установлСно, Ρ‡Ρ‚ΠΎ Ρƒ ΠΌΠ΅Ρ‚Π°Π»Π»ΠΎΠ², Π½Π°ΠΏΡ€ΠΈΠΌΠ΅Ρ€, ΡƒΠ΄Π΅Π»ΡŒΠ½ΠΎΠ΅ сопротивлСниС с ΠΏΠΎΠ²Ρ‹ΡˆΠ΅Π½ΠΈΠ΅ΠΌ Ρ‚Π΅ΠΌΠΏΠ΅Ρ€Π°Ρ‚ΡƒΡ€Ρ‹ увСличиваСтся.

Β 

ΠžΠ±Ρ€Π°Ρ‚ΠΈ Π²Π½ΠΈΠΌΠ°Π½ΠΈΠ΅!

Из всСх ΠΌΠ΅Ρ‚Π°Π»Π»ΠΎΠ² наимСньшим ΡƒΠ΄Π΅Π»ΡŒΠ½Ρ‹ΠΌ сопротивлСниСм ΠΎΠ±Π»Π°Π΄Π°ΡŽΡ‚ сСрСбро ΠΈ мСдь. Π‘Π»Π΅Π΄ΠΎΠ²Π°Ρ‚Π΅Π»ΡŒΠ½ΠΎ, сСрСбро ΠΈ мСдь β€” Π»ΡƒΡ‡ΡˆΠΈΠ΅ ΠΏΡ€ΠΎΠ²ΠΎΠ΄Π½ΠΈΠΊΠΈ элСктричСства.

ΠŸΡ€ΠΈ ΠΏΡ€ΠΎΠ²ΠΎΠ΄ΠΊΠ΅ элСктричСских Ρ†Π΅ΠΏΠ΅ΠΉ ΠΈΡΠΏΠΎΠ»ΡŒΠ·ΡƒΡŽΡ‚ Π°Π»ΡŽΠΌΠΈΠ½ΠΈΠ΅Π²Ρ‹Π΅, ΠΌΠ΅Π΄Π½Ρ‹Π΅ ΠΈ ΠΆΠ΅Π»Π΅Π·Π½Ρ‹Π΅ ΠΏΡ€ΠΎΠ²ΠΎΠ΄Π°.
Π’ΠΎ ΠΌΠ½ΠΎΠ³ΠΈΡ… случаях Π½ΡƒΠΆΠ½Ρ‹ ΠΏΡ€ΠΈΠ±ΠΎΡ€Ρ‹, ΠΈΠΌΠ΅ΡŽΡ‰ΠΈΠ΅ большоС сопротивлСниС. Π˜Ρ… ΠΈΠ·Π³ΠΎΡ‚Π°Π²Π»ΠΈΠ²Π°ΡŽΡ‚ ΠΈΠ· ΡΠΏΠ΅Ρ†ΠΈΠ°Π»ΡŒΠ½ΠΎ созданных сплавов β€” вСщСств с большим ΡƒΠ΄Π΅Π»ΡŒΠ½Ρ‹ΠΌ сопротивлСниСм. НапримСр, ΠΊΠ°ΠΊ Π²ΠΈΠ΄Π½ΠΎ ΠΈΠ· Ρ‚Π°Π±Π»ΠΈΡ†Ρ‹, сплав Π½ΠΈΡ…Ρ€ΠΎΠΌ ΠΈΠΌΠ΅Π΅Ρ‚ ΡƒΠ΄Π΅Π»ΡŒΠ½ΠΎΠ΅ сопротивлСниС ΠΏΠΎΡ‡Ρ‚ΠΈ Π² \(40\) Ρ€Π°Π· большСС, Ρ‡Π΅ΠΌ алюминий.

Β 

ΠžΠ±Ρ€Π°Ρ‚ΠΈ Π²Π½ΠΈΠΌΠ°Π½ΠΈΠ΅!

Π‘Ρ‚Π΅ΠΊΠ»ΠΎ ΠΈ Π΄Π΅Ρ€Π΅Π²ΠΎΒ ΠΈΠΌΠ΅ΡŽΡ‚ Ρ‚Π°ΠΊΠΎΠ΅ большоС ΡƒΠ΄Π΅Π»ΡŒΠ½ΠΎΠ΅ сопротивлСниС, Ρ‡Ρ‚ΠΎ ΠΏΠΎΡ‡Ρ‚ΠΈ совсСм Π½Π΅ проводят элСктричСский Ρ‚ΠΎΠΊ ΠΈ ΡΠ²Π»ΡΡŽΡ‚ΡΡ изоляторами.

Β 

На ΠΏΡ€Π°ΠΊΡ‚ΠΈΠΊΠ΅ часто приходится ΠΌΠ΅Π½ΡΡ‚ΡŒ силу Ρ‚ΠΎΠΊΠ° Π² Ρ†Π΅ΠΏΠΈ, дСлая Π΅Ρ‘ Ρ‚ΠΎ большС, Ρ‚ΠΎ мСньшС. Π’Π°ΠΊ, измСняя силу Ρ‚ΠΎΠΊΠ° Π² Π΄ΠΈΠ½Π°ΠΌΠΈΠΊΠ΅ Ρ€Π°Π΄ΠΈΠΎΠΏΡ€ΠΈΡ‘ΠΌΠ½ΠΈΠΊΠ°, ΠΌΡ‹ Ρ€Π΅Π³ΡƒΠ»ΠΈΡ€ΡƒΠ΅ΠΌ Π³Ρ€ΠΎΠΌΠΊΠΎΡΡ‚ΡŒ Π·Π²ΡƒΠΊΠ°. ИзмСнСниСм силы Ρ‚ΠΎΠΊΠ° Π² элСктродвигатСлС швСйной ΠΌΠ°ΡˆΠΈΠ½Ρ‹ ΠΌΠΎΠΆΠ½ΠΎ Ρ€Π΅Π³ΡƒΠ»ΠΈΡ€ΠΎΠ²Π°Ρ‚ΡŒ ΡΠΊΠΎΡ€ΠΎΡΡ‚ΡŒ Π΅Π³ΠΎ вращСния.
Β 

Для рСгулирования силы Ρ‚ΠΎΠΊΠ° Π² Ρ†Π΅ΠΏΠΈ ΠΏΡ€ΠΈΠΌΠ΅Π½ΡΡŽΡ‚ ΡΠΏΠ΅Ρ†ΠΈΠ°Π»ΡŒΠ½Ρ‹Π΅ ΠΏΡ€ΠΈΠ±ΠΎΡ€Ρ‹ β€” рСостаты.

ΠŸΡ€ΠΎΡΡ‚Π΅ΠΉΡˆΠΈΠΌ рСостатом ΠΌΠΎΠΆΠ΅Ρ‚ ΡΠ»ΡƒΠΆΠΈΡ‚ΡŒ ΠΏΡ€ΠΎΠ²ΠΎΠ»ΠΎΠΊΠ° ΠΈΠ· ΠΌΠ°Ρ‚Π΅Ρ€ΠΈΠ°Π»Π° с большим ΡƒΠ΄Π΅Π»ΡŒΠ½Ρ‹ΠΌ сопротивлСниСм, Π½Π°ΠΏΡ€ΠΈΠΌΠ΅Ρ€, никСлиновая ΠΈΠ»ΠΈ нихромовая. Π’ΠΊΠ»ΡŽΡ‡ΠΈΠ² Ρ‚Π°ΠΊΡƒΡŽ ΠΏΡ€ΠΎΠ²ΠΎΠ»ΠΎΡ‡ΠΊΡƒ Π² Ρ†Π΅ΠΏΡŒ источника элСктричСского Ρ‚ΠΎΠΊΠ° Ρ‡Π΅Ρ€Π΅Π· ΠΊΠΎΠ½Ρ‚Π°ΠΊΡ‚Ρ‹ А ΠΈ Π‘ ΠΈ пСрСдвигая ΠΏΠΎΠ΄Π²ΠΈΠΆΠ½Ρ‹ΠΉ ΠΊΠΎΠ½Ρ‚Π°ΠΊΡ‚ Π‘, ΠΌΠΎΠΆΠ½ΠΎ ΡƒΠΌΠ΅Π½ΡŒΡˆΠ°Ρ‚ΡŒ ΠΈΠ»ΠΈ ΡƒΠ²Π΅Π»ΠΈΡ‡ΠΈΠ²Π°Ρ‚ΡŒ Π΄Π»ΠΈΠ½Ρƒ Π²ΠΊΠ»ΡŽΡ‡Ρ‘Π½Π½ΠΎΠ³ΠΎ Π² Ρ†Π΅ΠΏΡŒ участка АБ. ΠŸΡ€ΠΈ этом Π±ΡƒΠ΄Π΅Ρ‚ ΠΌΠ΅Π½ΡΡ‚ΡŒΡΡ сопротивлСниС Ρ†Π΅ΠΏΠΈ, Π° ΡΠ»Π΅Π΄ΠΎΠ²Π°Ρ‚Π΅Π»ΡŒΠ½ΠΎ, ΠΈ сила Ρ‚ΠΎΠΊΠ° Π² Π½Π΅ΠΉ, это ΠΏΠΎΠΊΠ°ΠΆΠ΅Ρ‚ Π°ΠΌΠΏΠ΅Ρ€ΠΌΠ΅Ρ‚Ρ€.


Β 

РСостатам, примСняСмым Π½Π° ΠΏΡ€Π°ΠΊΡ‚ΠΈΠΊΠ΅, ΠΏΡ€ΠΈΠ΄Π°ΡŽΡ‚ Π±ΠΎΠ»Π΅Π΅ ΡƒΠ΄ΠΎΠ±Π½ΡƒΡŽ ΠΈ ΠΊΠΎΠΌΠΏΠ°ΠΊΡ‚Π½ΡƒΡŽ Ρ„ΠΎΡ€ΠΌΡƒ. Для этой Ρ†Π΅Π»ΠΈ ΠΈΡΠΏΠΎΠ»ΡŒΠ·ΡƒΡŽΡ‚ ΠΏΡ€ΠΎΠ²ΠΎΠ»ΠΎΠΊΡƒ с большим ΡƒΠ΄Π΅Π»ΡŒΠ½Ρ‹ΠΌ сопротивлСниСм. Один ΠΈΠ· рСостатов (ΠΏΠΎΠ»Π·ΡƒΠ½ΠΊΠΎΠ²Ρ‹ΠΉ рСостат) ΠΈΠ·ΠΎΠ±Ρ€Π°ΠΆΡ‘Π½ Π½Π° рисункС.

Β 

Β 

Π’ этом рСостатС никСлиновая ΠΏΡ€ΠΎΠ²ΠΎΠ»ΠΎΠΊΠ° Π½Π°ΠΌΠΎΡ‚Π°Π½Π° Π½Π° кСрамичСский Ρ†ΠΈΠ»ΠΈΠ½Π΄Ρ€. ΠŸΡ€ΠΎΠ²ΠΎΠ»ΠΎΠΊΠ° ΠΏΠΎΠΊΡ€Ρ‹Ρ‚Π° Ρ‚ΠΎΠ½ΠΊΠΈΠΌ слоСм Π½Π΅ проводящСй Ρ‚ΠΎΠΊ ΠΎΠΊΠ°Π»ΠΈΠ½Ρ‹, поэтому Π²ΠΈΡ‚ΠΊΠΈ Π΅Ρ‘ ΠΈΠ·ΠΎΠ»ΠΈΡ€ΠΎΠ²Π°Π½Ρ‹ Π΄Ρ€ΡƒΠ³ ΠΎΡ‚ Π΄Ρ€ΡƒΠ³Π°. Над ΠΎΠ±ΠΌΠΎΡ‚ΠΊΠΎΠΉ располоТСн мСталличСский ΡΡ‚Π΅Ρ€ΠΆΠ΅Π½ΡŒ, ΠΏΠΎ ΠΊΠΎΡ‚ΠΎΡ€ΠΎΠΌΡƒ ΠΌΠΎΠΆΠ΅Ρ‚ ΠΏΠ΅Ρ€Π΅ΠΌΠ΅Ρ‰Π°Ρ‚ΡŒΡΡ ΠΏΠΎΠ»Π·ΡƒΠ½ΠΎΠΊ. Π‘Π²ΠΎΠΈΠΌΠΈ ΠΊΠΎΠ½Ρ‚Π°ΠΊΡ‚Π°ΠΌΠΈ ΠΎΠ½ ΠΏΡ€ΠΈΠΆΠ°Ρ‚ ΠΊ Π²ΠΈΡ‚ΠΊΠ°ΠΌ ΠΎΠ±ΠΌΠΎΡ‚ΠΊΠΈ. ΠžΡ‚ трСния ΠΏΠΎΠ»Π·ΡƒΠ½ΠΊΠ° ΠΎ Π²ΠΈΡ‚ΠΊΠΈ слой ΠΎΠΊΠ°Π»ΠΈΠ½Ρ‹ ΠΏΠΎΠ΄ Π΅Π³ΠΎ ΠΊΠΎΠ½Ρ‚Π°ΠΊΡ‚Π°ΠΌΠΈ стираСтся, ΠΈ элСктричСский Ρ‚ΠΎΠΊ Π² Ρ†Π΅ΠΏΠΈ ΠΏΡ€ΠΎΡ…ΠΎΠ΄ΠΈΡ‚ ΠΎΡ‚ Π²ΠΈΡ‚ΠΊΠΎΠ² ΠΏΡ€ΠΎΠ²ΠΎΠ»ΠΎΠΊΠΈ ΠΊ ΠΏΠΎΠ»Π·ΡƒΠ½ΠΊΡƒ, Π° Ρ‡Π΅Ρ€Π΅Π· Π½Π΅Π³ΠΎ Π² ΡΡ‚Π΅Ρ€ΠΆΠ΅Π½ΡŒ, ΠΈΠΌΠ΅ΡŽΡ‰ΠΈΠΉ Π½Π° ΠΊΠΎΠ½Ρ†Π΅ Π·Π°ΠΆΠΈΠΌ \(1\).19) ΠΈΠΌΠ΅ΡŽΡ‚ ΠΎΡ‡Π΅Π½ΡŒ большоС ΡƒΠ΄Π΅Π»ΡŒΠ½ΠΎΠ΅ сопротивлСниС ΠΈ ΠΏΠΎΡ‡Ρ‚ΠΈ Π½Π΅ проводят элСктричСский Ρ‚ΠΎΠΊ, ΠΈΡ… Β ΠΈΡΠΏΠΎΠ»ΡŒΠ·ΡƒΡŽΡ‚ для изоляторов.

РСостаты

РСостат — ΠΏΡ€ΠΈΠ±ΠΎΡ€, ΠΊΠΎΡ‚ΠΎΡ€Ρ‹ΠΉ ΠΈΡΠΏΠΎΠ»ΡŒΠ·ΡƒΠ΅Ρ‚ΡΡ для рСгулирования силы Ρ‚ΠΎΠΊΠ° Π² Ρ†Π΅ΠΏΠΈ.

Π‘Π°ΠΌΡ‹ΠΉ простой рСостат — ΠΏΡ€ΠΎΠ²ΠΎΠ»ΠΎΠΊΠ° с большим ΡƒΠ΄Π΅Π»ΡŒΠ½Ρ‹ΠΌ сопротивлСниСм , такая ΠΊΠ°ΠΊ никСлиновая ΠΈΠ»ΠΈ нихромовая.

Π’ΠΈΠ΄Ρ‹ рСостатов:

ΠŸΠΎΠ»Π·ΡƒΠ½ΠΊΠΎΠ²Ρ‹ΠΉ рСостат — Π΅Ρ‰Π΅ ΠΎΠ΄ΠΈΠ½ Π²ΠΈΠ΄ рСостатов , Π² ΠΊΠΎΡ‚ΠΎΡ€ΠΎΠΌ Β ΡΡ‚Π°Π»ΡŒΠ½Π°Ρ ΠΏΡ€ΠΎΠ²ΠΎΠ»ΠΎΠΊΠ° Π½Π°ΠΌΠΎΡ‚Π°Π½Π° Π½Π° кСрамичСский Ρ†ΠΈΠ»ΠΈΠ½Π΄Ρ€.ΠŸΡ€ΠΎΠ²ΠΎΠ»ΠΎΠΊΠ° ΠΏΠΎΠΊΡ€Ρ‹Ρ‚Π° Ρ‚ΠΎΠ½ΠΊΠΈΠΌ слоСм ΠΎΠΊΠ°Π»ΠΈΠ½Ρ‹ , которая Π½Π΅ ΠΏΡ€ΠΎΠ²ΠΎΠ΄ΠΈΡ‚ элСктричСский Ρ‚ΠΎΠΊ , поэтому Π΅Π΅ Π²ΠΈΡ‚ΠΊΠΈ ΠΈΠ·ΠΎΠ»ΠΈΡ€ΠΎΠ²Π°Π½Ρ‹ Π΄Ρ€ΡƒΠ³ ΠΎΡ‚ Π΄Ρ€ΡƒΠ³Π°.Над ΠΎΠ±ΠΌΠΎΡ‚ΠΊΠΎΠΉ — мСталличСский ΡΡ‚Π΅Ρ€ΠΆΠ΅Π½ΡŒ ΠΏΠΎ ΠΊΠΎΡ‚ΠΎΡ€ΠΎΠΌΡƒ пСрСмСщаСтся ΠΏΠΎΠ»Π·ΡƒΠ½ΠΎΠΊ .

Он ΠΏΡ€ΠΈΠΆΠ°Ρ‚ ΠΊ Π²ΠΈΡ‚ΠΊΠ°ΠΌ ΠΎΠ±ΠΌΠΎΡ‚ΠΊΠΈ.ΠžΡ‚ трСния ΠΏΠΎΠ»Π·ΡƒΠ½ΠΊΠ° ΠΎ Π²ΠΈΡ‚ΠΊΠΈ слой ΠΎΠΊΠ°Π»ΠΈΠ½Ρ‹ стираСтся ΠΈ элСктричСский Ρ‚ΠΎΠΊ Π² Ρ†Π΅ΠΏΠΈ ΠΏΡ€ΠΎΡ…ΠΎΠ΄ΠΈΡ‚ ΠΎΡ‚ Π²ΠΈΡ‚ΠΊΠΎΠ² ΠΏΡ€ΠΎΠ²ΠΎΠ»ΠΎΠΊΠΈ ΠΊ ΠΏΠΎΠ»Π·ΡƒΠ½ΠΊΡƒ, ΠΏΠΎΡ‚ΠΎΠΌ Π² ΡΡ‚Π΅Ρ€ΠΆΠ΅Π½ΡŒ.Когда рСостат ΠΏΠΎΠ΄ΠΊΠ»ΡŽΡ‡ΠΈΠ»ΠΈ Π² Ρ†Π΅ΠΏΡŒ , ΠΌΠΎΠΆΠ½ΠΎ ΠΏΠ΅Ρ€Π΅Π΄Π²ΠΈΠ³Π°Ρ‚ΡŒ ΠΏΠΎΠ»Π·ΡƒΠ½ΠΎΠΊ , Ρ‚Π°ΠΊΠΈΠΌ ΠΎΠ±Ρ€Π°Π·ΠΎΠΌ ΡƒΠ²Π΅Π»ΠΈΡ‡ΠΈΠ²Π°Ρ‚ΡŒ ΠΈΠ»ΠΈ ΡƒΠΌΠ΅Π½ΡŒΡˆΠ°Ρ‚ΡŒ сопротивлСниС рСостата.

Жидкостный рСостат — прСдставляСт Π±Π°ΠΊ с элСктролитом, Π² ΠΊΠΎΡ‚ΠΎΡ€Ρ‹ΠΉ ΠΏΠΎΠ³Ρ€ΡƒΠΆΠ°ΡŽΡ‚ΡΡ мСталличСскиС пластины.Β 

ΠŸΡ€ΠΎΠ²ΠΎΠ»ΠΎΡ‡Π½Ρ‹ΠΉ рСостат — cостоит ΠΈΠ· ΠΏΡ€ΠΎΠ²ΠΎΠ»ΠΎΠΊΠΈ ΠΈΠ· ΠΌΠ°Ρ‚Π΅Ρ€ΠΈΠ°Π»Π° Π² ΠΊΠΎΡ‚ΠΎΡ€ΠΎΠΌ высокоС ΡƒΠ΄Π΅Π»ΡŒΠ½ΠΎΠ΅ сопротивлСниС, натянутый Π½Π° Ρ€Π°ΠΌΡƒ.Β 

НСльзя ΠΏΡ€Π΅Π²Ρ‹ΡˆΠ°Ρ‚ΡŒ силу Ρ‚ΠΎΠΊΠ° рСостата, ΠΏΠΎΡ‚ΠΎΠΌΡƒ Ρ‡Ρ‚ΠΎ ΠΎΠ±ΠΌΠΎΡ‚ΠΊΠ° рСостата ΠΌΠΎΠΆΠ΅Ρ‚ ΠΏΠ΅Ρ€Π΅Π³ΠΎΡ€Π΅Ρ‚ΡŒ.

РСостат ΠΌΡ‹ часто примСняСм Π² повсСднСвной ΠΆΠΈΠ·Π½ΠΈ, Π½Π°ΠΏΡ€ΠΈΠΌΠ΅Ρ€, рСгулируя Π³Ρ€ΠΎΠΌΠΊΠΎΡΡ‚ΡŒ Ρ‚Π΅Π»Π΅Π²ΠΈΠ·ΠΎΡ€Π° ΠΈ Ρ€Π°Π΄ΠΈΠΎ, увСличивая ΠΈ ΡƒΠΌΠ΅Π½ΡŒΡˆΠ°Ρ ΡΠΊΠΎΡ€ΠΎΡΡ‚ΡŒ Π΅Π·Π΄Ρ‹ Π½Π° машинС.Β 

НуТна ΠΏΠΎΠΌΠΎΡ‰ΡŒ Π² ΡƒΡ‡Π΅Π±Π΅?



ΠŸΡ€Π΅Π΄Ρ‹Π΄ΡƒΡ‰Π°Ρ Ρ‚Π΅ΠΌΠ°: Π—Π°ΠΊΠΎΠ½ Ома для участка Ρ†Π΅ΠΏΠΈ: Ρ„ΠΎΡ€ΠΌΡƒΠ»ΠΈΡ€ΠΎΠ²ΠΊΠ° ΠΈ Ρ„ΠΎΡ€ΠΌΡƒΠ»Π°, ΠΏΡ€ΠΈΠΌΠ΅Π½Π΅Π½ΠΈΠ΅
Π‘Π»Π΅Π΄ΡƒΡŽΡ‰Π°Ρ Ρ‚Π΅ΠΌΠ°:&nbsp&nbsp&nbspΠŸΠΎΡΠ»Π΅Π΄ΠΎΠ²Π°Ρ‚Π΅Π»ΡŒΠ½ΠΎΠ΅ ΠΈ ΠΏΠ°Ρ€Π°Π»Π»Π΅Π»ΡŒΠ½ΠΎΠ΅ соСдинСниС ΠΏΡ€ΠΎΠ²ΠΎΠ΄Π½ΠΈΠΊΠΎΠ²

ЭлСктричСскоС сопротивлСниС | Π€ΠΈΠ·ΠΈΠΊΠ°

На рисункС 33 ΠΈΠ·ΠΎΠ±Ρ€Π°ΠΆΠ΅Π½Π° элСктричСская Ρ†Π΅ΠΏΡŒ, Π² ΠΊΠΎΡ‚ΠΎΡ€ΡƒΡŽ Π²ΠΊΠ»ΡŽΡ‡Π΅Π½Π° панСль с Ρ€Π°Π·Π½Ρ‹ΠΌΠΈ ΠΏΡ€ΠΎΠ²ΠΎΠ΄Π½ΠΈΠΊΠ°ΠΌΠΈ. Π­Ρ‚ΠΈ ΠΏΡ€ΠΎΠ²ΠΎΠ΄Π½ΠΈΠΊΠΈ ΠΎΡ‚Π»ΠΈΡ‡Π°ΡŽΡ‚ΡΡ Π΄Ρ€ΡƒΠ³ ΠΎΡ‚ Π΄Ρ€ΡƒΠ³Π° ΠΌΠ°Ρ‚Π΅Ρ€ΠΈΠ°Π»ΠΎΠΌ, Π° Ρ‚Π°ΠΊΠΆΠ΅ Π΄Π»ΠΈΠ½ΠΎΠΉ ΠΈ ΠΏΠ»ΠΎΡ‰Π°Π΄ΡŒΡŽ ΠΏΠΎΠΏΠ΅Ρ€Π΅Ρ‡Π½ΠΎΠ³ΠΎ сСчСния. ΠŸΠΎΠ΄ΠΊΠ»ΡŽΡ‡Π°Ρ ΠΏΠΎ ΠΎΡ‡Π΅Ρ€Π΅Π΄ΠΈ эти ΠΏΡ€ΠΎΠ²ΠΎΠ΄Π½ΠΈΠΊΠΈ ΠΈ наблюдая Π·Π° показаниями Π°ΠΌΠΏΠ΅Ρ€ΠΌΠ΅Ρ‚Ρ€Π°, ΠΌΠΎΠΆΠ½ΠΎ Π·Π°ΠΌΠ΅Ρ‚ΠΈΡ‚ΡŒ, Ρ‡Ρ‚ΠΎ ΠΏΡ€ΠΈ ΠΎΠ΄Π½ΠΎΠΌ ΠΈ Ρ‚ΠΎΠΌ ΠΆΠ΅ источникС Ρ‚ΠΎΠΊΠ° сила Ρ‚ΠΎΠΊΠ° Π² Ρ€Π°Π·Π½Ρ‹Ρ… случаях оказываСтся Ρ€Π°Π·Π»ΠΈΡ‡Π½ΠΎΠΉ. Π‘ ΡƒΠ²Π΅Π»ΠΈΡ‡Π΅Π½ΠΈΠ΅ΠΌ Π΄Π»ΠΈΠ½Ρ‹ ΠΏΡ€ΠΎΠ²ΠΎΠ΄Π½ΠΈΠΊΠ° ΠΈ ΡƒΠΌΠ΅Π½ΡŒΡˆΠ΅Π½ΠΈΠ΅ΠΌ Π΅Π³ΠΎ сСчСния сила Ρ‚ΠΎΠΊΠ° Π² Π½Π΅ΠΌ становится мСньшС. Π£ΠΌΠ΅Π½ΡŒΡˆΠ°Π΅Ρ‚ΡΡ ΠΎΠ½Π° ΠΈ ΠΏΡ€ΠΈ Π·Π°ΠΌΠ΅Π½Π΅ Π½ΠΈΠΊΠ΅Π»ΠΈΠ½ΠΎΠ²ΠΎΠΉ ΠΏΡ€ΠΎΠ²ΠΎΠ»ΠΎΠΊΠΈ ΠΏΡ€ΠΎΠ²ΠΎΠ»ΠΎΠΊΠΎΠΉ Ρ‚Π°ΠΊΠΎΠΉ ΠΆΠ΅ Π΄Π»ΠΈΠ½Ρ‹ ΠΈ сСчСния, Π½ΠΎ ΠΈΠ·Π³ΠΎΡ‚ΠΎΠ²Π»Π΅Π½Π½ΠΎΠΉ ΠΈΠ· Π½ΠΈΡ…Ρ€ΠΎΠΌΠ°. Π­Ρ‚ΠΎ ΠΎΠ·Π½Π°Ρ‡Π°Π΅Ρ‚, Ρ‡Ρ‚ΠΎ Ρ€Π°Π·Π½Ρ‹Π΅ ΠΏΡ€ΠΎΠ²ΠΎΠ΄Π½ΠΈΠΊΠΈ ΠΎΠΊΠ°Π·Ρ‹Π²Π°ΡŽΡ‚ Ρ€Π°Π·Π»ΠΈΡ‡Π½ΠΎΠ΅ противодСйствиС Ρ‚ΠΎΠΊΡƒ. ΠŸΡ€ΠΎΡ‚ΠΈΠ²ΠΎΠ΄Π΅ΠΉΡΡ‚Π²ΠΈΠ΅ это Π²ΠΎΠ·Π½ΠΈΠΊΠ°Π΅Ρ‚ ΠΈΠ·-Π·Π° столкновСний носитСлСй Ρ‚ΠΎΠΊΠ° со встрСчными частицами вСщСства.

ЀизичСская Π²Π΅Π»ΠΈΡ‡ΠΈΠ½Π°, Ρ…Π°Ρ€Π°ΠΊΡ‚Π΅Ρ€ΠΈΠ·ΡƒΡŽΡ‰Π°Ρ противодСйствиС, ΠΎΠΊΠ°Π·Ρ‹Π²Π°Π΅ΠΌΠΎΠ΅ ΠΏΡ€ΠΎΠ²ΠΎΠ΄Π½ΠΈΠΊΠΎΠΌ элСктричСскому Ρ‚ΠΎΠΊΡƒ, обозначаСтся Π±ΡƒΠΊΠ²ΠΎΠΉ R ΠΈ называСтся элСктричСским сопротивлСниСм (ΠΈΠ»ΠΈ просто сопротивлСниСм

) ΠΏΡ€ΠΎΠ²ΠΎΠ΄Π½ΠΈΠΊΠ°:

R β€” сопротивлСниС.

Π•Π΄ΠΈΠ½ΠΈΡ†Π° сопротивлСния называСтся ΠΎΠΌΠΎΠΌ (Ом) Π² Ρ‡Π΅ΡΡ‚ΡŒ Π½Π΅ΠΌΠ΅Ρ†ΠΊΠΎΠ³ΠΎ ΡƒΡ‡Π΅Π½ΠΎΠ³ΠΎ Π“. Ома, ΠΊΠΎΡ‚ΠΎΡ€Ρ‹ΠΉ Π²ΠΏΠ΅Ρ€Π²Ρ‹Π΅ Π²Π²Π΅Π» это понятиС Π² Ρ„ΠΈΠ·ΠΈΠΊΡƒ. 1 Ом β€” это сопротивлСниС Ρ‚Π°ΠΊΠΎΠ³ΠΎ ΠΏΡ€ΠΎΠ²ΠΎΠ΄Π½ΠΈΠΊΠ°, Π² ΠΊΠΎΡ‚ΠΎΡ€ΠΎΠΌ ΠΏΡ€ΠΈ напряТСнии 1 Π’ сила Ρ‚ΠΎΠΊΠ° Ρ€Π°Π²Π½Π° 1 А. ΠŸΡ€ΠΈ сопротивлСнии 2 Ом сила Ρ‚ΠΎΠΊΠ° ΠΏΡ€ΠΈ Ρ‚ΠΎΠΌ ΠΆΠ΅ напряТСнии Π±ΡƒΠ΄Π΅Ρ‚ Π² 2 Ρ€Π°Π·Π° мСньшС, ΠΏΡ€ΠΈ сопротивлСнии 3 Ом β€” Π² 3 Ρ€Π°Π·Π° мСньшС ΠΈ Ρ‚. Π΄.

На ΠΏΡ€Π°ΠΊΡ‚ΠΈΠΊΠ΅ Π²ΡΡ‚Ρ€Π΅Ρ‡Π°ΡŽΡ‚ΡΡ ΠΈ Π΄Ρ€ΡƒΠ³ΠΈΠ΅ Π΅Π΄ΠΈΠ½ΠΈΡ†Ρ‹ сопротивлСния, Π½Π°ΠΏΡ€ΠΈΠΌΠ΅Ρ€ ΠΊΠΈΠ»ΠΎΠΎΠΌ (кОм) ΠΈ ΠΌΠ΅Π³Π°ΠΎΠΌ (МОм):

1 кОм= 1000 Ом, 1 МОм= 1 000 ООО Ом.

Π‘ΠΎΠΏΡ€ΠΎΡ‚ΠΈΠ²Π»Π΅Π½ΠΈΠ΅ ΠΎΠ΄Π½ΠΎΡ€ΠΎΠ΄Π½ΠΎΠ³ΠΎ ΠΏΡ€ΠΎΠ²ΠΎΠ΄Π½ΠΈΠΊΠ° постоянного сСчСния зависит ΠΎΡ‚ ΠΌΠ°Ρ‚Π΅Ρ€ΠΈΠ°Π»Π° ΠΏΡ€ΠΎΠ²ΠΎΠ΄Π½ΠΈΠΊΠ°, Π΅Π³ΠΎ Π΄Π»ΠΈΠ½Ρ‹ l ΠΈ ΠΏΠ»ΠΎΡ‰Π°Π΄ΠΈ ΠΏΠΎΠΏΠ΅Ρ€Π΅Ρ‡Π½ΠΎΠ³ΠΎ сСчСния S ΠΈ ΠΌΠΎΠΆΠ΅Ρ‚ Π±Ρ‹Ρ‚ΡŒ Π½Π°ΠΉΠ΄Π΅Π½ΠΎ ΠΏΠΎ Ρ„ΠΎΡ€ΠΌΡƒΠ»Π΅

R = ρl/S      (12.1)

Π³Π΄Π΅ ρ β€” ΡƒΠ΄Π΅Π»ΡŒΠ½ΠΎΠ΅ сопротивлСниС вСщСства, ΠΈΠ· ΠΊΠΎΡ‚ΠΎΡ€ΠΎΠ³ΠΎ ΠΈΠ·Π³ΠΎΡ‚ΠΎΠ²Π»Π΅Π½ ΠΏΡ€ΠΎΠ²ΠΎΠ΄Π½ΠΈΠΊ.

УдСльноС сопротивлСниС вСщСства β€” это физичСская Π²Π΅Π»ΠΈΡ‡ΠΈΠ½Π°, ΠΏΠΎΠΊΠ°Π·Ρ‹Π²Π°ΡŽΡ‰Π°Ρ, ΠΊΠ°ΠΊΠΈΠΌ сопротивлСниСм ΠΎΠ±Π»Π°Π΄Π°Π΅Ρ‚ сдСланный ΠΈΠ· этого вСщСства ΠΏΡ€ΠΎΠ²ΠΎΠ΄Π½ΠΈΠΊ Π΅Π΄ΠΈΠ½ΠΈΡ‡Π½ΠΎΠΉ Π΄Π»ΠΈΠ½Ρ‹ ΠΈ Π΅Π΄ΠΈΠ½ΠΈΡ‡Π½ΠΎΠΉ ΠΏΠ»ΠΎΡ‰Π°Π΄ΠΈ ΠΏΠΎΠΏΠ΅Ρ€Π΅Ρ‡Π½ΠΎΠ³ΠΎ сСчСния.

Из Ρ„ΠΎΡ€ΠΌΡƒΠ»Ρ‹ (12.1) слСдуСт, Ρ‡Ρ‚ΠΎ

ρ = RS/l

Π’Π°ΠΊ ΠΊΠ°ΠΊ Π² БИ Π΅Π΄ΠΈΠ½ΠΈΡ†Π΅ΠΉ сопротивлСния являСтся 1 Ом, Π΅Π΄ΠΈΠ½ΠΈΡ†Π΅ΠΉ ΠΏΠ»ΠΎΡ‰Π°Π΄ΠΈ 1 ΠΌ2, Π° Π΅Π΄ΠΈΠ½ΠΈΡ†Π΅ΠΉ Π΄Π»ΠΈΠ½Ρ‹ 1 ΠΌ, Ρ‚ΠΎ Π΅Π΄ΠΈΠ½ΠΈΡ†Π΅ΠΉ ΡƒΠ΄Π΅Π»ΡŒΠ½ΠΎΠ³ΠΎ сопротивлСния Π² БИ Π±ΡƒΠ΄Π΅Ρ‚

1 Ом · м2/м, или 1 Ом · м.

На ΠΏΡ€Π°ΠΊΡ‚ΠΈΠΊΠ΅ ΠΏΠ»ΠΎΡ‰Π°Π΄ΡŒ сСчСния Ρ‚ΠΎΠ½ΠΊΠΈΡ… ΠΏΡ€ΠΎΠ²ΠΎΠ΄ΠΎΠ² часто Π²Ρ‹Ρ€Π°ΠΆΠ°ΡŽΡ‚ Π² ΠΊΠ²Π°Π΄Ρ€Π°Ρ‚Π½Ρ‹Ρ… ΠΌΠΈΠ»Π»ΠΈΠΌΠ΅Ρ‚Ρ€Π°Ρ… (ΠΌΠΌ2). Π’ этом случаС Π±ΠΎΠ»Π΅Π΅ ΡƒΠ΄ΠΎΠ±Π½ΠΎΠΉ Π΅Π΄ΠΈΠ½ΠΈΡ†Π΅ΠΉ ΡƒΠ΄Π΅Π»ΡŒΠ½ΠΎΠ³ΠΎ сопротивлСния являСтся Ом·мм2/ΠΌ. Π’Π°ΠΊ ΠΊΠ°ΠΊ 1 ΠΌΠΌ2 = 0,000001 ΠΌ2, Ρ‚ΠΎ

1 Ом · мм2/м = 0,000001 Ом · м.

Π£ Ρ€Π°Π·Π½Ρ‹Ρ… вСщСств ΡƒΠ΄Π΅Π»ΡŒΠ½Ρ‹Π΅ сопротивлСния Ρ€Π°Π·Π»ΠΈΡ‡Π½Ρ‹. НСкоторыС ΠΈΠ· Π½ΠΈΡ… ΠΏΡ€ΠΈΠ²Π΅Π΄Π΅Π½Ρ‹ Π² Ρ‚Π°Π±Π»ΠΈΡ†Π΅ 3.

ΠŸΡ€ΠΈΠ²Π΅Π΄Π΅Π½Π½Ρ‹Π΅ Π² этой Ρ‚Π°Π±Π»ΠΈΡ†Π΅ значСния ΡΠΎΠΎΡ‚Π²Π΅Ρ‚ΡΡ‚Π²ΡƒΡŽΡ‚ Ρ‚Π΅ΠΌΠΏΠ΅Ρ€Π°Ρ‚ΡƒΡ€Π΅ 20 Β°Π‘. (Π‘ ΠΈΠ·ΠΌΠ΅Π½Π΅Π½ΠΈΠ΅ΠΌ Ρ‚Π΅ΠΌΠΏΠ΅Ρ€Π°Ρ‚ΡƒΡ€Ρ‹ сопротивлСниС вСщСства измСняСтся.) НапримСр, ΡƒΠ΄Π΅Π»ΡŒΠ½ΠΎΠ΅ сопротивлСниС ΠΆΠ΅Π»Π΅Π·Π° Ρ€Π°Π²Π½ΠΎ 0,1 Ом Β· ΠΌΠΌ2/ΠΌ. Π­Ρ‚ΠΎ ΠΎΠ·Π½Π°Ρ‡Π°Π΅Ρ‚, Ρ‡Ρ‚ΠΎ Ссли ΠΈΠ·Π³ΠΎΡ‚ΠΎΠ²ΠΈΡ‚ΡŒ ΠΈΠ· ΠΆΠ΅Π»Π΅Π·Π° ΠΏΡ€ΠΎΠ²ΠΎΠ΄ с ΠΏΠ»ΠΎΡ‰Π°Π΄ΡŒΡŽ сСчСния 1 ΠΌΠΌ2 ΠΈ Π΄Π»ΠΈΠ½ΠΎΠΉ 1 ΠΌ, Ρ‚ΠΎ ΠΏΡ€ΠΈ Ρ‚Π΅ΠΌΠΏΠ΅Ρ€Π°Ρ‚ΡƒΡ€Π΅ 20 Β°Π‘ ΠΎΠ½ Π±ΡƒΠ΄Π΅Ρ‚ ΠΎΠ±Π»Π°Π΄Π°Ρ‚ΡŒ сопротивлСниСм 0,1 Ом.

Из Ρ‚Π°Π±Π»ΠΈΡ†Ρ‹ 3 Π²ΠΈΠ΄Π½ΠΎ, Ρ‡Ρ‚ΠΎ наимСньшим ΡƒΠ΄Π΅Π»ΡŒΠ½Ρ‹ΠΌ сопротивлСниСм ΠΎΠ±Π»Π°Π΄Π°ΡŽΡ‚ сСрСбро ΠΈ мСдь. Π—Π½Π°Ρ‡ΠΈΡ‚, ΠΈΠΌΠ΅Π½Π½ΠΎ эти ΠΌΠ΅Ρ‚Π°Π»Π»Ρ‹ ΡΠ²Π»ΡΡŽΡ‚ΡΡ Π½Π°ΠΈΠ»ΡƒΡ‡ΡˆΠΈΠΌΠΈ ΠΏΡ€ΠΎΠ²ΠΎΠ΄Π½ΠΈΠΊΠ°ΠΌΠΈ элСктричСства.

Из Ρ‚ΠΎΠΉ ΠΆΠ΅ Ρ‚Π°Π±Π»ΠΈΡ†Ρ‹ Π²ΠΈΠ΄Π½ΠΎ, Ρ‡Ρ‚ΠΎ, Π½Π°ΠΎΠ±ΠΎΡ€ΠΎΡ‚, Ρ‚Π°ΠΊΠΈΠ΅ вСщСства, ΠΊΠ°ΠΊ Ρ„Π°Ρ€Ρ„ΠΎΡ€ ΠΈ эбонит, ΠΎΠ±Π»Π°Π΄Π°ΡŽΡ‚ ΠΎΡ‡Π΅Π½ΡŒ большим ΡƒΠ΄Π΅Π»ΡŒΠ½Ρ‹ΠΌ сопротивлСниСм. Π­Ρ‚ΠΎ ΠΈ позволяСт ΠΈΡΠΏΠΎΠ»ΡŒΠ·ΠΎΠ²Π°Ρ‚ΡŒ ΠΈΡ… Π² качСствС изоляторов.

??? 1. Π§Ρ‚ΠΎ Ρ…Π°Ρ€Π°ΠΊΡ‚Π΅Ρ€ΠΈΠ·ΡƒΠ΅Ρ‚ ΠΈ ΠΊΠ°ΠΊ обозначаСтся элСктричСскоС сопротивлСниС? 2. По ΠΊΠ°ΠΊΠΎΠΉ Ρ„ΠΎΡ€ΠΌΡƒΠ»Π΅ находится сопротивлСниС ΠΏΡ€ΠΎΠ²ΠΎΠ΄Π½ΠΈΠΊΠ°? 3. Как называСтся Π΅Π΄ΠΈΠ½ΠΈΡ†Π° сопротивлСния? 4. Π§Ρ‚ΠΎ ΠΏΠΎΠΊΠ°Π·Ρ‹Π²Π°Π΅Ρ‚ ΡƒΠ΄Π΅Π»ΡŒΠ½ΠΎΠ΅ сопротивлСниС? Какой Π±ΡƒΠΊΠ²ΠΎΠΉ ΠΎΠ½ΠΎ обозначаСтся? 5. Π’ ΠΊΠ°ΠΊΠΈΡ… Π΅Π΄ΠΈΠ½ΠΈΡ†Π°Ρ… ΠΈΠ·ΠΌΠ΅Ρ€ΡΡŽΡ‚ ΡƒΠ΄Π΅Π»ΡŒΠ½ΠΎΠ΅ сопротивлСниС? 6. Π˜ΠΌΠ΅ΡŽΡ‚ΡΡ Π΄Π²Π° ΠΏΡ€ΠΎΠ²ΠΎΠ΄Π½ΠΈΠΊΠ°. Π£ ΠΊΠ°ΠΊΠΎΠ³ΠΎ ΠΈΠ· Π½ΠΈΡ… большС сопротивлСниС, Ссли ΠΎΠ½ΠΈ: Π°) ΠΈΠΌΠ΅ΡŽΡ‚ ΠΎΠ΄ΠΈΠ½Π°ΠΊΠΎΠ²ΡƒΡŽ Π΄Π»ΠΈΠ½Ρƒ ΠΈ ΠΏΠ»ΠΎΡ‰Π°Π΄ΡŒ сСчСния, Π½ΠΎ ΠΎΠ΄ΠΈΠ½ ΠΈΠ· Π½ΠΈΡ… сдСлан ΠΈΠ· константана, Π° Π΄Ρ€ΡƒΠ³ΠΎΠΉ β€” ΠΈΠ· фСхраля; Π±) сдСланы ΠΈΠ· ΠΎΠ΄Π½ΠΎΠ³ΠΎ ΠΈ Ρ‚ΠΎΠ³ΠΎ ΠΆΠ΅ вСщСства, ΠΈΠΌΠ΅ΡŽΡ‚ ΠΎΠ΄ΠΈΠ½Π°ΠΊΠΎΠ²ΡƒΡŽ Ρ‚ΠΎΠ»Ρ‰ΠΈΠ½Ρƒ, Π½ΠΎ ΠΎΠ΄ΠΈΠ½ ΠΈΠ· Π½ΠΈΡ… Π² 2 Ρ€Π°Π·Π° Π΄Π»ΠΈΠ½Π½Π΅Π΅ Π΄Ρ€ΡƒΠ³ΠΎΠ³ΠΎ; Π²) сдСланы ΠΈΠ· ΠΎΠ΄Π½ΠΎΠ³ΠΎ ΠΈ Ρ‚ΠΎΠ³ΠΎ ΠΆΠ΅ вСщСства, ΠΈΠΌΠ΅ΡŽΡ‚ ΠΎΠ΄ΠΈΠ½Π°ΠΊΠΎΠ²ΡƒΡŽ Π΄Π»ΠΈΠ½Ρƒ, Π½ΠΎ ΠΎΠ΄ΠΈΠ½ ΠΈΠ· Π½ΠΈΡ… Π² 2 Ρ€Π°Π·Π° Ρ‚ΠΎΠ½ΡŒΡˆΠ΅ Π΄Ρ€ΡƒΠ³ΠΎΠ³ΠΎ? 7. ΠŸΡ€ΠΎΠ²ΠΎΠ΄Π½ΠΈΠΊΠΈ, рассматриваСмыС Π² ΠΏΡ€Π΅Π΄Ρ‹Π΄ΡƒΡ‰Π΅ΠΌ вопросС, ΠΏΠΎΠΎΡ‡Π΅Ρ€Π΅Π΄Π½ΠΎ ΠΏΠΎΠ΄ΠΊΠ»ΡŽΡ‡Π°ΡŽΡ‚ ΠΊ ΠΎΠ΄Π½ΠΎΠΌΡƒ ΠΈ Ρ‚ΠΎΠΌΡƒ ΠΆΠ΅ источнику Ρ‚ΠΎΠΊΠ°. Π’ ΠΊΠ°ΠΊΠΎΠΌ случаС сила Ρ‚ΠΎΠΊΠ° Π±ΡƒΠ΄Π΅Ρ‚ большС, Π² ΠΊΠ°ΠΊΠΎΠΌ мСньшС? ΠŸΡ€ΠΎΠ²Π΅Π΄ΠΈΡ‚Π΅ сравнСниС для ΠΊΠ°ΠΆΠ΄ΠΎΠΉ ΠΏΠ°Ρ€Ρ‹ рассматриваСмых ΠΏΡ€ΠΎΠ²ΠΎΠ΄Π½ΠΈΠΊΠΎΠ².

Расчёт сопротивлСния ΠΏΡ€ΠΎΠ²ΠΎΠ΄Π½ΠΈΠΊΠ°. УдСльноС сопротивлСниС.

ЦСль: ΠΈΡΡΠ»Π΅Π΄ΠΎΠ²Π°Ρ‚ΡŒ Π·Π°Π²ΠΈΡΠΈΠΌΠΎΡΡ‚ΡŒ сопротивлСния ΠΏΡ€ΠΎΠ²ΠΎΠ΄Π½ΠΈΠΊΠ° ΠΎΡ‚ Π΅Π³ΠΎ характСристик.

Π—Π°Π΄Π°Ρ‡ΠΈ

ΠΎΠ±ΡƒΡ‡Π°ΡŽΡ‰ΠΈΠ΅:

  • ΠΈΡΡΠ»Π΅Π΄ΠΎΠ²Π°Ρ‚ΡŒ Π·Π°Π²ΠΈΡΠΈΠΌΠΎΡΡ‚ΡŒ сопротивлСния ΠΏΡ€ΠΎΠ²ΠΎΠ΄Π½ΠΈΠΊΠ° ΠΎΡ‚ Π΅Π³ΠΎ Π΄Π»ΠΈΠ½Ρ‹, ΠΏΠ»ΠΎΡ‰Π°Π΄ΠΈ ΠΏΠΎΠΏΠ΅Ρ€Π΅Ρ‡Π½ΠΎΠ³ΠΎ сСчСния ΠΈ вСщСства, ΠΈΠ· ΠΊΠΎΡ‚ΠΎΡ€ΠΎΠ³ΠΎ ΠΎΠ½ ΠΈΠ·Π³ΠΎΡ‚ΠΎΠ²Π»Π΅Π½;
  • ΡΡ„ΠΎΡ€ΠΌΠΈΡ€ΠΎΠ²Π°Ρ‚ΡŒ ΠΏΠ΅Ρ€Π²ΠΈΡ‡Π½Ρ‹Π΅ прСдставлСния Π·Π½Π°Π½ΠΈΠΉ ΠΎ Π½ΠΎΠ²ΠΎΠΉ физичСской Π²Π΅Π»ΠΈΡ‡ΠΈΠ½Π΅ Β« ΡƒΠ΄Π΅Π»ΡŒΠ½ΠΎΠ΅ элСктричСскоС сопротивлСниС»;
  • ΠΏΡ€ΠΎΠ΄ΠΎΠ»ΠΆΠΈΡ‚ΡŒ Ρ„ΠΎΡ€ΠΌΠΈΡ€ΠΎΠ²Π°Π½ΠΈΠ΅ ΡƒΠΌΠ΅Π½ΠΈΠΉ Ρ€Π΅ΡˆΠ°Ρ‚ΡŒ Π·Π°Π΄Π°Ρ‡ΠΈ;

Ρ€Π°Π·Π²ΠΈΠ²Π°ΡŽΡ‰ΠΈΠ΅:

  • Ρ€Π°Π±ΠΎΡ‚Π°Ρ‚ΡŒ Π½Π°Π΄ Ρ„ΠΎΡ€ΠΌΠΈΡ€ΠΎΠ²Π°Π½ΠΈΠ΅ΠΌ ΠΈΡΡΠ»Π΅Π΄ΠΎΠ²Π°Ρ‚Π΅Π»ΡŒΡΠΊΠΈΡ… ΠΊΠΎΠΌΠΏΠ΅Ρ‚Π΅Π½Ρ†ΠΈΠΉ учащихся ΠΏΡƒΡ‚Π΅ΠΌ ΠΎΡ€Π³Π°Π½ΠΈΠ·Π°Ρ†ΠΈΠΈ Ρ„Ρ€ΠΎΠ½Ρ‚Π°Π»ΡŒΠ½ΠΎΠ³ΠΎ Π²ΠΈΡ€Ρ‚ΡƒΠ°Π»ΡŒΠ½ΠΎΠ³ΠΎ ΠΌΠΈΠ½ΠΈ-исслСдования с использованиСм элСктронных рСсурсов;
  • Ρ€Π°Π±ΠΎΡ‚Π°Ρ‚ΡŒ Π½Π°Π΄ Ρ„ΠΎΡ€ΠΌΠΈΡ€ΠΎΠ²Π°Π½ΠΈΠ΅ΠΌ ΡƒΠΌΠ΅Π½ΠΈΠΉ учащихся Π²ΠΎΡΠΏΡ€ΠΈΠ½ΠΈΠΌΠ°Ρ‚ΡŒ ΠΈ ΠΏΡ€Π΅Π΄ΡΡ‚Π°Π²Π»ΡΡ‚ΡŒ ΠΈΠ½Ρ„ΠΎΡ€ΠΌΠ°Ρ†ΠΈΡŽ Π² словСсной ΠΈ символичСской Ρ„ΠΎΡ€ΠΌΠ°Ρ… Ρ‡Π΅Ρ€Π΅Π· обсуТдСниС Ρ€Π΅Π·ΡƒΠ»ΡŒΡ‚Π°Ρ‚ΠΎΠ² дСмонстрационного экспСримСнта ΠΈ ΡΠ°ΠΌΠΎΡΡ‚ΠΎΡΡ‚Π΅Π»ΡŒΠ½Ρ‹Ρ… Π²ΠΈΡ€Ρ‚ΡƒΠ°Π»ΡŒΠ½Ρ‹Ρ… экспСримСнтов;
  • Ρ„ΠΎΡ€ΠΌΠΈΡ€ΠΎΠ²Π°Ρ‚ΡŒ умСния Π΄Π΅Π»Π°Ρ‚ΡŒ Π²Ρ‹Π²ΠΎΠ΄Ρ‹ Π½Π° основС ΠΏΡ€ΠΎΠ²Π΅Π΄Π΅Π½Π½ΠΎΠ³ΠΎ Π°Π½Π°Π»ΠΈΠ·Π°;
  • Ρ€Π°Π±ΠΎΡ‚Π°Ρ‚ΡŒ Π½Π°Π΄ Ρ„ΠΎΡ€ΠΌΠΈΡ€ΠΎΠ²Π°Π½ΠΈΠ΅ΠΌ ΠΊΠΎΠΌΠΌΡƒΠ½ΠΈΠΊΠ°Ρ‚ΠΈΠ²Π½Ρ‹Ρ… ΠΊΠΎΠΌΠΏΠ΅Ρ‚Π΅Π½Ρ†ΠΈΠΉ учащихся;

Π²ΠΎΡΠΏΠΈΡ‚Π°Ρ‚Π΅Π»ΡŒΠ½Ρ‹Π΅:

  • Π·Π½Π°ΠΊΠΎΠΌΠΈΡ‚ΡŒ с ΡΠΊΡΠΏΠ΅Ρ€ΠΈΠΌΠ΅Π½Ρ‚Π°Π»ΡŒΠ½Ρ‹ΠΌ ΠΌΠ΅Ρ‚ΠΎΠ΄ΠΎΠΌ Π½Π°ΡƒΡ‡Π½ΠΎΠ³ΠΎ познания ΠΏΡ€ΠΈΡ€ΠΎΠ΄Ρ‹; ΡΠΎΠ·Π΄Π°Ρ‚ΡŒ условия для развития ΡΠ°ΠΌΠΎΡΡ‚ΠΎΡΡ‚Π΅Π»ΡŒΠ½ΠΎΡΡ‚ΠΈ учащихся;
  • Ρ€Π°Π·Π²ΠΈΠ²Π°Ρ‚ΡŒ ΠΏΠΎΠ·Π½Π°Π²Π°Ρ‚Π΅Π»ΡŒΠ½Ρ‹ΠΉ интСрСс учащихся ΠΊ ΠΏΡ€Π΅Π΄ΠΌΠ΅Ρ‚Ρƒ.

Π’ΠΈΠΏ ΡƒΡ€ΠΎΠΊΠ°: ΠΊΠΎΠΌΠ±ΠΈΠ½ΠΈΡ€ΠΎΠ²Π°Π½Π½Ρ‹ΠΉ.

Π€ΠΎΡ€ΠΌΡ‹ Ρ€Π°Π±ΠΎΡ‚Ρ‹ учащихся:

  • групповая (ΠΈΡΡΠ»Π΅Π΄ΠΎΠ²Π°Ρ‚Π΅Π»ΡŒΡΠΊΠ°Ρ ΡΠ°ΠΌΠΎΡΡ‚ΠΎΡΡ‚Π΅Π»ΡŒΠ½Π°Ρ Ρ€Π°Π±ΠΎΡ‚Π° с элСктронными рСсурсами)
  • ΠΊΠΎΠ»Π»Π΅ΠΊΡ‚ΠΈΠ²Π½ΠΎΠ΅ обсуТдСниС Ρ€Π΅Π·ΡƒΠ»ΡŒΡ‚Π°Ρ‚ΠΎΠ² Π²ΠΈΡ€Ρ‚ΡƒΠ°Π»ΡŒΠ½Ρ‹Ρ… экспСримСнтов;
  • ΠΈΠ½Π΄ΠΈΠ²ΠΈΠ΄ΡƒΠ°Π»ΡŒΠ½Π°Ρ (Ρ‚Π΅ΠΊΡƒΡ‰ΠΈΠΉ ΠΊΠΎΠ½Ρ‚Ρ€ΠΎΠ»ΡŒ, ΡΠ°ΠΌΠΎΡΡ‚ΠΎΡΡ‚Π΅Π»ΡŒΠ½Π°Ρ Ρ€Π°Π±ΠΎΡ‚Π° с ЭОР К-Ρ‚ΠΈΠΏΠ°)

ΠœΠ΅Ρ‚ΠΎΠ΄Ρ‹ обучСния, ΠΈΡΠΏΠΎΠ»ΡŒΠ·ΡƒΠ΅ΠΌΡ‹Π΅ Π½Π° ΡƒΡ€ΠΎΠΊΠ΅: словСсныС, наглядныС, практичСскиС.

ДСмонстрационноС ΠΎΠ±ΠΎΡ€ΡƒΠ΄ΠΎΠ²Π°Π½ΠΈΠ΅: источник питания Π’Π‘-24М, Π»Π°ΠΌΠΏΠΎΡ‡ΠΊΠΈ Π½Π° 3Π’ ΠΈ 12 Π’, ΠΊΠ»ΡŽΡ‡, Ρ†ΠΈΡ„Ρ€ΠΎΠ²ΠΎΠΉ Π°ΠΌΠΏΠ΅Ρ€ΠΌΠ΅Ρ‚Ρ€, Ρ€Π΅ΠΎΡ…ΠΎΡ€Π΄, ΡΠΎΠ΅Π΄ΠΈΠ½ΠΈΡ‚Π΅Π»ΡŒΠ½Ρ‹Π΅ ΠΏΡ€ΠΎΠ²ΠΎΠ΄Π°.

БрСдства ИКВ: ПК (для учитСля), Π²ΠΈΠ΄Π΅ΠΎΠΏΡ€ΠΎΠ΅ΠΊΡ‚ΠΎΡ€, интСрактивная доска, Π½Π΅Ρ‚Π±ΡƒΠΊ (Π½Π° ΠΈΠ½Π΄ΠΈΠ²ΠΈΠ΄ΡƒΠ°Π»ΡŒΠ½ΠΎΠΌ Ρ€Π°Π±ΠΎΡ‡Π΅ΠΌ мСстС ΠΊΠ°ΠΆΠ΄ΠΎΠ³ΠΎ учащСгося). ΠŸΡ€Π΅Π·Π΅Π½Ρ‚Π°Ρ†ΠΈΡ SMART Notebook ΠΏΠΎ Ρ‚Π΅ΠΌΠ΅ «РасчСт сопротивлСния ΠΏΡ€ΠΎΠ²ΠΎΠ΄Π½ΠΈΠΊΠΎΠ². УдСльноС сопротивлСниС».

Π‘Ρ‚Ρ€ΡƒΠΊΡ‚ΡƒΡ€Π° ΠΈ Ρ…ΠΎΠ΄ ΡƒΡ€ΠΎΠΊΠ°

1.ΠžΡ€Π³Π°Π½ΠΈΠ·Π°Ρ†ΠΈΠΎΠ½Π½Ρ‹ΠΉ этап 

Π”Π΅ΡΡ‚Π΅Π»ΡŒΠ½ΠΎΡΡ‚ΡŒ учитСля

Π”Π΅ΡΡ‚Π΅Π»ΡŒΠ½ΠΎΡΡ‚ΡŒ ΡƒΡ‡Π΅Π½ΠΈΠΊΠ°

ΠŸΡ€ΠΈΠ²Π΅Ρ‚ΡΡ‚Π²ΡƒΠ΅Ρ‚ учащихся.

ΠŸΡ€ΠΎΠ²Π΅Ρ€ΡΠ΅Ρ‚ Π³ΠΎΡ‚ΠΎΠ²Π½ΠΎΡΡ‚ΡŒ учащихся ΠΊ Π·Π°Π½ΡΡ‚ΠΈΡŽ.

ΠŸΡ€ΠΈΠ²Π΅Ρ‚ΡΡ‚Π²ΡƒΡŽΡ‚ учитСля.

ΠŸΡ€ΠΎΠ²Π΅Ρ€ΡΡŽΡ‚ Π³ΠΎΡ‚ΠΎΠ²Π½ΠΎΡΡ‚ΡŒ ΠΊ ΡƒΡ€ΠΎΠΊΡƒ.

2. ΠŸΡ€ΠΎΠ²Π΅Ρ€ΠΊΠ° домашнСго задания

Π”Π΅ΡΡ‚Π΅Π»ΡŒΠ½ΠΎΡΡ‚ΡŒ учитСля

Π”Π΅ΡΡ‚Π΅Π»ΡŒΠ½ΠΎΡΡ‚ΡŒ ΡƒΡ‡Π΅Π½ΠΈΠΊΠ°

ΠŸΡ€ΠΎΠ²ΠΎΠ΄ΠΈΡ‚ ΠΏΡ€ΠΎΠ²Π΅Ρ€ΠΊΡƒ домашнСго задания.

(ΠŸΡ€ΠΈΠ»ΠΎΠΆΠ΅Π½ΠΈΠ΅ 1)

Π’Ρ‹ΠΏΠΎΠ»Π½ΡΡŽΡ‚ тСст (Π²Π°Ρ€ΠΈΠ°Π½Ρ‚Ρ‹ Ρ€Π°Π·Π½ΠΎΠ³ΠΎ уровня слоТности)

3. Актуализация Π·Π½Π°Π½ΠΈΠΉΒ 

Π”Π΅ΡΡ‚Π΅Π»ΡŒΠ½ΠΎΡΡ‚ΡŒ учитСля

Π”Π΅ΡΡ‚Π΅Π»ΡŒΠ½ΠΎΡΡ‚ΡŒ ΡƒΡ‡Π΅Π½ΠΈΠΊΠ°

Π“ΠΎΡ‚ΠΎΠ²ΠΈΡ‚ учащихся ΠΊ Π²ΠΎΡΠΏΡ€ΠΈΡΡ‚ΠΈΡŽ Π½ΠΎΠ²Ρ‹Ρ… Π·Π½Π°Π½ΠΈΠΉ.

ΠŸΡ€Π΅Π΄Π»Π°Π³Π°Π΅Ρ‚ Ρ€Π°Π·Π³Π°Π΄Π°Ρ‚ΡŒ Π°Π½Π°Π³Ρ€Π°ΠΌΠΌΡ‹, ΡΠΎΠΏΠΎΡΡ‚Π°Π²ΠΈΡ‚ΡŒ тСкст – ΠΈΠ·ΠΎΠ±Ρ€Π°ΠΆΠ΅Π½ΠΈΠ΅.


Π”Π°Π²Π°ΠΉΡ‚Π΅ вспомним, Ρ‡Ρ‚ΠΎ являСтся ΠΏΡ€ΠΈΡ‡ΠΈΠ½ΠΎΠΉ сопротивлСния ΠΏΡ€ΠΎΠ²ΠΎΠ΄Π½ΠΈΠΊΠ°?

Зависит Π»ΠΈ сила Ρ‚ΠΎΠΊΠ° ΠΎΡ‚ сопротивлСния ΠΏΡ€ΠΎΠ²ΠΎΠ΄Π½ΠΈΠΊΠ°? Как зависит? Зависит Π»ΠΈ сопротивлСниС ΠΎΡ‚ силы Ρ‚ΠΎΠΊΠ° ΠΈ напряТСния?

Π’Ρ‹ΠΏΠΎΠ»Π½ΡΡŽΡ‚ ΠΈΠ½Ρ‚Π΅Ρ€Π°ΠΊΡ‚ΠΈΠ²Π½Ρ‹Π΅ задания.

Π‘ΠΎΡΡ‚Π°Π²Π»ΡΡŽΡ‚ Π½Π°Π·Π²Π°Π½ΠΈΠ΅ ΠΏΡ€ΠΈΠ±ΠΎΡ€ΠΎΠ², ΠΏΡ€Π°Π²ΠΈΠ»Π° ΠΈΡ… ΠΏΠΎΠ΄ΠΊΠ»ΡŽΡ‡Π΅Π½ΠΈΡ для измСрСния физичСских Π²Π΅Π»ΠΈΡ‡ΠΈΠ½.

Π‘ΠΎΠΏΠΎΡΡ‚Π°Π²Π»ΡΡŽΡ‚ условноС ΠΎΠ±ΠΎΠ·Π½Π°Ρ‡Π΅Π½ΠΈΠ΅ ΠΏΡ€ΠΈΠ±ΠΎΡ€Π° Π½Π° схСмС с Π΅Π³ΠΎ Π½Π°Π·Π²Π°Π½ΠΈΠ΅ΠΌ.


ΠžΡ‚Π²Π΅Ρ‡Π°ΡŽΡ‚ Π½Π° вопросы учитСля

4. Π‘ΠΎΠ·Π΄Π°Π½ΠΈΠ΅ ΠΏΡ€ΠΎΠ±Π»Π΅ΠΌΠ½ΠΎΠΉ ситуации

Π”Π΅ΡΡ‚Π΅Π»ΡŒΠ½ΠΎΡΡ‚ΡŒ учитСля

Π”Π΅ΡΡ‚Π΅Π»ΡŒΠ½ΠΎΡΡ‚ΡŒ ΡƒΡ‡Π΅Π½ΠΈΠΊΠ°

1.ΠŸΡ€Π΅Π΄Π»Π°Π³Π°Π΅Ρ‚ вниманию учащихся ΡΠ»Π΅Π΄ΡƒΡŽΡ‰ΠΈΠΉ экспСримСнт:


К источнику Ρ‚ΠΎΠΊΠ° с напряТСниСм 3 Π’ Π²ΠΊΠ»ΡŽΡ‡Π°ΡŽΡ‚ ΠΏΠΎΠΎΡ‡Π΅Ρ€Ρ‘Π΄Π½ΠΎ Π»Π°ΠΌΠΏΠΎΡ‡ΠΊΠΈ Π½Π° 3Π’ ΠΈ 12 Π’. ΠŸΠΎΡ‡Π΅ΠΌΡƒ Π»Π°ΠΌΠΏΠΎΡ‡ΠΊΠ° Π½Π° 12 Π’ Π½Π΅ свСтится.

2. ΠŸΡ€ΠΎΠ²Π΅Ρ€ΡΠ΅Ρ‚ ΠΈΡΠΏΡ€Π°Π²Π½ΠΎΡΡ‚ΡŒ Π»Π°ΠΌΠΏΡ‹, ΡƒΠ²Π΅Π»ΠΈΡ‡ΠΈΠ² напряТСниС источника Ρ‚ΠΎΠΊΠ°.

Π’Ρ‹Π΄Π²ΠΈΠ³Π°ΡŽΡ‚ прСдполоТСния:

  • Π½Π΅ΠΈΡΠΏΡ€Π°Π²Π½ΠΎΡΡ‚ΡŒ Π»Π°ΠΌΠΏΡ‹.
  • большоС сопротивлСниС Π»Π°ΠΌΠΏΡ‹.

Β 

5. ΠŸΠΎΡΡ‚Π°Π½ΠΎΠ²ΠΊΠ° Ρ†Π΅Π»ΠΈ ΡƒΡ€ΠΎΠΊΠ°. Π˜Π·ΡƒΡ‡Π΅Π½ΠΈΠ΅ Π½ΠΎΠ²ΠΎΠΉ Ρ‚Π΅ΠΌΡ‹

Π”Π΅ΡΡ‚Π΅Π»ΡŒΠ½ΠΎΡΡ‚ΡŒ учитСля

Π”Π΅ΡΡ‚Π΅Π»ΡŒΠ½ΠΎΡΡ‚ΡŒ ΡƒΡ‡Π΅Π½ΠΈΠΊΠ°

1.ΠŸΠΎΠ΄Π²ΠΎΠ΄ΠΈΡ‚ учащихся ΠΊ Ρ†Π΅Π»ΠΈ ΡƒΡ€ΠΎΠΊΠ°. Π Π°Π·Π½Ρ‹Π΅ ΠΏΠΎΡ‚Ρ€Π΅Π±ΠΈΡ‚Π΅Π»ΠΈ Ρ‚ΠΎΠΊΠ° ΠΈΠΌΠ΅ΡŽΡ‚ Ρ€Π°Π·Π½ΠΎΠ΅ сопротивлСниС, Ρ‡Ρ‚ΠΎ Π½Π΅ΠΎΠ±Ρ…ΠΎΠ΄ΠΈΠΌΠΎ Π·Π½Π°Ρ‚ΡŒ, Ρ‡Ρ‚ΠΎΠ±Ρ‹ ΠΈΠ·Π³ΠΎΡ‚ΠΎΠ²ΠΈΡ‚ΡŒ ΠΏΡ€ΠΎΠ²ΠΎΠ΄Π½ΠΈΠΊ с Π½Π΅ΠΎΠ±Ρ…ΠΎΠ΄ΠΈΠΌΡ‹ΠΌ сопротивлСниСм? Π—Π°Π²ΠΈΡΠΈΠΌΠΎΡΡ‚ΡŒ сопротивлСния ΠΎΡ‚ гСомСтричСских Ρ€Π°Π·ΠΌΠ΅Ρ€ΠΎΠ² ΠΏΡ€ΠΎΠ²ΠΎΠ΄Π½ΠΈΠΊΠ° (Π΄Π»ΠΈΠ½Ρ‹ ΠΈ ΠΏΠ»ΠΎΡ‰Π°Π΄ΠΈ ΠΏΠΎΠΏΠ΅Ρ€Π΅Ρ‡Π½ΠΎΠ³ΠΎ сСчСния) ΠΈ вСщСства, ΠΈΠ· ΠΊΠΎΡ‚ΠΎΡ€ΠΎΠ³ΠΎ ΠΎΠ½ ΠΈΠ·Π³ΠΎΡ‚ΠΎΠ²Π»Π΅Π½, Π²ΠΏΠ΅Ρ€Π²Ρ‹Π΅ установил Π“Π΅ΠΎΡ€Π³ Ом.

1.Π€ΠΎΡ€ΠΌΡƒΠ»ΠΈΡ€ΡƒΡŽΡ‚ с ΡƒΡ‡ΠΈΡ‚Π΅Π»Π΅ΠΌ Ρ†Π΅Π»ΡŒ ΡƒΡ€ΠΎΠΊΠ°.

2. ВыяснитС: ΠΎΡ‚ Ρ‡Π΅Π³ΠΎ ΠΈ зависит сопротивлСниС ΠΏΡ€ΠΎΠ²ΠΎΠ΄Π½ΠΈΠΊΠ°? ΠšΡ€Π°Ρ‚ΠΊΠΈΠ΅ Ρ€Π΅ΠΊΠΎΠΌΠ΅Π½Π΄Π°Ρ†ΠΈΠΈ ΠΏΠΎ Ρ€Π°Π±ΠΎΡ‚Π΅ Π½Π°Π΄ ΡƒΡ‡Π΅Π±Π½Ρ‹ΠΌ ΠΌΠΎΠ΄ΡƒΠ»Π΅ΠΌ (РасчСт сопротивлСния ΠΏΡ€ΠΎΠ²ΠΎΠ΄Π½ΠΈΠΊΠ°. УдСльноС сопротивлСниС. РСостаты).

РаздСляСт учащихся ΠΏΠΎ Π³Ρ€ΡƒΠΏΠΏΠ°ΠΌ ΠΈ ΠΏΡ€Π΅Π΄Π»Π°Π³Π°Π΅Ρ‚ ΡΠ°ΠΌΠΎΡΡ‚ΠΎΡΡ‚Π΅Π»ΡŒΠ½ΠΎ Ρ€Π΅ΡˆΠΈΡ‚ΡŒ ΠΏΠΎΡΡ‚Π°Π²Π»Π΅Π½Π½ΡƒΡŽ Π·Π°Π΄Π°Ρ‡Ρƒ, ΡΠ΄Π΅Π»Π°Ρ‚ΡŒ Π²Ρ‹Π²ΠΎΠ΄Ρ‹ ΠΈ ΠΏΠΎΠ΄Π΅Π»ΠΈΡ‚ΡŒΡΡ Π²Ρ‹Π²ΠΎΠ΄Π°ΠΌΠΈ со всСм классом.


Β 

2. Π‘Π»ΡƒΡˆΠ°ΡŽΡ‚ Ρ€Π΅ΠΊΠΎΠΌΠ΅Π½Π΄Π°Ρ†ΠΈΠΈ учитСля для выполнСния практичСского задания. Π Π°Π±ΠΎΡ‚Π°ΡŽΡ‚ с ΡƒΡ‡Π΅Π±Π½Ρ‹ΠΌ ΠΌΠΎΠ΄ΡƒΠ»Π΅ΠΌ (1-5 ΠΏΡƒΠ½ΠΊΡ‚Ρ‹). ΠŸΡ€ΠΎΠ²ΠΎΠ΄ΡΡ‚ исслСдования ΠΈ ΡΠΎΡΡ‚Π°Π²Π»ΡΡŽΡ‚ ΠΊΡ€Π°Ρ‚ΠΊΠΈΠΉ конспСкт.

Π”Π΅Π»Π°ΡŽΡ‚ Π²Ρ‹Π²ΠΎΠ΄Ρ‹ ΠΏΠΎ Ρ€Π΅Π·ΡƒΠ»ΡŒΡ‚Π°Ρ‚Π°ΠΌ выполнСния ΠΊΠ°ΠΆΠ΄ΠΎΠ³ΠΎ задания Π² ΠΎΡ‚Π΄Π΅Π»ΡŒΠ½ΠΎΡΡ‚ΠΈ.

ΠžΠ±ΠΎΠ±Ρ‰Π°ΡŽΡ‚ Ρ€Π΅Π·ΡƒΠ»ΡŒΡ‚Π°Ρ‚Ρ‹ Ρ‚Ρ€Π΅Ρ… Π²ΠΈΡ€Ρ‚ΡƒΠ°Π»ΡŒΠ½Ρ‹Ρ… экспСримСнтов ΠΈ Ρ„ΠΎΡ€ΠΌΡƒΠ»ΠΈΡ€ΡƒΡŽΡ‚ ΠΎΠ±Ρ‰ΠΈΠΉ Π²Ρ‹Π²ΠΎΠ΄.

Β 

3.ΠŸΡ€Π΅Π΄Π»Π°Π³Π°Π΅Ρ‚ ΠΏΡ€ΠΎΠ²Π΅Ρ€ΠΈΡ‚ΡŒ Ρ€Π΅Π·ΡƒΠ»ΡŒΡ‚Π°Ρ‚Ρ‹ исслСдований Π½Π° ΡΠΊΡΠΏΠ΅Ρ€ΠΈΠΌΠ΅Π½Ρ‚Π°Π»ΡŒΠ½ΠΎΠΉ установкС.

Β 

3.БовмСстно с ΡƒΡ‡ΠΈΡ‚Π΅Π»Π΅ΠΌ проводят экспСримСнт ΠΏΠΎΠ΄Ρ‚Π²Π΅Ρ€ΠΆΠ΄Π°ΡŽΡ‰ΠΈΠΉ ΡΠΏΡ€Π°Π²Π΅Π΄Π»ΠΈΠ²ΠΎΡΡ‚ΡŒ сдСланных Π²Ρ‹Π²ΠΎΠ΄ΠΎΠ².


4. ΠŸΡ€Π΅Π΄Π»Π°Π³Π°Π΅Ρ‚ учащимся ΠΏΡ€ΠΎΠ΄ΠΎΠ»ΠΆΠΈΡ‚ΡŒ Ρ€Π°Π±ΠΎΡ‚Ρƒ с ΡƒΡ‡Π΅Π±Π½Ρ‹ΠΌ ΠΌΠΎΠ΄ΡƒΠ»Π΅ΠΌ.

ВыяснитС: какая физичСская Π²Π΅Π»ΠΈΡ‡ΠΈΠ½Π° Ρ…Π°Ρ€Π°ΠΊΡ‚Π΅Ρ€ΠΈΠ·ΡƒΠ΅Ρ‚ Π·Π°Π²ΠΈΡΠΈΠΌΠΎΡΡ‚ΡŒ сопротивлСния ΠΏΡ€ΠΎΠ²ΠΎΠ΄Π½ΠΈΠΊΠ° ΠΎΡ‚ ΠΌΠ°Ρ‚Π΅Ρ€ΠΈΠ°Π»Π°, ΠΈΠ· ΠΊΠΎΡ‚ΠΎΡ€ΠΎΠ³ΠΎ ΠΈΠ·Π³ΠΎΡ‚ΠΎΠ²Π»Π΅Π½ ΠΏΡ€ΠΎΠ²ΠΎΠ΄Π½ΠΈΠΊ.

4. ΠŸΡ€ΠΎΠ΄ΠΎΠ»ΠΆΠ°ΡŽΡ‚ Ρ€Π°Π±ΠΎΡ‚Ρƒ с ΡƒΡ‡Π΅Π±Π½Ρ‹ΠΌ ΠΌΠΎΠ΄ΡƒΠ»Π΅ΠΌ (6 ΠΏΡƒΠ½ΠΊΡ‚) ΠΈ Π²Ρ‹ΠΏΠΎΠ»Π½ΡΡŽΡ‚ записи Π² тСтрадях.

5.ΠŸΡ€Π΅Π΄Π»Π°Π³Π°Π΅Ρ‚ учащимся ΠΏΠΎΡ€Π°Π±ΠΎΡ‚Π°Ρ‚ΡŒ с Ρ‚Π°Π±Π»ΠΈΡ†Π΅ΠΉ ΡƒΠ΄Π΅Π»ΡŒΠ½ΠΎΠ³ΠΎ сопротивлСния Π½Π΅ΠΊΠΎΡ‚ΠΎΡ€Ρ‹Ρ… вСщСств(стр.105 ΡƒΡ‡Π΅Π±Π½ΠΈΠΊ Ρ„ΠΈΠ·ΠΈΠΊΠ° 8 класс, Π°Π²Ρ‚ΠΎΡ€ А.Π’.ΠŸΡ‘Ρ€Ρ‹ΡˆΠΊΠΈΠ½) ΠΈ ΠΎΡ‚Π²Π΅Ρ‚ΠΈΡ‚ΡŒ Π½Π° вопросы:

  • Π’ ΠΊΠ°ΠΊΠΈΡ… Π΅Π΄ΠΈΠ½ΠΈΡ†Π°Ρ… измСрСния прСдставлСны значСния ΡƒΠ΄Π΅Π»ΡŒΠ½ΠΎΠ³ΠΎ сопротивлСния Π² Ρ‚Π°Π±Π»ΠΈΡ†Π΅?
  • КакоС вСщСство ΠΈΠΌΠ΅Π΅Ρ‚ наибольшСС ΡƒΠ΄Π΅Π»ΡŒΠ½ΠΎΠ΅ сопротивлСниС?
  • КакоС вСщСство ΠΈΠΌΠ΅Π΅Ρ‚ наимСньшСС ΡƒΠ΄Π΅Π»ΡŒΠ½ΠΎΠ΅ сопротивлСниС?
  • ΠŸΠΎΡ‡Π΅ΠΌΡƒ ΠΏΡ€ΠΎΠ²ΠΎΠ΄Π° Π»ΠΈΠ½ΠΈΠΉ элСктропСрСдач Π½Π΅ ΠΈΠ·Π³ΠΎΡ‚ΠΎΠ²Π»ΡΡŽΡ‚ ΠΈΠ· Π·ΠΎΠ»ΠΎΡ‚Π° ΠΈ сСрСбра, вСдь Ρƒ Π½ΠΈΡ… ΠΌΠ°Π»ΠΎΠ΅ Π·Π½Π°Ρ‡Π΅Π½ΠΈΠ΅ ΡƒΠ΄Π΅Π»ΡŒΠ½ΠΎΠ³ΠΎ сопротивлСния?
  • КакоС вСщСство ΠΈΡΠΏΠΎΠ»ΡŒΠ·ΡƒΠ΅Ρ‚ΡΡ Π² ΠΏΡ€ΠΎΠ²ΠΎΠ΄Π°Ρ… Π»ΠΈΠ½ΠΈΠΉ элСктропСрСдач? ΠŸΠΎΡ‡Π΅ΠΌΡƒ?
  • Π§Π΅ΠΌΡƒ Ρ€Π°Π²Π½ΠΎ ΡƒΠ΄Π΅Π»ΡŒΠ½ΠΎΠ΅ сопротивлСниС Π½ΠΈΠΊΠ΅Π»ΠΈΠ½Π°? Π§Ρ‚ΠΎ ΠΎΠ·Π½Π°Ρ‡Π°Π΅Ρ‚ эта запись Π² Ρ‚Π°Π±Π»ΠΈΡ†Π΅?

5. Π Π°Π±ΠΎΡ‚Π°ΡŽΡ‚ с Ρ‚Π°Π±Π»ΠΈΡ†Π΅ΠΉ ΡƒΠ΄Π΅Π»ΡŒΠ½ΠΎΠ³ΠΎ сопротивлСния (ΡƒΡ‡Π΅Π±Π½ΠΈΠΊ) ΠΈ ΠΎΡ‚Π²Π΅Ρ‡Π°ΡŽΡ‚ Π½Π° вопросы учитСля.

Β 

6. ΠŸΡ€Π΅Π΄Π»Π°Π³Π°Π΅Ρ‚ учащимся ΠΎΠ±ΠΎΠ±Ρ‰ΠΈΡ‚ΡŒ ΠΏΠΎΠ»ΡƒΡ‡Π΅Π½Π½Ρ‹Π΅ закономСрности ΠΈ ΡΠΎΡΡ‚Π°Π²ΠΈΡ‚ΡŒ Ρ„ΠΎΡ€ΠΌΡƒΠ»Ρƒ для нахоТдСния сопротивлСния ΠΏΡ€ΠΎΠ²ΠΎΠ΄Π½ΠΈΠΊΠ°. ΠŸΡ€ΠΎΠ²Π΅ΡΡ‚ΠΈ ΠΏΡ€ΠΎΠ²Π΅Ρ€ΠΊΡƒ.


6. ΠΠ½Π°Π»ΠΈΠ·ΠΈΡ€ΡƒΡŽΡ‚ ΠΏΠΎΠ»ΡƒΡ‡Π΅Π½Π½Ρ‹Π΅ Ρ€Π΅Π·ΡƒΠ»ΡŒΡ‚Π°Ρ‚Ρ‹ ΠΈ ΡΠΎΡΡ‚Π°Π²Π»ΡΡŽΡ‚ Ρ„ΠΎΡ€ΠΌΡƒΠ»Ρƒ сопротивлСния. Π˜ΡΠΏΠΎΠ»ΡŒΠ·ΡƒΡ, ΡƒΡ‡Π΅Π±Π½Ρ‹ΠΉ ΠΌΠΎΠ΄ΡƒΠ»ΡŒ (7ΠΏΡƒΠ½ΠΊΡ‚) проводят ΠΏΡ€ΠΎΠ²Π΅Ρ€ΠΊΡƒ ΠΏΠΎΠ»ΡƒΡ‡Π΅Π½Π½ΠΎΠΉ Ρ„ΠΎΡ€ΠΌΡƒΠ»Ρ‹.

7. ΠŸΡ€Π΅Π΄Π»Π°Π³Π°Π΅Ρ‚ ΠΏΠΎΠ»ΡƒΡ‡ΠΈΡ‚ΡŒ Π΄ΠΎΠΏΠΎΠ»Π½ΠΈΡ‚Π΅Π»ΡŒΠ½Ρ‹Π΅ Ρ„ΠΎΡ€ΠΌΡƒΠ»Ρ‹.

Β 

7. Π‘Ρ€Π°Π²Π½ΠΈΠ²Π°ΡŽΡ‚ ΠΏΠΎΠ»ΡƒΡ‡Π΅Π½Π½Ρ‹Π΅ Ρ„ΠΎΡ€ΠΌΡƒΠ»Ρ‹.


6. Π­Ρ‚Π°ΠΏ ΠΏΠ΅Ρ€Π²ΠΈΡ‡Π½ΠΎΠ³ΠΎ освоСния Π·Π½Π°Π½ΠΈΠΉ

Π”Π΅ΡΡ‚Π΅Π»ΡŒΠ½ΠΎΡΡ‚ΡŒ учитСля

Π”Π΅ΡΡ‚Π΅Π»ΡŒΠ½ΠΎΡΡ‚ΡŒ ΡƒΡ‡Π΅Π½ΠΈΠΊΠ°

1. Π”Π°Π΅Ρ‚ Π·Π°Π΄Π°Π½ΠΈΠ΅ учащимся ΠΏΠΎ Ρ€Π°Π±ΠΎΡ‚Π΅ с тСстом для ΠΏΠ΅Ρ€Π²ΠΈΡ‡Π½ΠΎΠΉ диагностики уровня освоСния Π·Π½Π°Π½ΠΈΠΉ.

(УдСльноС элСктричСскоС сопротивлСниС вСщСств)

Π’ практичСский ΠΌΠΎΠ΄ΡƒΠ»ΡŒ Π²ΠΊΠ»ΡŽΡ‡Π΅Π½Ρ‹ 7 ΠΈΠ½Ρ‚Π΅Ρ€Π°ΠΊΡ‚ΠΈΠ²Π½Ρ‹Ρ… Π·Π°Π΄Π°Π½ΠΈΠΉ Ρ€Π°Π·Π»ΠΈΡ‡Π½Ρ‹Ρ… Ρ‚ΠΈΠΏΠΎΠ² с Π²ΠΎΠ·ΠΌΠΎΠΆΠ½ΠΎΡΡ‚ΡŒΡŽ Π°Π²Ρ‚ΠΎΠΌΠ°Ρ‚ΠΈΠ·ΠΈΡ€ΠΎΠ²Π°Π½Π½ΠΎΠΉ ΠΏΡ€ΠΎΠ²Π΅Ρ€ΠΊΠΈ для закрСплСния Π·Π½Π°Π½ΠΈΠΉ. ΠœΠΎΠ΄ΡƒΠ»ΡŒ относится ΠΊ II ΡƒΡ€ΠΎΠ²Π½ΡŽ интСрактивности

ΠžΡ‚Π²Π΅Ρ‡Π°Π΅Ρ‚ Π½Π° вопросы учащихся ΠΏΠΎ Ρ€Π°Π·ΡŠΡΡΠ½Π΅Π½ΠΈΡŽ ΠΏΡ€Π°Π²ΠΈΠ» выполнСния Ρ€Π°Π±ΠΎΡ‚Ρ‹.

1.Π‘Π»ΡƒΡˆΠ°ΡŽΡ‚ Ρ€Π΅ΠΊΠΎΠΌΠ΅Π½Π΄Π°Ρ†ΠΈΠΈ учитСля ΠΏΠΎ Π²Ρ‹ΠΏΠΎΠ»Π½Π΅Π½ΠΈΡŽ задания.

Π—Π°Π΄Π°ΡŽΡ‚ ΠΎΡ€Π³Π°Π½ΠΈΠ·Π°Ρ†ΠΈΠΎΠ½Π½Ρ‹Π΅ вопросы ΡƒΡ‡ΠΈΡ‚Π΅Π»ΡŽ.

Π’Ρ‹ΠΏΠΎΠ»Π½ΡΡŽΡ‚ практичСскоС Π·Π°Π΄Π°Π½ΠΈΠ΅.

2.ΠžΠΏΡ€Π΅Π΄Π΅Π»ΡΠ΅Ρ‚ ΡƒΡΠΏΠ΅ΡˆΠ½ΠΎΡΡ‚ΡŒ выполнСния задания. Π˜Π½Ρ‚Π΅Ρ€Π΅ΡΡƒΠ΅Ρ‚ΡΡ возникшими трудностями. ΠžΠΏΡ€Π΅Π΄Π΅Π»ΡΠ΅Ρ‚ задания, с ΠΊΠΎΡ‚ΠΎΡ€Ρ‹ΠΌΠΈ учащиСся Π½Π΅ смогли ΡΠΏΡ€Π°Π²ΠΈΡ‚ΡŒΡΡ.

2.Π‘ΠΎΠΎΠ±Ρ‰Π°ΡŽΡ‚, ΠΊΠ°ΠΊΠΈΠ΅ задания тСста Π²Ρ‹Π·Π²Π°Π»ΠΈ затруднСния.

3.ΠžΡ€Π³Π°Π½ΠΈΠ·ΡƒΠ΅Ρ‚ Ρ€Π°Π±ΠΎΡ‚Ρƒ учащихся ΠΏΠΎ дСмонстрации Π²Π΅Ρ€Π½Ρ‹Ρ… Ρ€Π΅ΡˆΠ΅Π½ΠΈΠΉ.

3.ΠžΠ±ΡŠΡΡΠ½ΡΡŽΡ‚ Ρ€Π΅ΡˆΠ΅Π½ΠΈΠ΅ Π·Π°Π΄Π°Π½ΠΈΠΉ.


7. Π­Ρ‚Π°ΠΏ закрСплСния ΠΏΠΎΠ»ΡƒΡ‡Π΅Π½Π½ΠΎΠ³ΠΎ ΠΌΠ°Ρ‚Π΅Ρ€ΠΈΠ°Π»Π°

Π”Π΅ΡΡ‚Π΅Π»ΡŒΠ½ΠΎΡΡ‚ΡŒ учитСля

Π”Π΅ΡΡ‚Π΅Π»ΡŒΠ½ΠΎΡΡ‚ΡŒ ΡƒΡ‡Π΅Π½ΠΈΠΊΠ°

1. Π£Ρ‡ΠΈΡ‚Π΅Π»ΡŒ ΠΏΡ€ΠΈΠ³Π»Π°ΡˆΠ°Π΅Ρ‚ ΡƒΡ‡Π΅Π½ΠΈΠΊΠ° ΠΊ доскС Π·Π°ΠΏΠΈΡΠ°Ρ‚ΡŒ Ρ€Π΅ΡˆΠ΅Π½ΠΈΠ΅ Π·Π°Π΄Π°Ρ‡ΠΈ ΠΈ ΠΏΡ€ΠΎΠ²Π΅Ρ€ΠΈΡ‚ΡŒ ΠΎΡ‚Π²Π΅Ρ‚.

Π§Π΅ΠΌΡƒ Ρ€Π°Π²Π½ΠΎ сопротивлСниС 100 ΠΌ ΠΌΠ΅Π΄Π½ΠΎΠ³ΠΎ ΠΏΡ€ΠΎΠ²ΠΎΠ΄Π° сСчСниСм 1 ΠΌΠΌ2?


2.ΠŸΡ€Π΅Π΄Π»Π°Π³Π°Π΅Ρ‚ учащимся для закрСплСния Π½ΠΎΠ²ΠΎΠ³ΠΎ ΠΌΠ°Ρ‚Π΅Ρ€ΠΈΠ°Π»Π° Π²Ρ‹ΠΏΠΎΠ»Π½Π΅Π½ΠΈΠ΅ практичСского модуля, для Ρ€Π΅ΡˆΠ΅Π½ΠΈΡ ΠΊΠΎΡ‚ΠΎΡ€ΠΎΠ³ΠΎ Π½Π΅ΠΎΠ±Ρ…ΠΎΠ΄ΠΈΠΌΡ‹ Π½ΠΎΠ²Ρ‹Π΅ знания. Для ΡΠΈΠ»ΡŒΠ½Ρ‹Ρ… учащихся тСстированиС ΠΈΠ· 9 вопросов, для слабых учащихся – Ρ‚Ρ€ΠΈ расчСтных Π·Π°Π΄Π°Ρ‡ΠΈ.

1.УчащиСся Π·Π°ΠΏΠΈΡΡ‹Π²Π°ΡŽΡ‚ условиС ΠΈ Ρ€Π΅ΡˆΠ΅Π½ΠΈΠ΅ Π·Π°Π΄Π°Ρ‡ΠΈ Π² Ρ‚Π΅Ρ‚Ρ€Π°Π΄ΡŒ.

Β 

2.ΠžΠΏΡ€Π΅Π΄Π΅Π»ΡΠ΅Ρ‚ ΡƒΡΠΏΠ΅ΡˆΠ½ΠΎΡΡ‚ΡŒ выполнСния Π·Π°Π΄Π°Π½ΠΈΠΉ. ЀиксируСт Ρ€Π΅Π·ΡƒΠ»ΡŒΡ‚Π°Ρ‚Ρ‹. ΠžΡ‚ΠΌΠ΅Ρ‡Π°Π΅Ρ‚ (для сСбя) учащихся, ΠΊΠΎΡ‚ΠΎΡ€Ρ‹Π΅ Π½Π°ΠΈΠ±ΠΎΠ»Π΅Π΅ (Π½Π°ΠΈΠΌΠ΅Π½Π΅Π΅) ΡƒΡΠΏΠ΅ΡˆΠ½ΠΎ ΡΠΏΡ€Π°Π²ΠΈΠ»ΠΈΡΡŒ с Π·Π°Π΄Π°Π½ΠΈΠ΅ΠΌ.

2.Π’Ρ‹ΠΏΠΎΠ»Π½ΡΡŽΡ‚ тСст.

Π‘ΠΎΠΎΠ±Ρ‰Π°ΡŽΡ‚ Ρ€Π΅Π·ΡƒΠ»ΡŒΡ‚Π°Ρ‚Ρ‹ ΡƒΡ‡ΠΈΡ‚Π΅Π»ΡŽ.


8. РСфлСксия (ПодвСдСниС ΠΈΡ‚ΠΎΠ³ΠΎΠ²)

Π”Π΅ΡΡ‚Π΅Π»ΡŒΠ½ΠΎΡΡ‚ΡŒ учитСля

Π”Π΅ΡΡ‚Π΅Π»ΡŒΠ½ΠΎΡΡ‚ΡŒ ΡƒΡ‡Π΅Π½ΠΈΠΊΠ°

ΠŸΡ€Π΅Π΄Π»Π°Π³Π°Π΅Ρ‚ учащимся Π²Π΅Ρ€Π½ΡƒΡ‚ΡŒΡΡ ΠΊ Ρ†Π΅Π»ΠΈ ΠΈ Π·Π°Π΄Π°Ρ‡Π°ΠΌ ΡƒΡ€ΠΎΠΊΠ°, ΠΏΡ€ΠΎΠ°Π½Π°Π»ΠΈΠ·ΠΈΡ€ΠΎΠ²Π°Ρ‚ΡŒ ΡΡ‚Π΅ΠΏΠ΅Π½ΡŒ ΠΈΡ… достиТСния, ΠΎΠ±ΡŠΡΡΠ½ΠΈΡ‚ΡŒ Ρ€Π΅Π·ΡƒΠ»ΡŒΡ‚Π°Ρ‚Ρ‹ экспСримСнта, поставлСнного Π² Π½Π°Ρ‡Π°Π»Π΅ ΡƒΡ€ΠΎΠΊΠ° с использованиСм Π½ΠΎΠ²ΠΎΠΉ Ρ‚Π΅Ρ€ΠΌΠΈΠ½ΠΎΠ»ΠΎΠ³ΠΈΠΈ, ΡΠ΄Π΅Π»Π°Ρ‚ΡŒ Π²Ρ‹Π²ΠΎΠ΄Ρ‹.

Π‘ΠΎΠΎΠ±Ρ‰Π°Π΅Ρ‚ ΠΎΡ†Π΅Π½ΠΊΠΈ Π·Π° ΡƒΡ€ΠΎΠΊ.

Π’ΡΠΏΠΎΠΌΠΈΠ½Π°ΡŽΡ‚ Ρ†Π΅Π»ΡŒ ΡƒΡ€ΠΎΠΊΠ°. ΠΠ½Π°Π»ΠΈΠ·ΠΈΡ€ΡƒΡŽΡ‚ ΡΡ‚Π΅ΠΏΠ΅Π½ΡŒ Π΅Π΅ достиТСния.

Π€ΠΎΡ€ΠΌΡƒΠ»ΠΈΡ€ΡƒΡŽΡ‚ Π²Ρ‹Π²ΠΎΠ΄Ρ‹.

ΠžΡ†Π΅Π½ΠΈΠ²Π°ΡŽΡ‚ ΡƒΡΠΏΠ΅ΡˆΠ½ΠΎΡΡ‚ΡŒ своСй Ρ€Π°Π±ΠΎΡ‚Ρ‹ Π½Π° ΡƒΡ€ΠΎΠΊΠ΅ ΠΈ ΡƒΡ€ΠΎΠ²Π΅Π½ΡŒ усвоСния Π·Π½Π°Π½ΠΈΠΉ.

9. Π”ΠΎΠΌΠ°ΡˆΠ½Π΅Π΅ Π·Π°Π΄Π°Π½ΠΈΠ΅

Π”Π΅ΡΡ‚Π΅Π»ΡŒΠ½ΠΎΡΡ‚ΡŒ учитСля

Π”Π΅ΡΡ‚Π΅Π»ΡŒΠ½ΠΎΡΡ‚ΡŒ ΡƒΡ‡Π΅Π½ΠΈΠΊΠ°

Π‘ΠΎΠΎΠ±Ρ‰Π°Π΅Ρ‚ домашнСС Π·Π°Π΄Π°Π½ΠΈΠ΅:

1.Β§45, 46; ΡƒΠΏΡ€Π°ΠΆΠ½Π΅Π½ΠΈΠ΅ 20 β„– 2 (Π°), 4.

2.Π˜Π½Ρ‚Π΅Ρ€Π°ΠΊΡ‚ΠΈΠ²Π½Π°Ρ лСкция для Ρ‚Π΅Ρ…, ΠΊΡ‚ΠΎ ΠΏΠ»ΠΎΡ…ΠΎ разобрался с ΠΌΠ°Ρ‚Π΅Ρ€ΠΈΠ°Π»ΠΎΠΌ

3.ВСст

Π—Π°ΠΏΠΈΡΡ‹Π²Π°ΡŽΡ‚ Π² Π΄Π½Π΅Π²Π½ΠΈΠΊΠ°Ρ… домашнСС Π·Π°Π΄Π°Π½ΠΈΠ΅.

УдСльноС сопротивлСниС ΠΈ ΡΠ»Π΅ΠΊΡ‚Ρ€ΠΎΠΏΡ€ΠΎΠ²ΠΎΠ΄ΠΈΠΌΠΎΡΡ‚ΡŒ: Ρ„ΠΎΡ€ΠΌΡƒΠ»Ρ‹ ΠΈ объяснСниС

Π’ Π΄Π°Π½Π½ΠΎΠΉ ΡΡ‚Π°Ρ‚ΡŒΠ΅ ΠΌΡ‹ ΠΏΠΎΠ΄Ρ€ΠΎΠ±Π½ΠΎ Ρ€Π°Π·Π±Π΅Ρ€Π΅ΠΌ Ρ‡Ρ‚ΠΎ Ρ‚Π°ΠΊΠΎΠ΅ ΡƒΠ΄Π΅Π»ΡŒΠ½ΠΎΠ΅ сопротивлСниС ΠΈ ΡΠ»Π΅ΠΊΡ‚Ρ€ΠΎΠΏΡ€ΠΎΠ²ΠΎΠ΄Π½ΠΎΡΡ‚ΡŒ, ясно опишСм всС Ρ„ΠΎΡ€ΠΌΡƒΠ»Ρ‹ с ΠΏΠΎΠΌΠΎΡ‰ΡŒΡŽ ΠΏΡ€ΠΈΠΌΠ΅Ρ€ΠΎΠ² Π·Π°Π΄Π°Ρ‡, Π° Ρ‚Π°ΠΊ ΠΆΠ΅ Π΄Π°Π΄ΠΈΠΌ Π²Π°ΠΌ Ρ‚Π°Π±Π»ΠΈΡ†Ρƒ ΡƒΠ΄Π΅Π»ΡŒΠ½Ρ‹Ρ… сопротивлСний Π½Π΅ΠΊΠΎΡ‚ΠΎΡ€Ρ‹Ρ… ΠΏΡ€ΠΎΠ²ΠΎΠ΄Π½ΠΈΠΊΠΎΠ².

ОписаниС

Π—Π°ΠΊΠΎΠ½ Ома гласит, Ρ‡Ρ‚ΠΎ, ΠΊΠΎΠ³Π΄Π° источник напряТСния (V) подаСтся ΠΌΠ΅ΠΆΠ΄Ρƒ двумя Ρ‚ΠΎΡ‡ΠΊΠ°ΠΌΠΈ Π² Ρ†Π΅ΠΏΠΈ, ΠΌΠ΅ΠΆΠ΄Ρƒ Π½ΠΈΠΌΠΈ Π±ΡƒΠ΄Π΅Ρ‚ ΠΏΡ€ΠΎΡ‚Π΅ΠΊΠ°Ρ‚ΡŒ элСктричСский Ρ‚ΠΎΠΊ (I), Π²Ρ‹Π·Π²Π°Π½Π½Ρ‹ΠΉ Π½Π°Π»ΠΈΡ‡ΠΈΠ΅ΠΌ разности ΠΏΠΎΡ‚Π΅Π½Ρ†ΠΈΠ°Π»ΠΎΠ² ΠΌΠ΅ΠΆΠ΄Ρƒ этими двумя Ρ‚ΠΎΡ‡ΠΊΠ°ΠΌΠΈ.Β ΠšΠΎΠ»ΠΈΡ‡Π΅ΡΡ‚Π²ΠΎ ΠΏΡ€ΠΎΡ‚Π΅ΠΊΠ°ΡŽΡ‰Π΅Π³ΠΎ элСктричСского Ρ‚ΠΎΠΊΠ° ΠΎΠ³Ρ€Π°Π½ΠΈΡ‡Π΅Π½ΠΎ Π²Π΅Π»ΠΈΡ‡ΠΈΠ½ΠΎΠΉ ΠΏΡ€ΠΈΡΡƒΡ‚ΡΡ‚Π²ΡƒΡŽΡ‰Π΅Π³ΠΎ сопротивлСния (R).Β Π”Ρ€ΡƒΠ³ΠΈΠΌΠΈ словами, напряТСниС стимулируСт ΠΏΡ€ΠΎΡ‚Π΅ΠΊΠ°Π½ΠΈΠ΅ Ρ‚ΠΎΠΊΠ° (Π΄Π²ΠΈΠΆΠ΅Π½ΠΈΠ΅ заряда), Π½ΠΎ это сопротивлСниС прСпятствуСт этому.

ΠœΡ‹ всСгда измСряСм элСктричСскоС сопротивлСниС Π² ΠžΠΌΠ°Ρ…, Π³Π΄Π΅ Ом обозначаСтся грСчСской Π±ΡƒΠΊΠ²ΠΎΠΉ ОмСга, Ξ©. Π’Π°ΠΊ, Π½Π°ΠΏΡ€ΠΈΠΌΠ΅Ρ€: 50 Ом, 10 кОм ΠΈΠ»ΠΈ 4,7 МОм ΠΈ Ρ‚.Π΄. ΠŸΡ€ΠΎΠ²ΠΎΠ΄Π½ΠΈΠΊΠΈ (Π½Π°ΠΏΡ€ΠΈΠΌΠ΅Ρ€, ΠΏΡ€ΠΎΠ²ΠΎΠ΄Π° ΠΈ ΠΊΠ°Π±Π΅Π»ΠΈ) ΠΎΠ±Ρ‹Ρ‡Π½ΠΎ ΠΈΠΌΠ΅ΡŽΡ‚ ΠΎΡ‡Π΅Π½ΡŒ Π½ΠΈΠ·ΠΊΠΈΠ΅ значСния сопротивлСния (ΠΌΠ΅Π½Π΅Π΅ 0,1 Ом), ΠΈ, Ρ‚Π°ΠΊΠΈΠΌ ΠΎΠ±Ρ€Π°Π·ΠΎΠΌ, ΠΌΡ‹ ΠΌΠΎΠΆΠ΅ΠΌ ΠΏΡ€Π΅Π½Π΅Π±Ρ€Π΅Ρ‡ΡŒ ΠΈΠΌΠΈ, ΠΊΠ°ΠΊ ΠΌΡ‹ ΠΏΡ€Π΅Π΄ΠΏΠΎΠ»Π°Π³Π°Π΅ΠΌ Π² расчСтах Π°Π½Π°Π»ΠΈΠ·Π° Ρ†Π΅ΠΏΠΈ, Ρ‡Ρ‚ΠΎ ΠΏΡ€ΠΎΠ²ΠΎΠ΄Π° ΠΈΠΌΠ΅ΡŽΡ‚ ноль сопротивлСниС. Π‘ Π΄Ρ€ΡƒΠ³ΠΎΠΉ стороны, изоляторы (Π½Π°ΠΏΡ€ΠΈΠΌΠ΅Ρ€, пластиковыС ΠΈΠ»ΠΈ Π²ΠΎΠ·Π΄ΡƒΡˆΠ½Ρ‹Π΅), ΠΊΠ°ΠΊ ΠΏΡ€Π°Π²ΠΈΠ»ΠΎ, ΠΈΠΌΠ΅ΡŽΡ‚ ΠΎΡ‡Π΅Π½ΡŒ высокиС значСния сопротивлСния (Π±ΠΎΠ»Π΅Π΅ 50 МОм), поэтому ΠΌΡ‹ ΠΌΠΎΠΆΠ΅ΠΌ ΠΈΡ… ΠΈΠ³Π½ΠΎΡ€ΠΈΡ€ΠΎΠ²Π°Ρ‚ΡŒ ΠΈ для Π°Π½Π°Π»ΠΈΠ·Π° Ρ†Π΅ΠΏΠΈ, ΠΏΠΎΡΠΊΠΎΠ»ΡŒΠΊΡƒ ΠΈΡ… Π·Π½Π°Ρ‡Π΅Π½ΠΈΠ΅ слишком Π²Π΅Π»ΠΈΠΊΠΎ.

Но элСктричСскоС сопротивлСниС ΠΌΠ΅ΠΆΠ΄Ρƒ двумя Ρ‚ΠΎΡ‡ΠΊΠ°ΠΌΠΈ ΠΌΠΎΠΆΠ΅Ρ‚ Π·Π°Π²ΠΈΡΠ΅Ρ‚ΡŒ ΠΎΡ‚ ΠΌΠ½ΠΎΠ³ΠΈΡ… Ρ„Π°ΠΊΡ‚ΠΎΡ€ΠΎΠ², Ρ‚Π°ΠΊΠΈΡ… ΠΊΠ°ΠΊ Π΄Π»ΠΈΠ½Π° ΠΏΡ€ΠΎΠ²ΠΎΠ΄Π½ΠΈΠΊΠΎΠ², ΠΏΠ»ΠΎΡ‰Π°Π΄ΡŒ ΠΈΡ… ΠΏΠΎΠΏΠ΅Ρ€Π΅Ρ‡Π½ΠΎΠ³ΠΎ сСчСния, Ρ‚Π΅ΠΌΠΏΠ΅Ρ€Π°Ρ‚ΡƒΡ€Π°, Π° Ρ‚Π°ΠΊΠΆΠ΅ фактичСский ΠΌΠ°Ρ‚Π΅Ρ€ΠΈΠ°Π», ΠΈΠ· ΠΊΠΎΡ‚ΠΎΡ€ΠΎΠ³ΠΎ ΠΎΠ½ ΠΈΠ·Π³ΠΎΡ‚ΠΎΠ²Π»Π΅Π½. НапримСр, Π΄Π°Π²Π°ΠΉΡ‚Π΅ ΠΏΡ€Π΅Π΄ΠΏΠΎΠ»ΠΎΠΆΠΈΠΌ, Ρ‡Ρ‚ΠΎ Ρƒ нас Π΅ΡΡ‚ΡŒ кусок ΠΏΡ€ΠΎΠ²ΠΎΠ΄Π° (ΠΏΡ€ΠΎΠ²ΠΎΠ΄Π½ΠΈΠΊ), ΠΊΠΎΡ‚ΠΎΡ€Ρ‹ΠΉ ΠΈΠΌΠ΅Π΅Ρ‚ Π΄Π»ΠΈΠ½ΡƒΒ L, ΠΏΠ»ΠΎΡ‰Π°Π΄ΡŒ ΠΏΠΎΠΏΠ΅Ρ€Π΅Ρ‡Π½ΠΎΠ³ΠΎ сСчСния AΒ ΠΈ сопротивлСниС R, ΠΊΠ°ΠΊ ΠΏΠΎΠΊΠ°Π·Π°Π½ΠΎ Π½ΠΈΠΆΠ΅.

ЭлСктричСскоС сопротивлСниС R этого простого ΠΏΡ€ΠΎΠ²ΠΎΠ΄Π½ΠΈΠΊΠ° являСтся Ρ„ΡƒΠ½ΠΊΡ†ΠΈΠ΅ΠΉ Π΅Π³ΠΎ Π΄Π»ΠΈΠ½Ρ‹, L ΠΈ ΠΏΠ»ΠΎΡ‰Π°Π΄ΠΈ ΠΏΠΎΠΏΠ΅Ρ€Π΅Ρ‡Π½ΠΎΠ³ΠΎ сСчСния A. Π—Π°ΠΊΠΎΠ½ Ома Π³ΠΎΠ²ΠΎΡ€ΠΈΡ‚ Π½Π°ΠΌ, Ρ‡Ρ‚ΠΎ для Π΄Π°Π½Π½ΠΎΠ³ΠΎ сопротивлСния R Ρ‚ΠΎΠΊ, ΠΏΡ€ΠΎΡ‚Π΅ΠΊΠ°ΡŽΡ‰ΠΈΠΉ Ρ‡Π΅Ρ€Π΅Π· ΠΏΡ€ΠΎΠ²ΠΎΠ΄Π½ΠΈΠΊ, ΠΏΡ€ΠΎΠΏΠΎΡ€Ρ†ΠΈΠΎΠ½Π°Π»Π΅Π½ ΠΏΡ€ΠΈΠ»ΠΎΠΆΠ΅Π½Π½ΠΎΠΌΡƒ Π½Π°ΠΏΡ€ΡΠΆΠ΅Π½ΠΈΡŽ, ΠΏΠΎΡΠΊΠΎΠ»ΡŒΠΊΡƒ I = V / R. Π’Π΅ΠΏΠ΅Ρ€ΡŒ ΠΏΡ€Π΅Π΄ΠΏΠΎΠ»ΠΎΠΆΠΈΠΌ, Ρ‡Ρ‚ΠΎ ΠΌΡ‹ соСдиняСм Π΄Π²Π° ΠΎΠ΄ΠΈΠ½Π°ΠΊΠΎΠ²Ρ‹Ρ… ΠΏΡ€ΠΎΠ²ΠΎΠ΄Π½ΠΈΠΊΠ° вмСстС Π² ΠΏΠΎΡΠ»Π΅Π΄ΠΎΠ²Π°Ρ‚Π΅Π»ΡŒΠ½ΠΎΠΉ ΠΊΠΎΠΌΠ±ΠΈΠ½Π°Ρ†ΠΈΠΈ, ΠΊΠ°ΠΊ ΠΏΠΎΠΊΠ°Π·Π°Π½ΠΎ Π½Π° рисункС.

Π—Π΄Π΅ΡΡŒ, соСдинив Π΄Π²Π° ΠΏΡ€ΠΎΠ²ΠΎΠ΄Π½ΠΈΠΊΠ° вмСстС Π² ΠΏΠΎΡΠ»Π΅Π΄ΠΎΠ²Π°Ρ‚Π΅Π»ΡŒΠ½ΠΎΠΉ ΠΊΠΎΠΌΠ±ΠΈΠ½Π°Ρ†ΠΈΠΈ, Ρ‚ΠΎ Π΅ΡΡ‚ΡŒ, ΠΊ ΠΊΠΎΠ½Ρ†Ρƒ, ΠΌΡ‹ фактичСски ΡƒΠ΄Π²ΠΎΠΈΠ»ΠΈ ΠΎΠ±Ρ‰ΡƒΡŽ Π΄Π»ΠΈΠ½Ρƒ ΠΏΡ€ΠΎΠ²ΠΎΠ΄Π½ΠΈΠΊΠ° (2L), Π² Ρ‚ΠΎ врСмя ΠΊΠ°ΠΊ ΠΏΠ»ΠΎΡ‰Π°Π΄ΡŒ ΠΏΠΎΠΏΠ΅Ρ€Π΅Ρ‡Π½ΠΎΠ³ΠΎ сСчСния A остаСтся Ρ‚ΠΎΡ‡Π½ΠΎ Ρ‚Π°ΠΊΠΎΠΉ ΠΆΠ΅, ΠΊΠ°ΠΊ ΠΈ Ρ€Π°Π½ΡŒΡˆΠ΅. Но ΠΏΠΎΠΌΠΈΠΌΠΎ удвоСния Π΄Π»ΠΈΠ½Ρ‹, ΠΌΡ‹ Ρ‚Π°ΠΊΠΆΠ΅ ΡƒΠ΄Π²ΠΎΠΈΠ»ΠΈ ΠΎΠ±Ρ‰Π΅Π΅ сопротивлСниС ΠΏΡ€ΠΎΠ²ΠΎΠ΄Π½ΠΈΠΊΠ°, Π΄Π°Π² 2R ΠΊΠ°ΠΊ: 1R + 1R = 2R.

Π’Π°ΠΊΠΈΠΌ ΠΎΠ±Ρ€Π°Π·ΠΎΠΌ ,Β ΠΌΡ‹ ΠΌΠΎΠΆΠ΅ΠΌ Π²ΠΈΠ΄Π΅Ρ‚ΡŒ ,Β Ρ‡Ρ‚ΠΎ сопротивлСниС ΠΏΡ€ΠΎΠ²ΠΎΠ΄Π½ΠΈΠΊΠ° ΠΏΡ€ΠΎΠΏΠΎΡ€Ρ†ΠΈΠΎΠ½Π°Π»ΡŒΠ½ΠΎ Π΅Π³ΠΎ Π΄Π»ΠΈΠ½Ρƒ, Ρ‚ΠΎ Π΅ΡΡ‚ΡŒ:Β R ∝ L.Β Π”Ρ€ΡƒΠ³ΠΈΠΌΠΈ словами, ΠΌΡ‹ ΠΎΠΆΠΈΠ΄Π°Π΅ΠΌ, Ρ‡Ρ‚ΠΎ элСктричСскоС сопротивлСниС ΠΏΡ€ΠΎΠ²ΠΎΠ΄Π½ΠΈΠΊΠ° (ΠΈΠ»ΠΈ ΠΏΡ€ΠΎΠ²ΠΎΠ΄Π°) Π±ΡƒΠ΄Π΅Ρ‚ ΠΏΡ€ΠΎΠΏΠΎΡ€Ρ†ΠΈΠΎΠ½Π°Π»ΡŒΠ½ΠΎ большС, Ρ‡Π΅ΠΌ ΠΎΠ½ΠΎ Π΄Π»ΠΈΠ½Π½Π΅Π΅.

ΠžΡ‚ΠΌΠ΅Ρ‚ΠΈΠΌ Ρ‚Π°ΠΊΠΆΠ΅, Ρ‡Ρ‚ΠΎ, удваивая Π΄Π»ΠΈΠ½Ρƒ ΠΈ, ΡΠ»Π΅Π΄ΠΎΠ²Π°Ρ‚Π΅Π»ΡŒΠ½ΠΎ, сопротивлСниС ΠΏΡ€ΠΎΠ²ΠΎΠ΄Π½ΠΈΠΊΠ° (2R), Ρ‡Ρ‚ΠΎΠ±Ρ‹ Π·Π°ΡΡ‚Π°Π²ΠΈΡ‚ΡŒ Ρ‚ΠΎΡ‚ ΠΆΠ΅ Ρ‚ΠΎΠΊ I, Ρ‡Ρ‚ΠΎΠ±Ρ‹ Ρ‚Π΅Ρ‡ΡŒ Ρ‡Π΅Ρ€Π΅Π· ΠΏΡ€ΠΎΠ²ΠΎΠ΄Π½ΠΈΠΊ, ΠΊΠ°ΠΊ ΠΈ Ρ€Π°Π½ΡŒΡˆΠ΅, Π½Π°ΠΌ Π½ΡƒΠΆΠ½ΠΎ ΡƒΠ΄Π²ΠΎΠΈΡ‚ΡŒ (ΡƒΠ²Π΅Π»ΠΈΡ‡ΠΈΡ‚ΡŒ) ΠΏΡ€ΠΈΠ»ΠΎΠΆΠ΅Π½Π½ΠΎΠ΅ напряТСниС I = (2 Π’) / (2R). Π”Π°Π»Π΅Π΅ ΠΏΡ€Π΅Π΄ΠΏΠΎΠ»ΠΎΠΆΠΈΠΌ, Ρ‡Ρ‚ΠΎ ΠΌΡ‹ соСдиняСм Π΄Π²Π° ΠΈΠ΄Π΅Π½Ρ‚ΠΈΡ‡Π½Ρ‹Ρ… ΠΏΡ€ΠΎΠ²ΠΎΠ΄Π½ΠΈΠΊΠ° вмСстС Π² ΠΏΠ°Ρ€Π°Π»Π»Π΅Π»ΡŒΠ½ΠΎΠΉ ΠΊΠΎΠΌΠ±ΠΈΠ½Π°Ρ†ΠΈΠΈ, ΠΊΠ°ΠΊ ΠΏΠΎΠΊΠ°Π·Π°Π½ΠΎ.

Π—Π΄Π΅ΡΡŒ, соСдиняя Π΄Π²Π° ΠΏΡ€ΠΎΠ²ΠΎΠ΄Π½ΠΈΠΊΠ° Π² ΠΏΠ°Ρ€Π°Π»Π»Π΅Π»ΡŒΠ½ΡƒΡŽ ΠΊΠΎΠΌΠ±ΠΈΠ½Π°Ρ†ΠΈΡŽ, ΠΌΡ‹ фактичСски ΡƒΠ΄Π²ΠΎΠΈΠ»ΠΈ ΠΎΠ±Ρ‰ΡƒΡŽ ΠΏΠ»ΠΎΡ‰Π°Π΄ΡŒ, Π΄Π°ΡŽΡ‰ΡƒΡŽ 2А, Π² Ρ‚ΠΎ врСмя ΠΊΠ°ΠΊ Π΄Π»ΠΈΠ½Π° ΠΏΡ€ΠΎΠ²ΠΎΠ΄Π½ΠΈΠΊΠΎΠ² L остаСтся Ρ‚Π°ΠΊΠΎΠΉ ΠΆΠ΅, ΠΊΠ°ΠΊ Ρƒ исходного ΠΎΠ΄ΠΈΠ½ΠΎΡ‡Π½ΠΎΠ³ΠΎ ΠΏΡ€ΠΎΠ²ΠΎΠ΄Π½ΠΈΠΊΠ°. Но ΠΏΠΎΠΌΠΈΠΌΠΎ удвоСния ΠΏΠ»ΠΎΡ‰Π°Π΄ΠΈ, ΠΏΡƒΡ‚Π΅ΠΌ ΠΏΠ°Ρ€Π°Π»Π»Π΅Π»ΡŒΠ½ΠΎΠ³ΠΎ соСдинСния Π΄Π²ΡƒΡ… ΠΏΡ€ΠΎΠ²ΠΎΠ΄Π½ΠΈΠΊΠΎΠ² ΠΌΡ‹ фактичСски Π²Π΄Π²ΠΎΠ΅ сократили ΠΎΠ±Ρ‰Π΅Π΅ сопротивлСниС ΠΏΡ€ΠΎΠ²ΠΎΠ΄Π½ΠΈΠΊΠ°, ΠΏΠΎΠ»ΡƒΡ‡ΠΈΠ² 1 / 2R, ΠΏΠΎΡΠΊΠΎΠ»ΡŒΠΊΡƒ Ρ‚Π΅ΠΏΠ΅Ρ€ΡŒ каТдая ΠΏΠΎΠ»ΠΎΠ²ΠΈΠ½Π° Ρ‚ΠΎΠΊΠ° ΠΏΡ€ΠΎΡ‚Π΅ΠΊΠ°Π΅Ρ‚ Ρ‡Π΅Ρ€Π΅Π· ΠΊΠ°ΠΆΠ΄ΡƒΡŽ Π²Π΅Ρ‚Π²ΡŒ ΠΏΡ€ΠΎΠ²ΠΎΠ΄Π½ΠΈΠΊΠ°.

Π’Π°ΠΊΠΈΠΌ ΠΎΠ±Ρ€Π°Π·ΠΎΠΌ, сопротивлСниС ΠΏΡ€ΠΎΠ²ΠΎΠ΄Π½ΠΈΠΊΠ° ΠΎΠ±Ρ€Π°Ρ‚Π½ΠΎ ΠΏΡ€ΠΎΠΏΠΎΡ€Ρ†ΠΈΠΎΠ½Π°Π»ΡŒΠ½ΠΎ Π΅Π³ΠΎ ΠΏΠ»ΠΎΡ‰Π°Π΄ΠΈ, Ρ‚ΠΎ Π΅ΡΡ‚ΡŒ:Β R 1 / ∝ AΒ ΠΈΠ»ΠΈ R ∝ 1 / A.Β Π”Ρ€ΡƒΠ³ΠΈΠΌΠΈ словами, ΠΌΡ‹ ΠΎΠΆΠΈΠ΄Π°Π΅ΠΌ, Ρ‡Ρ‚ΠΎ элСктричСскоС сопротивлСниС ΠΏΡ€ΠΎΠ²ΠΎΠ΄Π½ΠΈΠΊΠ° (ΠΈΠ»ΠΈ ΠΏΡ€ΠΎΠ²ΠΎΠ΄Π°) Π±ΡƒΠ΄Π΅Ρ‚ ΠΏΡ€ΠΎΠΏΠΎΡ€Ρ†ΠΈΠΎΠ½Π°Π»ΡŒΠ½ΠΎ мСньшС, Ρ‡Π΅ΠΌ большС Π΅Π³ΠΎ ΠΏΠ»ΠΎΡ‰Π°Π΄ΡŒ ΠΏΠΎΠΏΠ΅Ρ€Π΅Ρ‡Π½ΠΎΠ³ΠΎ сСчСния.

ΠšΡ€ΠΎΠΌΠ΅ Ρ‚ΠΎΠ³ΠΎ, удваивая ΠΏΠ»ΠΎΡ‰Π°Π΄ΡŒ ΠΈ, ΡΠ»Π΅Π΄ΠΎΠ²Π°Ρ‚Π΅Π»ΡŒΠ½ΠΎ, Π²Π΄Π²ΠΎΠ΅ увСличивая суммарноС сопротивлСниС Π²Π΅Ρ‚Π²ΠΈ ΠΏΡ€ΠΎΠ²ΠΎΠ΄Π½ΠΈΠΊΠ° (1 / 2R), для Ρ‚ΠΎΠ³ΠΎ ΠΆΠ΅ Ρ‚ΠΎΠΊΠ°,Β Ρ‡Ρ‚ΠΎΠ±Ρ‹Β I ΠΏΡ€ΠΎΡ‚Π΅ΠΊΠ°Π» Ρ‡Π΅Ρ€Π΅Π· ΠΏΠ°Ρ€Π°Π»Π»Π΅Π»ΡŒΠ½ΡƒΡŽ Π²Π΅Ρ‚Π²ΡŒ ΠΏΡ€ΠΎΠ²ΠΎΠ΄Π°, ΠΊΠ°ΠΊ Ρ€Π°Π½ΡŒΡˆΠ΅, Π½Π°ΠΌ Π½ΡƒΠΆΠ½ΠΎ Ρ‚ΠΎΠ»ΡŒΠΊΠΎ Π½Π°ΠΏΠΎΠ»ΠΎΠ²ΠΈΠ½Ρƒ ΡƒΠΌΠ΅Π½ΡŒΡˆΠΈΡ‚ΡŒ ΠΏΡ€ΠΈΠ»ΠΎΠΆΠ΅Π½Π½ΠΎΠ΅ напряТСниС I = (1 / 2V) / (1 / 2R).

НадССмся, ΠΌΡ‹ ΡƒΠ²ΠΈΠ΄ΠΈΠΌ, Ρ‡Ρ‚ΠΎ сопротивлСниС ΠΏΡ€ΠΎΠ²ΠΎΠ΄Π½ΠΈΠΊΠ° прямо ΠΏΡ€ΠΎΠΏΠΎΡ€Ρ†ΠΈΠΎΠ½Π°Π»ΡŒΠ½ΠΎ Π΄Π»ΠΈΠ½Π΅ (L) ΠΏΡ€ΠΎΠ²ΠΎΠ΄Π½ΠΈΠΊΠ°, Ρ‚ΠΎ Π΅ΡΡ‚ΡŒ: R ∝ L, ΠΈ ΠΎΠ±Ρ€Π°Ρ‚Π½ΠΎ ΠΏΡ€ΠΎΠΏΠΎΡ€Ρ†ΠΈΠΎΠ½Π°Π»ΡŒΠ½ΠΎ Π΅Π³ΠΎ ΠΏΠ»ΠΎΡ‰Π°Π΄ΠΈ (A), R ∝ 1 / A.Β Π’Π°ΠΊΠΈΠΌ ΠΎΠ±Ρ€Π°Π·ΠΎΠΌ, ΠΌΡ‹ ΠΌΠΎΠΆΠ΅ΠΌ ΠΏΡ€Π°Π²ΠΈΠ»ΡŒΠ½ΠΎ ΡΠΊΠ°Π·Π°Ρ‚ΡŒ, Ρ‡Ρ‚ΠΎ сопротивлСниС это:

ΠŸΡ€ΠΎΠΏΠΎΡ€Ρ†ΠΈΠΎΠ½Π°Π»ΡŒΠ½ΠΎΡΡ‚ΡŒ сопротивлСния

Но ΠΏΠΎΠΌΠΈΠΌΠΎ Π΄Π»ΠΈΠ½Ρ‹ ΠΈ ΠΏΠ»ΠΎΡ‰Π°Π΄ΠΈ ΠΏΡ€ΠΎΠ²ΠΎΠ΄Π½ΠΈΠΊΠ°, ΠΌΡ‹ Ρ‚Π°ΠΊΠΆΠ΅ ΠΎΠΆΠΈΠ΄Π°Π΅ΠΌ, Ρ‡Ρ‚ΠΎ элСктричСскоС сопротивлСниС ΠΏΡ€ΠΎΠ²ΠΎΠ΄Π½ΠΈΠΊΠ° Π±ΡƒΠ΄Π΅Ρ‚ Π·Π°Π²ΠΈΡΠ΅Ρ‚ΡŒ ΠΎΡ‚ фактичСского ΠΌΠ°Ρ‚Π΅Ρ€ΠΈΠ°Π»Π°, ΠΈΠ· ΠΊΠΎΡ‚ΠΎΡ€ΠΎΠ³ΠΎ ΠΎΠ½ ΠΈΠ·Π³ΠΎΡ‚ΠΎΠ²Π»Π΅Π½, ΠΏΠΎΡ‚ΠΎΠΌΡƒ Ρ‡Ρ‚ΠΎ Ρ€Π°Π·Π½Ρ‹Π΅ проводящиС ΠΌΠ°Ρ‚Π΅Ρ€ΠΈΠ°Π»Ρ‹, мСдь, сСрСбро, алюминий ΠΈ Ρ‚.Π΄., ΠΈΠΌΠ΅ΡŽΡ‚ Ρ€Π°Π·Π½Ρ‹Π΅ физичСскиС ΠΈ элСктричСскиС свойства. Π’Π°ΠΊΠΈΠΌ ΠΎΠ±Ρ€Π°Π·ΠΎΠΌ, ΠΌΡ‹ ΠΌΠΎΠΆΠ΅ΠΌ ΠΏΡ€Π΅ΠΎΠ±Ρ€Π°Π·ΠΎΠ²Π°Ρ‚ΡŒ Π·Π½Π°ΠΊ ΠΏΡ€ΠΎΠΏΠΎΡ€Ρ†ΠΈΠΎΠ½Π°Π»ΡŒΠ½ΠΎΡΡ‚ΠΈ (∝) Π²Ρ‹ΡˆΠ΅ΠΏΡ€ΠΈΠ²Π΅Π΄Π΅Π½Π½ΠΎΠ³ΠΎ уравнСния Π² Π·Π½Π°ΠΊ равСнства, просто Π΄ΠΎΠ±Π°Π²ΠΈΠ² Β«ΠΏΡ€ΠΎΠΏΠΎΡ€Ρ†ΠΈΠΎΠ½Π°Π»ΡŒΠ½ΡƒΡŽ константу» Π² Π²Ρ‹ΡˆΠ΅ΠΏΡ€ΠΈΠ²Π΅Π΄Π΅Π½Π½ΠΎΠ΅ ΡƒΡ€Π°Π²Π½Π΅Π½ΠΈΠ΅, давая:

Π£Ρ€Π°Π²Π½Π΅Π½ΠΈΠ΅ ΡƒΠ΄Π΅Π»ΡŒΠ½ΠΎΠ³ΠΎ элСктричСского сопротивлСния

Π“Π΄Π΅: R β€” сопротивлСниС Π² ΠΎΠΌΠ°Ρ… (Ξ©), L β€” Π΄Π»ΠΈΠ½Π° Π² ΠΌΠ΅Ρ‚Ρ€Π°Ρ… (ΠΌ), A β€” ΠΏΠ»ΠΎΡ‰Π°Π΄ΡŒ Π² ΠΊΠ²Π°Π΄Ρ€Π°Ρ‚Π½Ρ‹Ρ… ΠΌΠ΅Ρ‚Ρ€Π°Ρ… (ΠΌΒ 2 ), ΠΈ Π³Π΄Π΅ извСстна ΠΏΡ€ΠΎΠΏΠΎΡ€Ρ†ΠΈΠΎΠ½Π°Π»ΡŒΠ½Π°Ρ постоянная ρ (грСчСская Π±ΡƒΠΊΠ²Π° Β«rhoΒ») β€” ΡƒΠ΄Π΅Π»ΡŒΠ½ΠΎΠ΅ сопротивлСниС .

УдСльноС элСктричСскоС сопротивлСниС

УдСльноС элСктричСскоС сопротивлСниС ΠΊΠΎΠ½ΠΊΡ€Π΅Ρ‚Π½ΠΎΠ³ΠΎ ΠΌΠ°Ρ‚Π΅Ρ€ΠΈΠ°Π»Π° ΠΏΡ€ΠΎΠ²ΠΎΠ΄Π½ΠΈΠΊΠ° являСтся ΠΌΠ΅Ρ€ΠΎΠΉ Ρ‚ΠΎΠ³ΠΎ, насколько сильно ΠΌΠ°Ρ‚Π΅Ρ€ΠΈΠ°Π» противостоит ΠΏΠΎΡ‚ΠΎΠΊΡƒ элСктричСского Ρ‚ΠΎΠΊΠ° Ρ‡Π΅Ρ€Π΅Π· Π½Π΅Π³ΠΎ.Β Π­Ρ‚ΠΎΡ‚ коэффициСнт ΡƒΠ΄Π΅Π»ΡŒΠ½ΠΎΠ³ΠΎ сопротивлСния, ΠΈΠ½ΠΎΠ³Π΄Π° Π½Π°Π·Ρ‹Π²Π°Π΅ΠΌΡ‹ΠΉ Π΅Π³ΠΎ Β«ΡƒΠ΄Π΅Π»ΡŒΠ½Ρ‹ΠΌ элСктричСским сопротивлСниСм», позволяСт ΡΡ€Π°Π²Π½ΠΈΠ²Π°Ρ‚ΡŒ сопротивлСниС Ρ€Π°Π·Π»ΠΈΡ‡Π½Ρ‹Ρ… Ρ‚ΠΈΠΏΠΎΠ² ΠΏΡ€ΠΎΠ²ΠΎΠ΄Π½ΠΈΠΊΠΎΠ² Π΄Ρ€ΡƒΠ³ с Π΄Ρ€ΡƒΠ³ΠΎΠΌ ΠΏΡ€ΠΈ ΠΎΠΏΡ€Π΅Π΄Π΅Π»Π΅Π½Π½ΠΎΠΉ Ρ‚Π΅ΠΌΠΏΠ΅Ρ€Π°Ρ‚ΡƒΡ€Π΅ Π² соотвСтствии с ΠΈΡ… физичСскими свойствами Π±Π΅Π· ΡƒΡ‡Π΅Ρ‚Π° ΠΈΡ… Π΄Π»ΠΈΠ½Ρ‹ ΠΈΠ»ΠΈ ΠΏΠ»ΠΎΡ‰Π°Π΄ΠΈ ΠΏΠΎΠΏΠ΅Ρ€Π΅Ρ‡Π½ΠΎΠ³ΠΎ сСчСния.Β Π’Π°ΠΊΠΈΠΌ ΠΎΠ±Ρ€Π°Π·ΠΎΠΌ, Ρ‡Π΅ΠΌ Π²Ρ‹ΡˆΠ΅ Π·Π½Π°Ρ‡Π΅Π½ΠΈΠ΅ ΡƒΠ΄Π΅Π»ΡŒΠ½ΠΎΠ³ΠΎ сопротивлСния ρ, Ρ‚Π΅ΠΌ большС сопротивлСниС, ΠΈ Π½Π°ΠΎΠ±ΠΎΡ€ΠΎΡ‚.

НапримСр, ΡƒΠ΄Π΅Π»ΡŒΠ½ΠΎΠ΅ сопротивлСниС Ρ…ΠΎΡ€ΠΎΡˆΠ΅Π³ΠΎ ΠΏΡ€ΠΎΠ²ΠΎΠ΄Π½ΠΈΠΊΠ°, Ρ‚Π°ΠΊΠΎΠ³ΠΎ ΠΊΠ°ΠΊ мСдь, составляСт порядка 1,72 Ρ… 10Β -8 Ом (ΠΈΠ»ΠΈ 17,2 нОм), Ρ‚ΠΎΠ³Π΄Π° ΠΊΠ°ΠΊ ΡƒΠ΄Π΅Π»ΡŒΠ½ΠΎΠ΅ сопротивлСниС ΠΏΠ»ΠΎΡ…ΠΎΠ³ΠΎ ΠΏΡ€ΠΎΠ²ΠΎΠ΄Π½ΠΈΠΊΠ° (изолятора), Ρ‚Π°ΠΊΠΎΠ³ΠΎ ΠΊΠ°ΠΊ Π²ΠΎΠ·Π΄ΡƒΡ…, ΠΌΠΎΠΆΠ΅Ρ‚ Π±Ρ‹Ρ‚ΡŒ Π·Π½Π°Ρ‡ΠΈΡ‚Π΅Π»ΡŒΠ½ΠΎ Π²Ρ‹ΡˆΠ΅ 1,5 Ρ… 10 14Β ΠΈΠ»ΠΈ 150 Ρ‚Ρ€Π»Π½.

Π’Π°ΠΊΠΈΠ΅ ΠΌΠ°Ρ‚Π΅Ρ€ΠΈΠ°Π»Ρ‹, ΠΊΠ°ΠΊ мСдь ΠΈ алюминий, извСстны Π½ΠΈΠ·ΠΊΠΈΠΌ ΡƒΡ€ΠΎΠ²Π½Π΅ΠΌ ΡƒΠ΄Π΅Π»ΡŒΠ½ΠΎΠ³ΠΎ сопротивлСния, благодаря Ρ‡Π΅ΠΌΡƒ элСктричСский Ρ‚ΠΎΠΊ Π»Π΅Π³ΠΊΠΎ ΠΏΡ€ΠΎΡ…ΠΎΠ΄ΠΈΡ‚ Ρ‡Π΅Ρ€Π΅Π· Π½ΠΈΡ…, Ρ‡Ρ‚ΠΎ Π΄Π΅Π»Π°Π΅Ρ‚ эти ΠΌΠ°Ρ‚Π΅Ρ€ΠΈΠ°Π»Ρ‹ ΠΈΠ΄Π΅Π°Π»ΡŒΠ½Ρ‹ΠΌΠΈ для изготовлСния элСктричСских ΠΏΡ€ΠΎΠ²ΠΎΠ΄ΠΎΠ² ΠΈ ΠΊΠ°Π±Π΅Π»Π΅ΠΉ. Π‘Π΅Ρ€Π΅Π±Ρ€ΠΎ ΠΈ Π·ΠΎΠ»ΠΎΡ‚ΠΎ ΠΈΠΌΠ΅ΡŽΡ‚ ΠΎΡ‡Π΅Π½ΡŒ Π½ΠΈΠ·ΠΊΠΈΠ΅ значСния ΡƒΠ΄Π΅Π»ΡŒΠ½ΠΎΠ³ΠΎ сопротивлСния, Π½ΠΎ ΠΏΠΎ понятным ΠΏΡ€ΠΈΡ‡ΠΈΠ½Π°ΠΌ Π΄ΠΎΡ€ΠΎΠΆΠ΅ Π΄Π΅Π»Π°Ρ‚ΡŒ ΠΈΠ· Π½ΠΈΡ… элСктричСскиС ΠΏΡ€ΠΎΠ²ΠΎΠ΄Π°.

Π’ΠΎΠ³Π΄Π° Ρ„Π°ΠΊΡ‚ΠΎΡ€Ρ‹, ΠΊΠΎΡ‚ΠΎΡ€Ρ‹Π΅ Π²Π»ΠΈΡΡŽΡ‚ Π½Π° сопротивлСниС (R) ΠΏΡ€ΠΎΠ²ΠΎΠ΄Π½ΠΈΠΊΠ° Π² ΠΎΠΌΠ°Ρ…, ΠΌΠΎΠ³ΡƒΡ‚ Π±Ρ‹Ρ‚ΡŒ пСрСчислСны ΠΊΠ°ΠΊ:

  • УдСльноС сопротивлСниС (ρ) ΠΌΠ°Ρ‚Π΅Ρ€ΠΈΠ°Π»Π°, ΠΈΠ· ΠΊΠΎΡ‚ΠΎΡ€ΠΎΠ³ΠΎ сдСлан ΠΏΡ€ΠΎΠ²ΠΎΠ΄Π½ΠΈΠΊ.
  • ΠžΠ±Ρ‰Π°Ρ Π΄Π»ΠΈΠ½Π° (L) ΠΏΡ€ΠΎΠ²ΠΎΠ΄Π½ΠΈΠΊΠ°.
  • ΠŸΠ»ΠΎΡ‰Π°Π΄ΡŒ ΠΏΠΎΠΏΠ΅Ρ€Π΅Ρ‡Π½ΠΎΠ³ΠΎ сСчСния (А) ΠΏΡ€ΠΎΠ²ΠΎΠ΄Π½ΠΈΠΊΠ°.
  • Π’Π΅ΠΌΠΏΠ΅Ρ€Π°Ρ‚ΡƒΡ€Π° ΠΏΡ€ΠΎΠ²ΠΎΠ΄Π½ΠΈΠΊΠ°.

ΠŸΡ€ΠΈΠΌΠ΅Ρ€ ΡƒΠ΄Π΅Π»ΡŒΠ½ΠΎΠ³ΠΎ сопротивлСния β„– 1

РассчитайтС ΠΎΠ±Ρ‰Π΅Π΅ сопротивлСниС постоянному Ρ‚ΠΎΠΊΡƒ 100-ΠΌΠ΅Ρ‚Ρ€ΠΎΠ²ΠΎΠ³ΠΎ Ρ€ΡƒΠ»ΠΎΠ½Π°Β ΠΌΠ΅Π΄Π½ΠΎΠ³ΠΎ ΠΏΡ€ΠΎΠ²ΠΎΠ΄Π°Β 2,5 ΠΌΠΌΒ 2, Ссли ΡƒΠ΄Π΅Π»ΡŒΠ½ΠΎΠ΅ сопротивлСниС ΠΌΠ΅Π΄ΠΈ ΠΏΡ€ΠΈ 20 oΒ C составляСт 1,72 x 10Β -8  Ом ΠΌΠ΅Ρ‚Ρ€.

ΠŸΡ€ΠΈΠ²Π΅Π΄Π΅Π½Π½Ρ‹Π΅ Π΄Π°Π½Π½Ρ‹Π΅: ΡƒΠ΄Π΅Π»ΡŒΠ½ΠΎΠ΅ сопротивлСниС ΠΌΠ΅Π΄ΠΈ ΠΏΡ€ΠΈ 20Β oΒ C составляСт 1,72 x 10Β -8Β , Π΄Π»ΠΈΠ½Π° ΠΊΠ°Ρ‚ΡƒΡˆΠΊΠΈ L = 100 ΠΌ, ΠΏΠ»ΠΎΡ‰Π°Π΄ΡŒ ΠΏΠΎΠΏΠ΅Ρ€Π΅Ρ‡Π½ΠΎΠ³ΠΎ сСчСния ΠΏΡ€ΠΎΠ²ΠΎΠ΄Π½ΠΈΠΊΠ° составляСт 2,5 ΠΌΠΌΒ 2, Ρ‡Ρ‚ΠΎ Π΄Π°Π΅Ρ‚ ΠΏΠ»ΠΎΡ‰Π°Π΄ΡŒ: A = 2,5 x 10 -6Β ΠΌΒ 2Β .

ΠžΡ‚Π²Π΅Ρ‚: 688 МОм ΠΈΠ»ΠΈ 0,688 Ом.

УдСльноС элСктричСскоС сопротивлСниС ΠΌΠ°Ρ‚Π΅Ρ€ΠΈΠ°Π»Π°

Π Π°Π½Π΅Π΅ ΠΌΡ‹ Π³ΠΎΠ²ΠΎΡ€ΠΈΠ»ΠΈ, Ρ‡Ρ‚ΠΎ ΡƒΠ΄Π΅Π»ΡŒΠ½ΠΎΠ΅ сопротивлСниС β€” это элСктричСскоС сопротивлСниС Π½Π° Π΅Π΄ΠΈΠ½ΠΈΡ†Ρƒ Π΄Π»ΠΈΠ½Ρ‹ ΠΈ Π½Π° Π΅Π΄ΠΈΠ½ΠΈΡ†Ρƒ ΠΏΠ»ΠΎΡ‰Π°Π΄ΠΈ ΠΏΠΎΠΏΠ΅Ρ€Π΅Ρ‡Π½ΠΎΠ³ΠΎ сСчСния ΠΏΡ€ΠΎΠ²ΠΎΠ΄Π½ΠΈΠΊΠ°, Ρ‚Π°ΠΊΠΈΠΌ ΠΎΠ±Ρ€Π°Π·ΠΎΠΌ, показывая, Ρ‡Ρ‚ΠΎ ΡƒΠ΄Π΅Π»ΡŒΠ½ΠΎΠ΅ сопротивлСниС ρ ΠΈΠΌΠ΅Π΅Ρ‚ Ρ€Π°Π·ΠΌΠ΅Ρ€Ρ‹ Π² Ом-ΠΌΠ΅Ρ‚Ρ€Π°Ρ… ΠΈΠ»ΠΈ Ом Β· ΠΌ, ΠΊΠ°ΠΊ это ΠΎΠ±Ρ‹Ρ‡Π½ΠΎ ΠΏΠΈΡˆΠ΅Ρ‚ΡΡ.Β Π’Π°ΠΊΠΈΠΌ ΠΎΠ±Ρ€Π°Π·ΠΎΠΌ, для ΠΊΠΎΠ½ΠΊΡ€Π΅Ρ‚Π½ΠΎΠ³ΠΎ ΠΌΠ°Ρ‚Π΅Ρ€ΠΈΠ°Π»Π° ΠΏΡ€ΠΈ ΠΎΠΏΡ€Π΅Π΄Π΅Π»Π΅Π½Π½ΠΎΠΉ Ρ‚Π΅ΠΌΠΏΠ΅Ρ€Π°Ρ‚ΡƒΡ€Π΅ Π΅Π³ΠΎ ΡƒΠ΄Π΅Π»ΡŒΠ½ΠΎΠ΅ элСктричСскоС сопротивлСниС опрСдСляСтся ΠΊΠ°ΠΊ.

ЭлСктричСская ΠΏΡ€ΠΎΠ²ΠΎΠ΄ΠΈΠΌΠΎΡΡ‚ΡŒ

Π₯отя ΠΊΠ°ΠΊ элСктричСскоС сопротивлСниС (R), Ρ‚Π°ΠΊ ΠΈ ΡƒΠ΄Π΅Π»ΡŒΠ½ΠΎΠ΅ сопротивлСниС ρ, ΡΠ²Π»ΡΡŽΡ‚ΡΡ Ρ„ΡƒΠ½ΠΊΡ†ΠΈΠ΅ΠΉ физичСской ΠΏΡ€ΠΈΡ€ΠΎΠ΄Ρ‹ ΠΈΡΠΏΠΎΠ»ΡŒΠ·ΡƒΠ΅ΠΌΠΎΠ³ΠΎ ΠΌΠ°Ρ‚Π΅Ρ€ΠΈΠ°Π»Π°, Π° Ρ‚Π°ΠΊΠΆΠ΅ Π΅Π³ΠΎ физичСской Ρ„ΠΎΡ€ΠΌΡ‹ ΠΈ Ρ€Π°Π·ΠΌΠ΅Ρ€Π°, Π²Ρ‹Ρ€Π°ΠΆΠ΅Π½Π½Ρ‹Ρ… Π΅Π³ΠΎ Π΄Π»ΠΈΠ½ΠΎΠΉ (L) ΠΈ ΠΏΠ»ΠΎΡ‰Π°Π΄ΡŒΡŽ Π΅Π³ΠΎ сСчСния ( А), ΠŸΡ€ΠΎΠ²ΠΎΠ΄ΠΈΠΌΠΎΡΡ‚ΡŒΒ ΠΈΠ»ΠΈ ΡƒΠ΄Π΅Π»ΡŒΠ½Π°Ρ ΠΏΡ€ΠΎΠ²ΠΎΠ΄ΠΈΠΌΠΎΡΡ‚ΡŒ относится ΠΊ лСгкости, с ΠΊΠΎΡ‚ΠΎΡ€ΠΎΠΉ элСктричСский Ρ‚ΠΎΠΊ ΠΏΡ€ΠΎΡ…ΠΎΠ΄ΠΈΡ‚ Ρ‡Π΅Ρ€Π΅Π· ΠΌΠ°Ρ‚Π΅Ρ€ΠΈΠ°Π».

ΠŸΡ€ΠΎΠ²ΠΎΠ΄ΠΈΠΌΠΎΡΡ‚ΡŒ (G) являСтся ΠΎΠ±Ρ€Π°Ρ‚Π½ΠΎΠΉ Π²Π΅Π»ΠΈΡ‡ΠΈΠ½ΠΎΠΉ сопротивлСния (1 / R) с Π΅Π΄ΠΈΠ½ΠΈΡ†Π΅ΠΉ проводимости, ΡΠ²Π»ΡΡŽΡ‰Π΅ΠΉΡΡ симСнсом (S), ΠΈ Π΅ΠΉ даСтся ΠΏΠ΅Ρ€Π΅Π²Π΅Ρ€Π½ΡƒΡ‚Ρ‹ΠΉ символ ΠΎΠΌΠΎΠ² mho, ℧.Β Π’Π°ΠΊΠΈΠΌ ΠΎΠ±Ρ€Π°Π·ΠΎΠΌ, ΠΊΠΎΠ³Π΄Π° ΠΏΡ€ΠΎΠ²ΠΎΠ΄Π½ΠΈΠΊ ΠΈΠΌΠ΅Π΅Ρ‚ ΠΏΡ€ΠΎΠ²ΠΎΠ΄ΠΈΠΌΠΎΡΡ‚ΡŒ 1 симСнс (1S), ΠΎΠ½ ΠΈΠΌΠ΅Π΅Ρ‚ сопротивлСниС 1 Ом (1 Ом).Β Π’Π°ΠΊΠΈΠΌ ΠΎΠ±Ρ€Π°Π·ΠΎΠΌ, Ссли Π΅Π³ΠΎ сопротивлСниС удваиваСтся, ΠΏΡ€ΠΎΠ²ΠΎΠ΄ΠΈΠΌΠΎΡΡ‚ΡŒ ΡƒΠΌΠ΅Π½ΡŒΡˆΠ°Π΅Ρ‚ΡΡ Π²Π΄Π²ΠΎΠ΅, ΠΈ Π½Π°ΠΎΠ±ΠΎΡ€ΠΎΡ‚, ΠΊΠ°ΠΊ: БимСнс = 1 / Ом, ΠΈΠ»ΠΈ Ом = 1 / Ом.

Π’ Ρ‚ΠΎ врСмя ΠΊΠ°ΠΊ сопротивлСниС ΠΏΡ€ΠΎΠ²ΠΎΠ΄Π½ΠΈΠΊΠΎΠ² Π΄Π°Π΅Ρ‚ ΡΡ‚Π΅ΠΏΠ΅Π½ΡŒ сопротивлСния ΠΏΠΎΡ‚ΠΎΠΊΡƒ элСктричСского Ρ‚ΠΎΠΊΠ°, ΠΏΡ€ΠΎΠ²ΠΎΠ΄ΠΈΠΌΠΎΡΡ‚ΡŒ ΠΏΡ€ΠΎΠ²ΠΎΠ΄Π½ΠΈΠΊΠ° ΡƒΠΊΠ°Π·Ρ‹Π²Π°Π΅Ρ‚ Π½Π° Π»Π΅Π³ΠΊΠΎΡΡ‚ΡŒ, с ΠΊΠΎΡ‚ΠΎΡ€ΠΎΠΉ ΠΎΠ½ пропускаСт элСктричСский Ρ‚ΠΎΠΊ.Β Π’Π°ΠΊΠΈΠΌ ΠΎΠ±Ρ€Π°Π·ΠΎΠΌ, ΠΌΠ΅Ρ‚Π°Π»Π»Ρ‹, Ρ‚Π°ΠΊΠΈΠ΅ ΠΊΠ°ΠΊ мСдь, алюминий ΠΈΠ»ΠΈ сСрСбро, ΠΈΠΌΠ΅ΡŽΡ‚ ΠΎΡ‡Π΅Π½ΡŒ большиС значСния проводимости, Ρ‡Ρ‚ΠΎ ΠΎΠ·Π½Π°Ρ‡Π°Π΅Ρ‚, Ρ‡Ρ‚ΠΎ ΠΎΠ½ΠΈ ΡΠ²Π»ΡΡŽΡ‚ΡΡ Ρ…ΠΎΡ€ΠΎΡˆΠΈΠΌΠΈ ΠΏΡ€ΠΎΠ²ΠΎΠ΄Π½ΠΈΠΊΠ°ΠΌΠΈ.

ΠŸΡ€ΠΎΠ²ΠΎΠ΄ΠΈΠΌΠΎΡΡ‚ΡŒ, Οƒ (грСчСская Π±ΡƒΠΊΠ²Π° сигма), являСтся ΠΎΠ±Ρ€Π°Ρ‚Π½ΠΎΠΉ Π²Π΅Π»ΠΈΡ‡ΠΈΠ½ΠΎΠΉ ΡƒΠ΄Π΅Π»ΡŒΠ½ΠΎΠ³ΠΎ сопротивлСния.Β Π­Ρ‚ΠΎ 1 / ρ ΠΈ измСряСтся Π² симСнах Π½Π° ΠΌΠ΅Ρ‚Ρ€ (S / m).Β ΠŸΠΎΡΠΊΠΎΠ»ΡŒΠΊΡƒ ΡΠ»Π΅ΠΊΡ‚Ρ€ΠΎΠΏΡ€ΠΎΠ²ΠΎΠ΄Π½ΠΎΡΡ‚ΡŒ Οƒ = 1 / ρ, ΠΏΡ€Π΅Π΄Ρ‹Π΄ΡƒΡ‰Π΅Π΅ Π²Ρ‹Ρ€Π°ΠΆΠ΅Π½ΠΈΠ΅ для элСктричСского сопротивлСния R ΠΌΠΎΠΆΠ½ΠΎ ΠΏΠ΅Ρ€Π΅ΠΏΠΈΡΠ°Ρ‚ΡŒ Π² Π²ΠΈΠ΄Π΅:

ЭлСктричСскоС сопротивлСниС ΠΊΠ°ΠΊ функция проводимости

Π’ΠΎΠ³Π΄Π° ΠΌΡ‹ ΠΌΠΎΠΆΠ΅ΠΌ ΡΠΊΠ°Π·Π°Ρ‚ΡŒ, Ρ‡Ρ‚ΠΎ ΠΏΡ€ΠΎΠ²ΠΎΠ΄ΠΈΠΌΠΎΡΡ‚ΡŒ β€” это ΡΡ„Ρ„Π΅ΠΊΡ‚ΠΈΠ²Π½ΠΎΡΡ‚ΡŒ, посрСдством ΠΊΠΎΡ‚ΠΎΡ€ΠΎΠΉ ΠΏΡ€ΠΎΠ²ΠΎΠ΄Π½ΠΈΠΊ пропускаСт элСктричСский Ρ‚ΠΎΠΊ ΠΈΠ»ΠΈ сигнал Π±Π΅Π· ΠΏΠΎΡ‚Π΅Ρ€ΠΈ сопротивлСния. ΠŸΠΎΡΡ‚ΠΎΠΌΡƒ ΠΌΠ°Ρ‚Π΅Ρ€ΠΈΠ°Π» ΠΈΠ»ΠΈ ΠΏΡ€ΠΎΠ²ΠΎΠ΄Π½ΠΈΠΊ, ΠΊΠΎΡ‚ΠΎΡ€Ρ‹ΠΉ ΠΈΠΌΠ΅Π΅Ρ‚ Π²Ρ‹ΡΠΎΠΊΡƒΡŽ ΠΏΡ€ΠΎΠ²ΠΎΠ΄ΠΈΠΌΠΎΡΡ‚ΡŒ, Π±ΡƒΠ΄Π΅Ρ‚ ΠΈΠΌΠ΅Ρ‚ΡŒ Π½ΠΈΠ·ΠΊΠΎΠ΅ ΡƒΠ΄Π΅Π»ΡŒΠ½ΠΎΠ΅ сопротивлСниС, ΠΈ Π½Π°ΠΎΠ±ΠΎΡ€ΠΎΡ‚, ΠΏΠΎΡΠΊΠΎΠ»ΡŒΠΊΡƒ 1 симСнс (S) Ρ€Π°Π²Π΅Π½ 1 Ом -1Β .Β Π’Π°ΠΊΠΈΠΌ ΠΎΠ±Ρ€Π°Π·ΠΎΠΌ, мСдь, которая являСтся Ρ…ΠΎΡ€ΠΎΡˆΠΈΠΌ ΠΏΡ€ΠΎΠ²ΠΎΠ΄Π½ΠΈΠΊΠΎΠΌ элСктричСского Ρ‚ΠΎΠΊΠ°, ΠΈΠΌΠ΅Π΅Ρ‚ ΠΏΡ€ΠΎΠ²ΠΎΠ΄ΠΈΠΌΠΎΡΡ‚ΡŒ 58,14 x 10Β 6Β Π‘ΠΈΠΌΠ΅Π½ Π½Π° ΠΌΠ΅Ρ‚Ρ€.

ΠŸΡ€ΠΈΠΌΠ΅Ρ€ ΡƒΠ΄Π΅Π»ΡŒΠ½ΠΎΠ³ΠΎ сопротивлСния β„–2

КабСль Π΄Π»ΠΈΠ½ΠΎΠΉ 20 ΠΌΠ΅Ρ‚Ρ€ΠΎΠ² ΠΈΠΌΠ΅Π΅Ρ‚ ΠΏΠ»ΠΎΡ‰Π°Π΄ΡŒ ΠΏΠΎΠΏΠ΅Ρ€Π΅Ρ‡Π½ΠΎΠ³ΠΎ сСчСния 1 ΠΌΠΌΒ 2Β ΠΈ сопротивлСниС 5 Ом.Β Π Π°ΡΡΡ‡ΠΈΡ‚Π°Ρ‚ΡŒ ΠΏΡ€ΠΎΠ²ΠΎΠ΄ΠΈΠΌΠΎΡΡ‚ΡŒ кабСля.

ΠŸΡ€ΠΈΠ²Π΅Π΄Π΅Π½Π½Ρ‹Π΅ Π΄Π°Π½Π½Ρ‹Π΅: сопротивлСниС постоянному Ρ‚ΠΎΠΊΡƒ, R = 5 Ом, Π΄Π»ΠΈΠ½Π° кабСля, L = 20 ΠΌ, Π° ΠΏΠ»ΠΎΡ‰Π°Π΄ΡŒ ΠΏΠΎΠΏΠ΅Ρ€Π΅Ρ‡Π½ΠΎΠ³ΠΎ сСчСния ΠΏΡ€ΠΎΠ²ΠΎΠ΄Π½ΠΈΠΊΠ° составляСт 1 ΠΌΠΌΒ 2, Ρ‡Ρ‚ΠΎΒ Π΄Π°Π΅Ρ‚ ΠΏΠ»ΠΎΡ‰Π°Π΄ΡŒ: A = 1 x 10Β -6Β ΠΌΒ 2Β .

ΠžΡ‚Π²Π΅Ρ‚: 4 ΠΌΠ΅Π³Π°-симСна Π½Π° ΠΌΠ΅Ρ‚Ρ€ Π΄Π»ΠΈΠ½Ρ‹.

Π’Π°Π±Π»ΠΈΡ†Π° ΡƒΠ΄Π΅Π»ΡŒΠ½Ρ‹Ρ… сопротивлСний ΠΏΡ€ΠΎΠ²ΠΎΠ΄Π½ΠΈΠΊΠΎΠ²

ΠŸΡ€ΠΎΠ²ΠΎΠ΄Π½ΠΈΠΊΠ£Π΄Π΅Π»ΡŒΠ½ΠΎΠ΅ сопротивлСниС
ρ

Π’Π΅ΠΌΠΏΠ΅Ρ€Π°Ρ‚ΡƒΡ€Π½Ρ‹ΠΉ коэффициСнт Ξ±
Алюминий0,0284,2
Π‘Ρ€ΠΎΠ½Π·Π°0,095 β€” 0,1β€”
Висмут1,2β€”
Π’ΠΎΠ»ΡŒΡ„Ρ€Π°ΠΌ0,055
Π–Π΅Π»Π΅Π·ΠΎ0,16
Π—ΠΎΠ»ΠΎΡ‚ΠΎ0,0234
Π˜Ρ€ΠΈΠ΄ΠΈΠΉ0,0474β€”
ΠšΠΎΠ½ΡΡ‚Π°Π½Ρ‚Π°Π½0,50,05
Π›Π°Ρ‚ΡƒΠ½ΡŒ0,025 β€” 0,1080,1-0,4
Магний0,0453,9
Манганин0,43 β€” 0,510,01
МСдь0,01754,3
МолибдСн0,059β€”
ΠΠ΅ΠΉΠ·ΠΈΠ»ΡŒΠ±Π΅Ρ€0,20,25
Натрий0,047β€”
НикСлин0,420,1
НикСль0,0876,5
Нихром1,05 β€” 1,40,1
Олово0,124,4
ΠŸΠ»Π°Ρ‚ΠΈΠ½Π°0.1073,9
Π Ρ‚ΡƒΡ‚ΡŒ0,941,0
Π‘Π²ΠΈΠ½Π΅Ρ†0,223,7
Π‘Π΅Ρ€Π΅Π±Ρ€ΠΎ0,0154,1
Π‘Ρ‚Π°Π»ΡŒ0,103 β€” 0,1371-4
Π’ΠΈΡ‚Π°Π½0,6β€”
Π€Π΅Ρ…Ρ€Π°Π»ΡŒ1,15 β€” 1,350,1
Π₯Ρ€ΠΎΠΌΠ°Π»ΡŒ1,3 β€” 1,5β€”
Π¦ΠΈΠ½ΠΊ0,0544,2
Π§ΡƒΠ³ΡƒΠ½0,5-1,01,0

Π“Π΄Π΅: ΡƒΠ΄Π΅Π»ΡŒΠ½ΠΎΠ΅ сопротивлСниС ρ измСряСтся Π² Ом*ΠΌΠΌ2/ΠΌ ΠΈ Ρ‚Π΅ΠΌΠΏΠ΅Ρ€Π°Ρ‚ΡƒΡ€Π½Ρ‹ΠΉ коэффициСнт элСктричСского сопротивлСния ΠΌΠ΅Ρ‚Π°Π»Π»ΠΎΠ² Ξ± измСряСтся Π² 10Β -3*C-1(ΠΈΠ»ΠΈ KΒ -1) .

ΠšΡ€Π°Ρ‚ΠΊΠΎΠ΅ описаниС ΡƒΠ΄Π΅Π»ΡŒΠ½ΠΎΠ³ΠΎ сопротивлСния

ΠœΡ‹ ΠΏΠΎΠ³ΠΎΠ²ΠΎΡ€ΠΈΠ»ΠΈ Π² этой ΡΡ‚Π°Ρ‚ΡŒΠ΅ ΠΎΠ± ΡƒΠ΄Π΅Π»ΡŒΠ½ΠΎΠΌ сопротивлСнии, Ρ‡Ρ‚ΠΎ ΡƒΠ΄Π΅Π»ΡŒΠ½ΠΎΠ΅ сопротивлСниС β€” это свойство ΠΌΠ°Ρ‚Π΅Ρ€ΠΈΠ°Π»Π° ΠΈΠ»ΠΈ ΠΏΡ€ΠΎΠ²ΠΎΠ΄Π½ΠΈΠΊΠ°, ΠΊΠΎΡ‚ΠΎΡ€ΠΎΠ΅ ΡƒΠΊΠ°Π·Ρ‹Π²Π°Π΅Ρ‚, насколько Ρ…ΠΎΡ€ΠΎΡˆΠΎ ΠΌΠ°Ρ‚Π΅Ρ€ΠΈΠ°Π» ΠΏΡ€ΠΎΠ²ΠΎΠ΄ΠΈΡ‚ элСктричСский Ρ‚ΠΎΠΊ. ΠœΡ‹ Ρ‚Π°ΠΊΠΆΠ΅ Π²ΠΈΠ΄Π΅Π»ΠΈ, Ρ‡Ρ‚ΠΎ элСктричСскоС сопротивлСниС (R) ΠΏΡ€ΠΎΠ²ΠΎΠ΄Π½ΠΈΠΊΠ° зависит Π½Π΅ Ρ‚ΠΎΠ»ΡŒΠΊΠΎ ΠΎΡ‚ ΠΌΠ°Ρ‚Π΅Ρ€ΠΈΠ°Π»Π°, ΠΈΠ· ΠΊΠΎΡ‚ΠΎΡ€ΠΎΠ³ΠΎ сдСлан ΠΏΡ€ΠΎΠ²ΠΎΠ΄Π½ΠΈΠΊ, ΠΌΠ΅Π΄ΠΈ, сСрСбра, алюминия ΠΈ Ρ‚.Π΄., Π½ΠΎ Ρ‚Π°ΠΊΠΆΠ΅ ΠΎΡ‚ Π΅Π³ΠΎ физичСских Ρ€Π°Π·ΠΌΠ΅Ρ€ΠΎΠ².

Π‘ΠΎΠΏΡ€ΠΎΡ‚ΠΈΠ²Π»Π΅Π½ΠΈΠ΅ ΠΏΡ€ΠΎΠ²ΠΎΠ΄Π½ΠΈΠΊΠ° прямо ΠΏΡ€ΠΎΠΏΠΎΡ€Ρ†ΠΈΠΎΠ½Π°Π»ΡŒΠ½ΠΎ Π΅Π³ΠΎ Π΄Π»ΠΈΠ½Π΅ (L) ΠΊΠ°ΠΊ R ∝ L. Π’Π°ΠΊΠΈΠΌ ΠΎΠ±Ρ€Π°Π·ΠΎΠΌ, ΡƒΠ΄Π²ΠΎΠ΅Π½ΠΈΠ΅ Π΅Π³ΠΎ Π΄Π»ΠΈΠ½Ρ‹ ΡƒΠ΄Π²ΠΎΠΈΡ‚ Π΅Π³ΠΎ сопротивлСниС, Π² Ρ‚ΠΎ врСмя ΠΊΠ°ΠΊ ΠΏΠΎΡΠ»Π΅Π΄ΠΎΠ²Π°Ρ‚Π΅Π»ΡŒΠ½ΠΎΠ΅ ΡƒΠ΄Π²ΠΎΠ΅Π½ΠΈΠ΅ ΠΏΡ€ΠΎΠ²ΠΎΠ΄Π½ΠΈΠΊΠ° ΡƒΠΌΠ΅Π½ΡŒΡˆΠΈΡ‚ Π²Π΄Π²ΠΎΠ΅ Π΅Π³ΠΎ сопротивлСниС. Π’Π°ΠΊΠΆΠ΅ сопротивлСниС ΠΏΡ€ΠΎΠ²ΠΎΠ΄Π½ΠΈΠΊΠ° ΠΎΠ±Ρ€Π°Ρ‚Π½ΠΎ ΠΏΡ€ΠΎΠΏΠΎΡ€Ρ†ΠΈΠΎΠ½Π°Π»ΡŒΠ½ΠΎ Π΅Π³ΠΎ ΠΏΠ»ΠΎΡ‰Π°Π΄ΠΈ ΠΏΠΎΠΏΠ΅Ρ€Π΅Ρ‡Π½ΠΎΠ³ΠΎ сСчСния (A) ΠΊΠ°ΠΊ R ∝ 1 / A. Π’Π°ΠΊΠΈΠΌ ΠΎΠ±Ρ€Π°Π·ΠΎΠΌ, ΡƒΠ΄Π²ΠΎΠ΅Π½ΠΈΠ΅ Π΅Π³ΠΎ ΠΏΠ»ΠΎΡ‰Π°Π΄ΠΈ ΠΏΠΎΠΏΠ΅Ρ€Π΅Ρ‡Π½ΠΎΠ³ΠΎ сСчСния ΡƒΠΌΠ΅Π½ΡŒΡˆΠΈΠ»ΠΎ Π±Ρ‹ Π΅Π³ΠΎ сопротивлСниС Π²Π΄Π²ΠΎΠ΅, Ρ‚ΠΎΠ³Π΄Π° ΠΊΠ°ΠΊ ΡƒΠ΄Π²ΠΎΠ΅Π½ΠΈΠ΅ Π΅Π³ΠΎ ΠΏΠ»ΠΎΡ‰Π°Π΄ΠΈ ΠΏΠΎΠΏΠ΅Ρ€Π΅Ρ‡Π½ΠΎΠ³ΠΎ сСчСния ΡƒΠ΄Π²ΠΎΠΈΠ»ΠΎ Π±Ρ‹ Π΅Π³ΠΎ сопротивлСниС.

ΠœΡ‹ Ρ‚Π°ΠΊΠΆΠ΅ ΡƒΠ·Π½Π°Π»ΠΈ, Ρ‡Ρ‚ΠΎ ΡƒΠ΄Π΅Π»ΡŒΠ½ΠΎΠ΅ сопротивлСниС (символ: ρ) ΠΏΡ€ΠΎΠ²ΠΎΠ΄Π½ΠΈΠΊΠ° (ΠΈΠ»ΠΈ ΠΌΠ°Ρ‚Π΅Ρ€ΠΈΠ°Π»Π°) связано с физичСским свойством, ΠΈΠ· ΠΊΠΎΡ‚ΠΎΡ€ΠΎΠ³ΠΎ ΠΎΠ½ ΠΈΠ·Π³ΠΎΡ‚ΠΎΠ²Π»Π΅Π½, ΠΈ Π²Π°Ρ€ΡŒΠΈΡ€ΡƒΠ΅Ρ‚ΡΡ ΠΎΡ‚ ΠΌΠ°Ρ‚Π΅Ρ€ΠΈΠ°Π»Π° ΠΊ ΠΌΠ°Ρ‚Π΅Ρ€ΠΈΠ°Π»Ρƒ. НапримСр, ΡƒΠ΄Π΅Π»ΡŒΠ½ΠΎΠ΅ сопротивлСниС ΠΌΠ΅Π΄ΠΈ ΠΎΠ±Ρ‹Ρ‡Π½ΠΎ даСтся ΠΊΠ°ΠΊ: 1,72 Ρ… 10Β -8 Ом Β· ΠΌ. УдСльноС сопротивлСниС ΠΊΠΎΠ½ΠΊΡ€Π΅Ρ‚Π½ΠΎΠ³ΠΎ ΠΌΠ°Ρ‚Π΅Ρ€ΠΈΠ°Π»Π° измСряСтся Π² Π΅Π΄ΠΈΠ½ΠΈΡ†Π°Ρ… Ом-ΠΌΠ΅Ρ‚Ρ€ΠΎΠ² (Ом), ΠΊΠΎΡ‚ΠΎΡ€ΠΎΠ΅ Ρ‚Π°ΠΊΠΆΠ΅ зависит ΠΎΡ‚ Ρ‚Π΅ΠΌΠΏΠ΅Ρ€Π°Ρ‚ΡƒΡ€Ρ‹.

Π’ зависимости ΠΎΡ‚ значСния ΡƒΠ΄Π΅Π»ΡŒΠ½ΠΎΠ³ΠΎ элСктричСского сопротивлСния ΠΊΠΎΠ½ΠΊΡ€Π΅Ρ‚Π½ΠΎΠ³ΠΎ ΠΌΠ°Ρ‚Π΅Ρ€ΠΈΠ°Π»Π° Π΅Π³ΠΎ ΠΌΠΎΠΆΠ½ΠΎ ΠΊΠ»Π°ΡΡΠΈΡ„ΠΈΡ†ΠΈΡ€ΠΎΠ²Π°Ρ‚ΡŒ ΠΊΠ°ΠΊ Β«ΠΏΡ€ΠΎΠ²ΠΎΠ΄Π½ΠΈΠΊΒ», «изолятор» ΠΈΠ»ΠΈ Β«ΠΏΠΎΠ»ΡƒΠΏΡ€ΠΎΠ²ΠΎΠ΄Π½ΠΈΠΊΒ».Β ΠžΠ±Ρ€Π°Ρ‚ΠΈΡ‚Π΅ Π²Π½ΠΈΠΌΠ°Π½ΠΈΠ΅, Ρ‡Ρ‚ΠΎ ΠΏΠΎΠ»ΡƒΠΏΡ€ΠΎΠ²ΠΎΠ΄Π½ΠΈΠΊΠΈ β€” это ΠΌΠ°Ρ‚Π΅Ρ€ΠΈΠ°Π»Ρ‹, Π² ΠΊΠΎΡ‚ΠΎΡ€Ρ‹Ρ… ΠΈΡ… ΠΏΡ€ΠΎΠ²ΠΎΠ΄ΠΈΠΌΠΎΡΡ‚ΡŒ зависит ΠΎΡ‚ примСсСй, добавляСмых Π² ΠΌΠ°Ρ‚Π΅Ρ€ΠΈΠ°Π».

УдСльноС сопротивлСниС Ρ‚Π°ΠΊΠΆΠ΅ Π²Π°ΠΆΠ½ΠΎ Π² систСмах распрСдСлСния элСктроэнСргии, Ρ‚Π°ΠΊ ΠΊΠ°ΠΊ ΡΡ„Ρ„Π΅ΠΊΡ‚ΠΈΠ²Π½ΠΎΡΡ‚ΡŒ систСмы зазСмлСния для систСмы элСктропитания ΠΈ распрСдСлСния сильно зависит ΠΎΡ‚ ΡƒΠ΄Π΅Π»ΡŒΠ½ΠΎΠ³ΠΎ сопротивлСния Π·Π΅ΠΌΠ»ΠΈ ΠΈ ΠΌΠ°Ρ‚Π΅Ρ€ΠΈΠ°Π»Π° ΠΏΠΎΡ‡Π²Ρ‹ Π² мСстС располоТСния систСмы.

ΠŸΡ€ΠΎΠ²ΠΎΠ΄ΠΈΠΌΠΎΡΡ‚ΡŒ β€” это имя, Π΄Π°Π½Π½ΠΎΠ΅ двиТСнию свободных элСктронов Π² Ρ„ΠΎΡ€ΠΌΠ΅ элСктричСского Ρ‚ΠΎΠΊΠ°.Β ΠŸΡ€ΠΎΠ²ΠΎΠ΄ΠΈΠΌΠΎΡΡ‚ΡŒ, Οƒ являСтся ΠΎΠ±Ρ€Π°Ρ‚Π½ΠΎΠΉ Π²Π΅Π»ΠΈΡ‡ΠΈΠ½ΠΎΠΉ ΡƒΠ΄Π΅Π»ΡŒΠ½ΠΎΠ³ΠΎ сопротивлСния.Β Π­Ρ‚ΠΎ 1 / ρ ΠΈ ΠΈΠΌΠ΅Π΅Ρ‚ Π΅Π΄ΠΈΠ½ΠΈΡ†Ρƒ измСрСния симСнс Π½Π° ΠΌΠ΅Ρ‚Ρ€, S / m.Β ΠŸΡ€ΠΎΠ²ΠΎΠ΄ΠΈΠΌΠΎΡΡ‚ΡŒ Π²Π°Ρ€ΡŒΠΈΡ€ΡƒΠ΅Ρ‚ΡΡ ΠΎΡ‚ нуля (для идСального изолятора) Π΄ΠΎ бСсконСчности (для идСального ΠΏΡ€ΠΎΠ²ΠΎΠ΄Π½ΠΈΠΊΠ°).Β Π’Π°ΠΊΠΈΠΌ ΠΎΠ±Ρ€Π°Π·ΠΎΠΌ, свСрхпроводник ΠΈΠΌΠ΅Π΅Ρ‚ Π±Π΅ΡΠΊΠΎΠ½Π΅Ρ‡Π½ΡƒΡŽ ΠΏΡ€ΠΎΠ²ΠΎΠ΄ΠΈΠΌΠΎΡΡ‚ΡŒ ΠΈ практичСски Π½ΡƒΠ»Π΅Π²ΠΎΠ΅ омичСскоС сопротивлСниС.

ЭлСктричСскоС сопротивлСниС ΠΏΡ€ΠΎΠ²ΠΎΠ΄Π½ΠΈΠΊΠ°

ЭлСктричСскоС сопротивлСниС — физичСская Π²Π΅Π»ΠΈΡ‡ΠΈΠ½Π°, которая ΠΏΠΎΠΊΠ°Π·Ρ‹Π²Π°Π΅Ρ‚, ΠΊΠ°ΠΊΠΎΠ΅ прСпятствиС создаСтся Ρ‚ΠΎΠΊΡƒ ΠΏΡ€ΠΈ Π΅Π³ΠΎ ΠΏΡ€ΠΎΡ…ΠΎΠΆΠ΄Π΅Π½ΠΈΠΈ ΠΏΠΎ ΠΏΡ€ΠΎΠ²ΠΎΠ΄Π½ΠΈΠΊΡƒ. Π•Π΄ΠΈΠ½ΠΈΡ†Π°ΠΌΠΈ измСрСния слуТат ΠžΠΌΡ‹, Π² Ρ‡Π΅ΡΡ‚ΡŒ Π“Π΅ΠΎΡ€Π³Π° Ома. Π’ своСм Π·Π°ΠΊΠΎΠ½Π΅ ΠΎΠ½ Π²Ρ‹Π²Π΅Π» Ρ„ΠΎΡ€ΠΌΡƒΠ»Ρƒ для нахоТдСния сопротивлСния, которая ΠΏΡ€ΠΈΠ²Π΅Π΄Π΅Π½Π° Π½ΠΈΠΆΠ΅.Β 

Рассмотрим сопротивлСниС ΠΏΡ€ΠΎΠ²ΠΎΠ΄Π½ΠΈΠΊΠΎΠ² Π½Π° ΠΏΡ€ΠΈΠΌΠ΅Ρ€Π΅ ΠΌΠ΅Ρ‚Π°Π»Π»ΠΎΠ². ΠœΠ΅Ρ‚Π°Π»Π»Ρ‹ ΠΈΠΌΠ΅ΡŽΡ‚ Π²Π½ΡƒΡ‚Ρ€Π΅Π½Π½Π΅Π΅ строСниС Π² Π²ΠΈΠ΄Π΅ кристалличСской Ρ€Π΅ΡˆΠ΅Ρ‚ΠΊΠΈ. Π­Ρ‚Π° Ρ€Π΅ΡˆΠ΅Ρ‚ΠΊΠ° ΠΈΠΌΠ΅Π΅Ρ‚ ΡΡ‚Ρ€ΠΎΠ³ΡƒΡŽ ΡƒΠΏΠΎΡ€ΡΠ΄ΠΎΡ‡Π΅Π½Π½ΠΎΡΡ‚ΡŒ, Π° Π΅Ρ‘ ΡƒΠ·Π»Π°ΠΌΠΈ ΡΠ²Π»ΡΡŽΡ‚ΡΡ ΠΏΠΎΠ»ΠΎΠΆΠΈΡ‚Π΅Π»ΡŒΠ½ΠΎ заряТСнныС ΠΈΠΎΠ½Ρ‹. НоситСлями заряда Π² ΠΌΠ΅Ρ‚Π°Π»Π»Π΅ Π²Ρ‹ΡΡ‚ΡƒΠΏΠ°ΡŽΡ‚ β€œΡΠ²ΠΎΠ±ΠΎΠ΄Π½Ρ‹Π΅β€ элСктроны, ΠΊΠΎΡ‚ΠΎΡ€Ρ‹Π΅ Π½Π΅ ΠΏΡ€ΠΈΠ½Π°Π΄Π»Π΅ΠΆΠ°Ρ‚ ΠΎΠΏΡ€Π΅Π΄Π΅Π»Π΅Π½Π½ΠΎΠΌΡƒ Π°Ρ‚ΠΎΠΌΡƒ, Π° Ρ…Π°ΠΎΡ‚ΠΈΡ‡Π½ΠΎ ΠΏΠ΅Ρ€Π΅ΠΌΠ΅Ρ‰Π°ΡŽΡ‚ΡΡ ΠΌΠ΅ΠΆΠ΄Ρƒ ΡƒΠ·Π»Π°ΠΌΠΈ Ρ€Π΅ΡˆΠ΅Ρ‚ΠΊΠΈ. Из ΠΊΠ²Π°Π½Ρ‚ΠΎΠ²ΠΎΠΉ Ρ„ΠΈΠ·ΠΈΠΊΠΈ извСстно, Ρ‡Ρ‚ΠΎ Π΄Π²ΠΈΠΆΠ΅Π½ΠΈΠ΅ элСктронов Π² ΠΌΠ΅Ρ‚Π°Π»Π»Π΅ это распространСниС элСктромагнитной Π²ΠΎΠ»Π½Ρ‹ Π² Ρ‚Π²Π΅Ρ€Π΄ΠΎΠΌ Ρ‚Π΅Π»Π΅. Π’ΠΎ Π΅ΡΡ‚ΡŒ элСктрон Π² ΠΏΡ€ΠΎΠ²ΠΎΠ΄Π½ΠΈΠΊΠ΅ двиТСтся со ΡΠΊΠΎΡ€ΠΎΡΡ‚ΡŒΡŽ свСта (практичСски), ΠΈ Π΄ΠΎΠΊΠ°Π·Π°Π½ΠΎ, Ρ‡Ρ‚ΠΎ ΠΎΠ½ проявляСт свойства Π½Π΅ Ρ‚ΠΎΠ»ΡŒΠΊΠΎ ΠΊΠ°ΠΊ частица, Π½ΠΎ Π΅Ρ‰Π΅ ΠΈ ΠΊΠ°ΠΊ Π²ΠΎΠ»Π½Π°. А сопротивлСниС ΠΌΠ΅Ρ‚Π°Π»Π»Π° Π²ΠΎΠ·Π½ΠΈΠΊΠ°Π΅Ρ‚ Π² Ρ€Π΅Π·ΡƒΠ»ΡŒΡ‚Π°Ρ‚Π΅ рассСяния элСктромагнитных Π²ΠΎΠ»Π½ (Ρ‚ΠΎ Π΅ΡΡ‚ΡŒ элСктронов) Π½Π° Ρ‚Π΅ΠΏΠ»ΠΎΠ²Ρ‹Ρ… колСбаниях Ρ€Π΅ΡˆΠ΅Ρ‚ΠΊΠΈ ΠΈ Π΅Ρ‘ Π΄Π΅Ρ„Π΅ΠΊΡ‚Π°Ρ….Β  ΠŸΡ€ΠΈ столкновСнии элСктронов с ΡƒΠ·Π»Π°ΠΌΠΈ кристалличСской Ρ€Π΅ΡˆΠ΅Ρ‚ΠΊΠΈ Ρ‡Π°ΡΡ‚ΡŒ энСргии пСрСдаСтся ΡƒΠ·Π»Π°ΠΌ, вслСдствиС Ρ‡Π΅Π³ΠΎ выдСляСтся энСргия. Π­Ρ‚Ρƒ ΡΠ½Π΅Ρ€Π³ΠΈΡŽ ΠΌΠΎΠΆΠ½ΠΎ Π²Ρ‹Ρ‡ΠΈΡΠ»ΠΈΡ‚ΡŒ при постоянном Ρ‚ΠΎΠΊΠ΅, благодаря Π·Π°ΠΊΠΎΠ½Ρƒ ДТоуля-Π›Π΅Π½Ρ†Π° – Q=I2Rt. Как Π²ΠΈΠ΄ΠΈΡ‚Π΅ Ρ‡Π΅ΠΌ большС сопротивлСниС, Ρ‚Π΅ΠΌ большС энСргии выдСляСтся.Β 

УдСльноС сопротивлСниС

БущСствуСт Ρ‚Π°ΠΊΠΎΠ΅ Π²Π°ΠΆΠ½ΠΎΠ΅ понятиС ΠΊΠ°ΠΊΒ ΡƒΠ΄Π΅Π»ΡŒΠ½ΠΎΠ΅ сопротивлСниС, это Ρ‚ΠΎΠΆΠ΅ самоС сопротивлСниС, Ρ‚ΠΎΠ»ΡŒΠΊΠΎ Π² Π΅Π΄ΠΈΠ½ΠΈΡ†Π΅ Π΄Π»ΠΈΠ½Ρ‹. Π£ ΠΊΠ°ΠΆΠ΄ΠΎΠ³ΠΎ ΠΌΠ΅Ρ‚Π°Π»Π»Π° ΠΎΠ½ΠΎ своС, Π½Π°ΠΏΡ€ΠΈΠΌΠ΅Ρ€ Ρƒ ΠΌΠ΅Π΄ΠΈ ΠΎΠ½ΠΎ Ρ€Π°Π²Π½ΠΎ 0,0175 Ом*ΠΌΠΌ2/ΠΌ, Ρƒ алюминия 0,0271 Ом*ΠΌΠΌ2/ΠΌ . Β Π­Ρ‚ΠΎ Π·Π½Π°Ρ‡ΠΈΡ‚, брусок ΠΈΠ· ΠΌΠ΅Π΄ΠΈ Π΄Π»ΠΈΠ½ΠΎΠΉ 1 ΠΌ ΠΈ ΠΏΠ»ΠΎΡ‰Π°Π΄ΡŒΡŽ ΠΏΠΎΠΏΠ΅Ρ€Π΅Ρ‡Π½ΠΎΠ³ΠΎ сСчСния 1 ΠΌΠΌ2Β Π±ΡƒΠ΄Π΅Ρ‚ ΠΈΠΌΠ΅Ρ‚ΡŒ сопротивлСниС 0,0175 Ом, Π° Ρ‚Π°ΠΊΠΎΠΉ ΠΆΠ΅ брусок, Π½ΠΎ ΠΈΠ· алюминия Π±ΡƒΠ΄Π΅Ρ‚ ΠΈΠΌΠ΅Ρ‚ΡŒ сопротивлСниС 0,0271 Ом. Π’Ρ‹Ρ…ΠΎΠ΄ΠΈΡ‚ Ρ‡Ρ‚ΠΎ ΡΠ»Π΅ΠΊΡ‚Ρ€ΠΎΠΏΡ€ΠΎΠ²ΠΎΠ΄Π½ΠΎΡΡ‚ΡŒ ΠΌΠ΅Π΄ΠΈ Π²Ρ‹ΡˆΠ΅ Ρ‡Π΅ΠΌ Ρƒ алюминия. Π£ ΠΊΠ°ΠΆΠ΄ΠΎΠ³ΠΎ ΠΌΠ΅Ρ‚Π°Π»Π»Π° ΡƒΠ΄Π΅Π»ΡŒΠ½ΠΎΠ΅ сопротивлСниС своС, Π° Ρ€Π°ΡΡΡ‡ΠΈΡ‚Π°Ρ‚ΡŒ сопротивлСниС всСго ΠΏΡ€ΠΎΠ²ΠΎΠ΄Π½ΠΈΠΊΠ° ΠΌΠΎΠΆΠ½ΠΎ ΠΏΠΎ Ρ„ΠΎΡ€ΠΌΡƒΠ»Π΅ Β Β 

Π³Π΄Π΅Β p – ΡƒΠ΄Π΅Π»ΡŒΠ½ΠΎΠ΅ сопротивлСниС ΠΌΠ΅Ρ‚Π°Π»Π»Π°, l – Π΄Π»ΠΈΠ½Π° ΠΏΡ€ΠΎΠ²ΠΎΠ΄Π½ΠΈΠΊΠ°, s – ΠΏΠ»ΠΎΡ‰Π°Π΄ΡŒ ΠΏΠΎΠΏΠ΅Ρ€Π΅Ρ‡Π½ΠΎΠ³ΠΎ сСчСния.

ЗначСния ΡƒΠ΄Π΅Π»ΡŒΠ½Ρ‹Ρ… сопротивлСний ΠΏΡ€ΠΈΠ²Π΅Π΄Π΅Π½Ρ‹ Π²Β Ρ‚Π°Π±Π»ΠΈΡ†Π΅ ΡƒΠ΄Π΅Π»ΡŒΠ½Ρ‹Ρ… сопротивлСний ΠΌΠ΅Ρ‚Π°Π»Π»ΠΎΠ²Β (20Β°C)Β Β Β 

ВСщСство

p, Ом*мм2/2

Ξ±,10-3Β 1/K

Алюминий

0.0271

3.8

Π’ΠΎΠ»ΡŒΡ„Ρ€Π°ΠΌ

0.055

4.2

Π–Π΅Π»Π΅Π·ΠΎ

0.098

6

Π—ΠΎΠ»ΠΎΡ‚ΠΎ

0.023

4

Π›Π°Ρ‚ΡƒΠ½ΡŒ

0.025-0.06

1

Манганин

0.42-0.48

0,002-0,05

МСдь

0.0175

4.1

НикСль

0.1

2.7

ΠšΠΎΠ½ΡΡ‚Π°Π½Ρ‚Π°Π½

0.44-0.52

0.02

Нихром

1.1

0.15

Π‘Π΅Ρ€Π΅Π±Ρ€ΠΎ

0.016

4

Π¦ΠΈΠ½ΠΊ

0.059

2.7

ΠšΡ€ΠΎΠΌΠ΅ ΡƒΠ΄Π΅Π»ΡŒΠ½ΠΎΠ³ΠΎ сопротивлСния Π² Ρ‚Π°Π±Π»ΠΈΡ†Π΅ Π΅ΡΡ‚ΡŒ значСния ВКБ, ΠΎΠ± этом коэффициСнтС Ρ‡ΡƒΡ‚ΡŒ ΠΏΠΎΠ·ΠΆΠ΅.

Π—Π°Π²ΠΈΡΠΈΠΌΠΎΡΡ‚ΡŒ ΡƒΠ΄Π΅Π»ΡŒΠ½ΠΎΠ³ΠΎ сопротивлСния ΠΎΡ‚ Π΄Π΅Ρ„ΠΎΡ€ΠΌΠ°Ρ†ΠΈΠΉ


ΠŸΡ€ΠΈ Ρ…ΠΎΠ»ΠΎΠ΄Π½ΠΎΠΉ ΠΎΠ±Ρ€Π°Π±ΠΎΡ‚ΠΊΠ΅ ΠΌΠ΅Ρ‚Π°Π»Π»ΠΎΠ² Π΄Π°Π²Π»Π΅Π½ΠΈΠ΅ΠΌ, ΠΌΠ΅Ρ‚Π°Π»Π» испытываСт ΠΏΠ»Π°ΡΡ‚ΠΈΡ‡Π΅ΡΠΊΡƒΡŽ Π΄Π΅Ρ„ΠΎΡ€ΠΌΠ°Ρ†ΠΈΡŽ. ΠŸΡ€ΠΈ пластичСской Π΄Π΅Ρ„ΠΎΡ€ΠΌΠ°Ρ†ΠΈΠΈ кристалличСская Ρ€Π΅ΡˆΠ΅Ρ‚ΠΊΠ° искаТаСтся, количСство Π΄Π΅Ρ„Π΅ΠΊΡ‚ΠΎΠ² становится большС. Π‘ ΡƒΠ²Π΅Π»ΠΈΡ‡Π΅Π½ΠΈΠ΅ΠΌ Π΄Π΅Ρ„Π΅ΠΊΡ‚ΠΎΠ² кристалличСской Ρ€Π΅ΡˆΠ΅Ρ‚ΠΊΠΈ, сопротивлСниС Ρ‚Π΅Ρ‡Π΅Π½ΠΈΡŽ элСктронов ΠΏΠΎ ΠΏΡ€ΠΎΠ²ΠΎΠ΄Π½ΠΈΠΊΡƒ растСт, ΡΠ»Π΅Π΄ΠΎΠ²Π°Ρ‚Π΅Π»ΡŒΠ½ΠΎ, ΡƒΠ΄Π΅Π»ΡŒΠ½ΠΎΠ΅ сопротивлСниС ΠΌΠ΅Ρ‚Π°Π»Π»Π° увСличиваСтся. К ΠΏΡ€ΠΈΠΌΠ΅Ρ€Ρƒ, ΠΏΡ€ΠΎΠ²ΠΎΠ»ΠΎΠΊΡƒ ΠΈΠ·Π³ΠΎΡ‚Π°Π²Π»ΠΈΠ²Π°ΡŽΡ‚ ΠΌΠ΅Ρ‚ΠΎΠ΄ΠΎΠΌ протяТки, это Π·Π½Π°Ρ‡ΠΈΡ‚, Ρ‡Ρ‚ΠΎ ΠΌΠ΅Ρ‚Π°Π»Π» испытываСт ΠΏΠ»Π°ΡΡ‚ΠΈΡ‡Π΅ΡΠΊΡƒΡŽ Π΄Π΅Ρ„ΠΎΡ€ΠΌΠ°Ρ†ΠΈΡŽ, Π² Ρ€Π΅Π·ΡƒΠ»ΡŒΡ‚Π°Ρ‚Π΅ Ρ‡Π΅Π³ΠΎ, ΡƒΠ΄Π΅Π»ΡŒΠ½ΠΎΠ΅ сопротивлСниС растСт. На ΠΏΡ€Π°ΠΊΡ‚ΠΈΠΊΠ΅ для ΡƒΠΌΠ΅Π½ΡŒΡˆΠ΅Π½ΠΈΡ сопротивлСния ΠΏΡ€ΠΈΠΌΠ΅Π½ΡΡŽΡ‚ рСкристаллизационный ΠΎΡ‚ΠΆΠΈΠ³, это слоТный тСхнологичСский процСсс, послС ΠΊΠΎΡ‚ΠΎΡ€ΠΎΠ³ΠΎ кристалличСская Ρ€Π΅ΡˆΠ΅Ρ‚ΠΊΠ° ΠΊΠ°ΠΊ Π±Ρ‹, β€œΡ€Π°ΡΠΏΡ€Π°Π²Π»ΡΠ΅Ρ‚ΡΡβ€ ΠΈ количСство Π΄Π΅Ρ„Π΅ΠΊΡ‚ΠΎΠ² ΡƒΠΌΠ΅Π½ΡŒΡˆΠ°Π΅Ρ‚ΡΡ, ΡΠ»Π΅Π΄ΠΎΠ²Π°Ρ‚Π΅Π»ΡŒΠ½ΠΎ, ΠΈ сопротивлСниС ΠΌΠ΅Ρ‚Π°Π»Π»Π° Ρ‚ΠΎΠΆΠ΅.

ΠŸΡ€ΠΈ растяТСнии ΠΈΠ»ΠΈ сТатии, ΠΌΠ΅Ρ‚Π°Π»Π» испытываСт ΡƒΠΏΡ€ΡƒΠ³ΡƒΡŽ Π΄Π΅Ρ„ΠΎΡ€ΠΌΠ°Ρ†ΠΈΡŽ. ΠŸΡ€ΠΈ ΡƒΠΏΡ€ΡƒΠ³ΠΎΠΉ Π΄Π΅Ρ„ΠΎΡ€ΠΌΠ°Ρ†ΠΈΠΈ Π²Ρ‹Π·Π²Π°Π½Π½ΠΎΠΉ растяТСниСм, Π°ΠΌΠΏΠ»ΠΈΡ‚ΡƒΠ΄Ρ‹ Ρ‚Π΅ΠΏΠ»ΠΎΠ²Ρ‹Ρ… ΠΊΠΎΠ»Π΅Π±Π°Π½ΠΈΠΉ ΡƒΠ·Π»ΠΎΠ² кристалличСской Ρ€Π΅ΡˆΠ΅Ρ‚ΠΊΠΈ ΡƒΠ²Π΅Π»ΠΈΡ‡ΠΈΠ²Π°ΡŽΡ‚ΡΡ, ΡΠ»Π΅Π΄ΠΎΠ²Π°Ρ‚Π΅Π»ΡŒΠ½ΠΎ, элСктроны ΠΈΡΠΏΡ‹Ρ‚Ρ‹Π²Π°ΡŽΡ‚ большиС затруднСния, ΠΈ Π² связи с этим, увСличиваСтся ΡƒΠ΄Π΅Π»ΡŒΠ½ΠΎΠ΅ сопротивлСниС. ΠŸΡ€ΠΈ ΡƒΠΏΡ€ΡƒΠ³ΠΎΠΉ Π΄Π΅Ρ„ΠΎΡ€ΠΌΠ°Ρ†ΠΈΠΈ Π²Ρ‹Π·Π²Π°Π½Π½ΠΎΠΉ сТатиСм, Π°ΠΌΠΏΠ»ΠΈΡ‚ΡƒΠ΄Ρ‹ Ρ‚Π΅ΠΏΠ»ΠΎΠ²Ρ‹Ρ… ΠΊΠΎΠ»Π΅Π±Π°Π½ΠΈΠΉ ΡƒΠ·Π»ΠΎΠ² ΡƒΠΌΠ΅Π½ΡŒΡˆΠ°ΡŽΡ‚ΡΡ, ΡΠ»Π΅Π΄ΠΎΠ²Π°Ρ‚Π΅Π»ΡŒΠ½ΠΎ, элСктронам ΠΏΡ€ΠΎΡ‰Π΅ Π΄Π²ΠΈΠ³Π°Ρ‚ΡŒΡΡ, ΠΈ ΡƒΠ΄Π΅Π»ΡŒΠ½ΠΎΠ΅ сопротивлСниС ΡƒΠΌΠ΅Π½ΡŒΡˆΠ°Π΅Ρ‚ΡΡ.

ВлияниС Ρ‚Π΅ΠΌΠΏΠ΅Ρ€Π°Ρ‚ΡƒΡ€Ρ‹ Π½Π° ΡƒΠ΄Π΅Π»ΡŒΠ½ΠΎΠ΅ сопротивлСниС

Как ΠΌΡ‹ ΡƒΠΆΠ΅ выяснили Π²Ρ‹ΡˆΠ΅, ΠΏΡ€ΠΈΡ‡ΠΈΠ½ΠΎΠΉ сопротивлСния в ΠΌΠ΅Ρ‚Π°Π»Π»Π΅Β ΡΠ²Π»ΡΡŽΡ‚ΡΡ ΡƒΠ·Π»Ρ‹ кристалличСской Ρ€Π΅ΡˆΠ΅Ρ‚ΠΊΠΈ ΠΈ ΠΈΡ… колСбания. Π’Π°ΠΊ Π²ΠΎΡ‚, ΠΏΡ€ΠΈ ΡƒΠ²Π΅Π»ΠΈΡ‡Π΅Π½ΠΈΠΈ Ρ‚Π΅ΠΌΠΏΠ΅Ρ€Π°Ρ‚ΡƒΡ€Ρ‹, Ρ‚Π΅ΠΏΠ»ΠΎΠ²Ρ‹Π΅ колСбания ΡƒΠ·Π»ΠΎΠ² ΡƒΠ²Π΅Π»ΠΈΡ‡ΠΈΠ²Π°ΡŽΡ‚ΡΡ, Π° Π·Π½Π°Ρ‡ΠΈΡ‚, ΡƒΠ΄Π΅Π»ΡŒΠ½ΠΎΠ΅ сопротивлСниС Ρ‚Π°ΠΊΠΆΠ΅ увСличиваСтся. БущСствуСт такая Π²Π΅Π»ΠΈΡ‡ΠΈΠ½Π° ΠΊΠ°ΠΊΒ Ρ‚Π΅ΠΌΠΏΠ΅Ρ€Π°Ρ‚ΡƒΡ€Π½Ρ‹ΠΉ коэффициСнт сопротивлСния (ВКБ), ΠΊΠΎΡ‚ΠΎΡ€Ρ‹ΠΉ ΠΏΠΎΠΊΠ°Π·Ρ‹Π²Π°Π΅Ρ‚ насколько увСличиваСтся, ΠΈΠ»ΠΈ ΡƒΠΌΠ΅Π½ΡŒΡˆΠ°Π΅Ρ‚ΡΡ ΡƒΠ΄Π΅Π»ΡŒΠ½ΠΎΠ΅ сопротивлСниС ΠΌΠ΅Ρ‚Π°Π»Π»Π° ΠΏΡ€ΠΈ Π½Π°Π³Ρ€Π΅Π²Π΅ ΠΈΠ»ΠΈ ΠΎΡ…Π»Π°ΠΆΠ΄Π΅Π½ΠΈΠΈ. НапримСр, Ρ‚Π΅ΠΌΠΏΠ΅Ρ€Π°Ρ‚ΡƒΡ€Π½Ρ‹ΠΉ коэффициСнт ΠΌΠ΅Π΄ΠΈ ΠΏΡ€ΠΈ 20 градусах ΠΏΠΎ Ρ†Π΅Π»ΡŒΡΠΈΡŽ Ρ€Π°Π²Π΅Π½Β 4.1Β Β·Β 10Β βˆ’Β 3Β 1/градус. Π­Ρ‚ΠΎ ΠΎΠ·Π½Π°Ρ‡Π°Π΅Ρ‚ Ρ‡Ρ‚ΠΎ ΠΏΡ€ΠΈ Π½Π°Π³Ρ€Π΅Π²Π΅, ΠΊ ΠΏΡ€ΠΈΠΌΠ΅Ρ€Ρƒ, ΠΌΠ΅Π΄Π½ΠΎΠΉ ΠΏΡ€ΠΎΠ²ΠΎΠ»ΠΎΠΊΠΈ Π½Π° 1 градус Ρ†Π΅Π»ΡŒΡΠΈΡ, Π΅Ρ‘ ΡƒΠ΄Π΅Π»ΡŒΠ½ΠΎΠ΅ сопротивлСниС увСличится Π½Π°Β 4.1Β Β·Β 10Β βˆ’Β 3Β  Ом. УдСльноС сопротивлСниС ΠΏΡ€ΠΈ ΠΈΠ·ΠΌΠ΅Π½Π΅Π½ΠΈΠΈ Ρ‚Π΅ΠΌΠΏΠ΅Ρ€Π°Ρ‚ΡƒΡ€Ρ‹ ΠΌΠΎΠΆΠ½ΠΎ Π²Ρ‹Ρ‡ΠΈΡΠ»ΠΈΡ‚ΡŒ ΠΏΠΎ Ρ„ΠΎΡ€ΠΌΡƒΠ»Π΅Β 

Π³Π΄Π΅ r это ΡƒΠ΄Π΅Π»ΡŒΠ½ΠΎΠ΅ сопротивлСниС послС Π½Π°Π³Ρ€Π΅Π²Π°, r0 – ΡƒΠ΄Π΅Π»ΡŒΠ½ΠΎΠ΅ сопротивлСниС Π΄ΠΎ Π½Π°Π³Ρ€Π΅Π²Π°, a – Ρ‚Π΅ΠΌΠΏΠ΅Ρ€Π°Ρ‚ΡƒΡ€Π½Ρ‹ΠΉ коэффициСнт сопротивлСния, t2 – Ρ‚Π΅ΠΌΠΏΠ΅Ρ€Π°Ρ‚ΡƒΡ€Π° Π΄ΠΎ Π½Π°Π³Ρ€Π΅Π²Π°, t1Β Β — Ρ‚Π΅ΠΌΠΏΠ΅Ρ€Π°Ρ‚ΡƒΡ€Π° послС Π½Π°Π³Ρ€Π΅Π²Π°.Β 

ΠŸΠΎΠ΄ΡΡ‚Π°Π²ΠΈΠ² наши значСния, ΠΌΡ‹ ΠΏΠΎΠ»ΡƒΡ‡ΠΈΠΌ: r=0,0175*(1+0.0041*(154-20))=0,0271 Ом*ΠΌΠΌ2/ΠΌ. Как Π²ΠΈΠ΄ΠΈΡ‚Π΅ наш брусок ΠΈΠ· ΠΌΠ΅Π΄ΠΈ Π΄Π»ΠΈΠ½ΠΎΠΉ 1 ΠΌ ΠΈ ΠΏΠ»ΠΎΡ‰Π°Π΄ΡŒΡŽ ΠΏΠΎΠΏΠ΅Ρ€Π΅Ρ‡Π½ΠΎΠ³ΠΎ сСчСния 1 ΠΌΠΌ2, послС Π½Π°Π³Ρ€Π΅Π²Π° Π΄ΠΎ 154 градусов, ΠΈΠΌΠ΅Π» Π±Ρ‹ сопротивлСниС, ΠΊΠ°ΠΊ Ρƒ Ρ‚Π°ΠΊΠΎΠ³ΠΎ ΠΆΠ΅ бруска, Ρ‚ΠΎΠ»ΡŒΠΊΠΎ ΠΈΠ· алюминия ΠΈ ΠΏΡ€ΠΈ Ρ‚Π΅ΠΌΠΏΠ΅Ρ€Π°Ρ‚ΡƒΡ€Π΅ Ρ€Π°Π²Π½ΠΎΠΉ 20 градусов Ρ†Π΅Π»ΡŒΡΠΈΡ.Β 

Бвойство измСнСния сопротивлСния ΠΏΡ€ΠΈ ΠΈΠ·ΠΌΠ΅Π½Π΅Π½ΠΈΠΈ Ρ‚Π΅ΠΌΠΏΠ΅Ρ€Π°Ρ‚ΡƒΡ€Ρ‹, ΠΈΡΠΏΠΎΠ»ΡŒΠ·ΡƒΠ΅Ρ‚ΡΡ Π² Ρ‚Π΅Ρ€ΠΌΠΎΠΌΠ΅Ρ‚Ρ€Π°Ρ… сопротивлСния. Π­Ρ‚ΠΈ ΠΏΡ€ΠΈΠ±ΠΎΡ€Ρ‹ ΠΌΠΎΠ³ΡƒΡ‚ ΠΈΠ·ΠΌΠ΅Ρ€ΡΡ‚ΡŒ Ρ‚Π΅ΠΌΠΏΠ΅Ρ€Π°Ρ‚ΡƒΡ€Ρƒ ΠΎΡΠ½ΠΎΠ²Ρ‹Π²Π°ΡΡΡŒ Π½Π° показаниях сопротивлСния. Π£ Ρ‚Π΅Ρ€ΠΌΠΎΠΌΠ΅Ρ‚Ρ€ΠΎΠ² сопротивлСния высокая Ρ‚ΠΎΡ‡Π½ΠΎΡΡ‚ΡŒ ΠΈΠ·ΠΌΠ΅Ρ€Π΅Π½ΠΈΠΉ, Π½ΠΎ ΠΌΠ°Π»Ρ‹Π΅ Π΄ΠΈΠ°ΠΏΠ°Π·ΠΎΠ½Ρ‹ Ρ‚Π΅ΠΌΠΏΠ΅Ρ€Π°Ρ‚ΡƒΡ€.

На ΠΏΡ€Π°ΠΊΡ‚ΠΈΠΊΠ΅, свойства ΠΏΡ€ΠΎΠ²ΠΎΠ΄Π½ΠΈΠΊΠΎΠ² ΠΏΡ€Π΅ΠΏΡΡ‚ΡΡ‚Π²ΠΎΠ²Π°Ρ‚ΡŒ ΠΏΡ€ΠΎΡ…ΠΎΠΆΠ΄Π΅Π½ΠΈΡŽΒ Ρ‚ΠΎΠΊΠ°Β ΠΈΡΠΏΠΎΠ»ΡŒΠ·ΡƒΡŽΡ‚ΡΡ ΠΎΡ‡Π΅Π½ΡŒ ΡˆΠΈΡ€ΠΎΠΊΠΎ.Β  ΠŸΡ€ΠΈΠΌΠ΅Ρ€ΠΎΠΌ ΠΌΠΎΠΆΠ΅Ρ‚ ΡΠ»ΡƒΠΆΠΈΡ‚ΡŒ Π»Π°ΠΌΠΏΠ° накаливания, Π³Π΄Π΅ Π½ΠΈΡ‚ΡŒ ΠΈΠ· Π²ΠΎΠ»ΡŒΡ„Ρ€Π°ΠΌΠ°, нагрСваСтся Π·Π° счСт высокого сопротивлСния ΠΌΠ΅Ρ‚Π°Π»Π»Π°, большой Π΄Π»ΠΈΠ½Ρ‹ ΠΈ ΡƒΠ·ΠΊΠΎΠ³ΠΎ сСчСния. Или любой Π½Π°Π³Ρ€Π΅Π²Π°Ρ‚Π΅Π»ΡŒΠ½Ρ‹ΠΉ ΠΏΡ€ΠΈΠ±ΠΎΡ€, Π³Π΄Π΅ ΡΠΏΠΈΡ€Π°Π»ΡŒ разогрСваСтся благодаря высокому ΡΠΎΠΏΡ€ΠΎΡ‚ΠΈΠ²Π»Π΅Π½ΠΈΡŽ. Π’ элСктротСхникС, элСмСнт Π³Π»Π°Π²Π½Ρ‹ΠΌ свойством ΠΊΠΎΡ‚ΠΎΡ€ΠΎΠ³ΠΎ являСтся сопротивлСниС, называСтся – рСзистор. РСзистор примСняСтся практичСски Π² любой элСктричСской схСмС.Β 

  • ΠŸΡ€ΠΎΡΠΌΠΎΡ‚Ρ€ΠΎΠ²: 9576
  • Π’ΠΈΠ΄Π΅ΠΎ с вопросом: РасчСт сопротивлСния ΠΏΡ€ΠΎΠ²ΠΎΠ΄Π½ΠΈΠΊΠ°

    Π‘Ρ‚Π΅Π½ΠΎΠ³Ρ€Π°ΠΌΠΌΠ° видСозаписи

    Π”ΠΈΠ°ΠΌΠ΅Ρ‚Ρ€ алюминиСвой ΠΏΡ€ΠΎΠ²ΠΎΠ»ΠΎΠΊΠΈ 10 ΠΌΠΈΠ»Π»ΠΈΠΌΠ΅Ρ‚Ρ€ΠΎΠ². НайдитС сопротивлСниС Ρ‚Π°ΠΊΠΎΠ³ΠΎ ΠΏΡ€ΠΎΠ²ΠΎΠ΄Π° Π΄Π»ΠΈΠ½ΠΎΠΉ 0,56 ΠΊΠΌ, ΠΈΡΠΏΠΎΠ»ΡŒΠ·ΡƒΠ΅ΠΌΠΎΠ³ΠΎ для ΠΏΠ΅Ρ€Π΅Π΄Π°Ρ‡ΠΈ энСргии. Π˜ΡΠΏΠΎΠ»ΡŒΠ·ΡƒΠΉΡ‚Π΅ Π·Π½Π°Ρ‡Π΅Π½ΠΈΠ΅ ΡƒΠ΄Π΅Π»ΡŒΠ½ΠΎΠ³ΠΎ сопротивлСния алюминия 2,65 ΡƒΠΌΠ½ΠΎΠΆΠ΅Π½Π½ΠΎΠ΅ Π½Π° 10 Π΄ΠΎ ΠΎΡ‚Ρ€ΠΈΡ†Π°Ρ‚Π΅Π»ΡŒΠ½ΠΎΠΉ восьмой ΠΎΠΌΠΌΠ΅Ρ‚Ρ€Π°.

    ΠœΡ‹ ΠΌΠΎΠΆΠ΅ΠΌ Π½Π°Ρ‡Π°Ρ‚ΡŒ с рисования эскиза описываСмого ΠΏΡ€ΠΎΠ²ΠΎΠ΄Π°. Π₯отя эскиз нарисован Π½Π΅ Π² ΠΌΠ°ΡΡˆΡ‚Π°Π±Π΅, ΠΎΠ½ Π΄Π°Π΅Ρ‚ Π½Π°ΠΌ прСдставлСниС ΠΎ Ρ€Π°Π·ΠΌΠ΅Ρ€Π°Ρ… алюминиСвой ΠΏΡ€ΠΎΠ²ΠΎΠ»ΠΎΠΊΠΈ.Наряду с Ρ€Π°Π·ΠΌΠ΅Ρ€Π°ΠΌΠΈ, Π½Π°ΠΌ сообщили, Ρ‡Ρ‚ΠΎ этот ΠΏΡ€ΠΎΠ²ΠΎΠ΄ ΠΈΠΌΠ΅Π΅Ρ‚ ΡƒΠ΄Π΅Π»ΡŒΠ½ΠΎΠ΅ сопротивлСниС, ΠΎΠ±ΠΎΠ·Π½Π°Ρ‡Π΅Π½Π½ΠΎΠ΅ грСчСской Π±ΡƒΠΊΠ²ΠΎΠΉ, ΠΎΡ‚ 2,65 ΡƒΠΌΠ½ΠΎΠΆΠ΅Π½Π½ΠΎΠ΅ Π½Π° 10 Π΄ΠΎ ΠΎΡ‚Ρ€ΠΈΡ†Π°Ρ‚Π΅Π»ΡŒΠ½ΠΎΠΉ восьмой ΠΎΠΌΠΌΠ΅Ρ‚Ρ€Π°. Учитывая всС это, ΠΌΡ‹ Ρ…ΠΎΡ‚ΠΈΠΌ Π½Π°ΠΉΡ‚ΠΈ сопротивлСниС этого ΠΏΡ€ΠΎΠ²ΠΎΠ΄Π°.

    Для этого Π½Π΅ΠΎΠ±Ρ…ΠΎΠ΄ΠΈΠΌΠΎ Π²ΡΠΏΠΎΠΌΠ½ΠΈΡ‚ΡŒ ΠΌΠ°Ρ‚Π΅ΠΌΠ°Ρ‚ΠΈΡ‡Π΅ΡΠΊΡƒΡŽ связь ΠΌΠ΅ΠΆΠ΄Ρƒ сопротивлСниСм ΠΈ ΡƒΠ΄Π΅Π»ΡŒΠ½Ρ‹ΠΌ сопротивлСниСм. Π‘ΠΎΠΏΡ€ΠΎΡ‚ΠΈΠ²Π»Π΅Π½ΠΈΠ΅ π‘Ÿ ΠΏΡ€ΠΎΠ²ΠΎΠ΄Π° Ρ€Π°Π²Π½ΠΎ Π΅Π³ΠΎ ΡƒΠ΄Π΅Π»ΡŒΠ½ΠΎΠΌΡƒ ΡΠΎΠΏΡ€ΠΎΡ‚ΠΈΠ²Π»Π΅Π½ΠΈΡŽ, ΡƒΠΌΠ½ΠΎΠΆΠ΅Π½Π½ΠΎΠΌΡƒ Π½Π° Π΅Π³ΠΎ Π΄Π»ΠΈΠ½Ρƒ, Π΄Π΅Π»Π΅Π½Π½ΡƒΡŽ Π½Π° Π΅Π³ΠΎ ΠΏΠ»ΠΎΡ‰Π°Π΄ΡŒ ΠΏΠΎΠΏΠ΅Ρ€Π΅Ρ‡Π½ΠΎΠ³ΠΎ сСчСния. И Π² нашСм случаС, ΠΏΠΎΡΠΊΠΎΠ»ΡŒΠΊΡƒ наша ΠΏΠ»ΠΎΡ‰Π°Π΄ΡŒ ΠΏΠΎΠΏΠ΅Ρ€Π΅Ρ‡Π½ΠΎΠ³ΠΎ сСчСния круглая, ΠΌΡ‹ Ρ‚Π°ΠΊΠΆΠ΅ ΠΌΠΎΠΆΠ΅ΠΌ Π²ΡΠΏΠΎΠΌΠ½ΠΈΡ‚ΡŒ ΠΏΠ»ΠΎΡ‰Π°Π΄ΡŒ ΠΊΡ€ΡƒΠ³Π° с Ρ‚ΠΎΡ‡ΠΊΠΈ зрСния Π΅Π³ΠΎ Π΄ΠΈΠ°ΠΌΠ΅Ρ‚Ρ€Π°, эта ΠΏΠ»ΠΎΡ‰Π°Π΄ΡŒ Ρ€Π°Π²Π½Π° πœ‹, Π΄Π΅Π»Π΅Π½Π½ΠΎΠΌΡƒ Π½Π° Ρ‡Π΅Ρ‚Ρ‹Ρ€Π΅ Ρ€Π°Π·Π° ΠΊΠ²Π°Π΄Ρ€Π°Ρ‚ Π΅Π³ΠΎ Π΄ΠΈΠ°ΠΌΠ΅Ρ‚Ρ€Π°.

    Наша Π·Π°Π΄Π°Ρ‡Π° состоит Π² Ρ‚ΠΎΠΌ, Ρ‡Ρ‚ΠΎΠ±Ρ‹ ΠΈΡΠΏΠΎΠ»ΡŒΠ·ΠΎΠ²Π°Ρ‚ΡŒ это ΡΠΎΠΎΡ‚Π½ΠΎΡˆΠ΅Π½ΠΈΠ΅ ΠΈ ΠΏΡ€Π΅Π΄ΠΎΡΡ‚Π°Π²Π»Π΅Π½Π½ΡƒΡŽ ΠΈΠ½Ρ„ΠΎΡ€ΠΌΠ°Ρ†ΠΈΡŽ для опрСдСлСния π‘Ÿ, Π³Π΄Π΅ π‘Ÿ — сопротивлСниС алюминиСвой ΠΏΡ€ΠΎΠ²ΠΎΠ»ΠΎΠΊΠΈ. ΠœΡ‹ ΠΌΠΎΠΆΠ΅ΠΌ Π½Π°Ρ‡Π°Ρ‚ΡŒ с Ρ‚ΠΎΠ³ΠΎ, Ρ‡Ρ‚ΠΎ подставим Π² это ΡƒΡ€Π°Π²Π½Π΅Π½ΠΈΠ΅ Π·Π°Π΄Π°Π½Π½ΠΎΠ΅ Π·Π½Π°Ρ‡Π΅Π½ΠΈΠ΅ ΡƒΠ΄Π΅Π»ΡŒΠ½ΠΎΠ³ΠΎ сопротивлСния 𝜚, Π° Ρ‚Π°ΠΊΠΆΠ΅ Π΄Π»ΠΈΠ½Ρƒ ΠΏΡ€ΠΎΠ²ΠΎΠ΄Π° Π² ΠΌΠ΅Ρ‚Ρ€Π°Ρ…. ΠžΠ±Ρ€Π°Ρ‚ΠΈΡ‚Π΅ Π²Π½ΠΈΠΌΠ°Π½ΠΈΠ΅, Ρ‡Ρ‚ΠΎ ΠΌΡ‹ пСрСсчитали ΠΊΠΈΠ»ΠΎΠΌΠ΅Ρ‚Ρ€Ρ‹. Как Ρ‚ΠΎΠ»ΡŒΠΊΠΎ эти значСния для 𝜚 ΠΈ 𝐿 входят Π² нашС ΡƒΡ€Π°Π²Π½Π΅Π½ΠΈΠ΅, наша послСдняя Π·Π°Π΄Π°Ρ‡Π° — ввСсти Π²Ρ‹Ρ€Π°ΠΆΠ΅Π½ΠΈΠ΅ для ΠΏΠ»ΠΎΡ‰Π°Π΄ΠΈ ΠΏΠΎΠΏΠ΅Ρ€Π΅Ρ‡Π½ΠΎΠ³ΠΎ сСчСния этого ΠΏΡ€ΠΎΠ²ΠΎΠ΄Π°.

    Глядя Π½Π° нашС ΡƒΡ€Π°Π²Π½Π΅Π½ΠΈΠ΅ для ΠΏΠ»ΠΎΡ‰Π°Π΄ΠΈ ΠΊΡ€ΡƒΠ³Π°, ΠΌΡ‹ Π²ΠΈΠ΄ΠΈΠΌ, Ρ‡Ρ‚ΠΎ ΠΎΠ½ΠΎ выраТаСтся Π² Π΄ΠΈΠ°ΠΌΠ΅Ρ‚Ρ€Π΅, ΠΊΠΎΡ‚ΠΎΡ€Ρ‹ΠΉ ΠΌΡ‹ ΠΈ Π΄Π°Π΅ΠΌ Π² Ρ„ΠΎΡ€ΠΌΡƒΠ»ΠΈΡ€ΠΎΠ²ΠΊΠ΅ Π·Π°Π΄Π°Ρ‡ΠΈ.Но Π² настоящСС врСмя наш Π΄ΠΈΠ°ΠΌΠ΅Ρ‚Ρ€ выраТаСтся Π² ΠΌΠΈΠ»Π»ΠΈΠΌΠ΅Ρ‚Ρ€Π°Ρ…, ΠΈ ΠΌΡ‹ Ρ…ΠΎΡ‚ΠΈΠΌ ΠΏΡ€Π΅ΠΎΠ±Ρ€Π°Π·ΠΎΠ²Π°Ρ‚ΡŒ Π΅Π³ΠΎ Π² ΠΌΠ΅Ρ‚Ρ€Ρ‹, Ρ‡Ρ‚ΠΎΠ±Ρ‹ ΠΎΠ½ΠΈ соотвСтствовали Π΅Π΄ΠΈΠ½ΠΈΡ†Π°ΠΌ Π² ΠΎΡΡ‚Π°Π»ΡŒΠ½ΠΎΠΉ части этого выраТСния для π‘Ÿ. Π’Π°ΠΊΠΈΠΌ ΠΎΠ±Ρ€Π°Π·ΠΎΠΌ, вмСсто 10 ΠΌΠΈΠ»Π»ΠΈΠΌΠ΅Ρ‚Ρ€ΠΎΠ², ΠΌΡ‹ запишСм Π΄ΠΈΠ°ΠΌΠ΅Ρ‚Ρ€ ΠΊΠ°ΠΊ 10 Π² ΠΎΡ‚Ρ€ΠΈΡ†Π°Ρ‚Π΅Π»ΡŒΠ½ΠΎΠΉ Ρ‚Ρ€Π΅Ρ‚ΠΈ ΠΌΠ΅Ρ‚Ρ€Π°. И это количСство возводится Π² ΠΊΠ²Π°Π΄Ρ€Π°Ρ‚ ΠΈ умноТаСтся Π½Π° Π½Π° Ρ‡Π΅Ρ‚Ρ‹Ρ€Π΅, Ρ‡Ρ‚ΠΎΠ±Ρ‹ Π½Π°ΠΉΡ‚ΠΈ ΠΏΠ»ΠΎΡ‰Π°Π΄ΡŒ ΠΏΠΎΠΏΠ΅Ρ€Π΅Ρ‡Π½ΠΎΠ³ΠΎ сСчСния.

    ΠŸΡ€Π΅ΠΆΠ΄Π΅ Ρ‡Π΅ΠΌ Π²Ρ‹Ρ‡ΠΈΡΠ»ΠΈΡ‚ΡŒ Π·Π½Π°Ρ‡Π΅Π½ΠΈΠ΅ π‘Ÿ, ΠΎΠ±Ρ€Π°Ρ‚ΠΈΡ‚Π΅ Π²Π½ΠΈΠΌΠ°Π½ΠΈΠ΅, Ρ‡Ρ‚ΠΎ происходит с Π΅Π΄ΠΈΠ½ΠΈΡ†Π°ΠΌΠΈ измСрСния Π² этом Π²Ρ‹Ρ€Π°ΠΆΠ΅Π½ΠΈΠΈ. Π’ частности, Π΅Π΄ΠΈΠ½ΠΈΡ†Ρ‹ измСрСния ΡΠΎΠΊΡ€Π°Ρ‰Π°ΡŽΡ‚ΡΡ ΠΎΡ‚ числитСля ΠΈ знамСнатСля, ΠΈ ΠΌΡ‹ остаСмся с Π΅Π΄ΠΈΠ½ΠΈΡ†Π°ΠΌΠΈ измСрСния сопротивлСния — ΠΎΠΌΠ°ΠΌΠΈ.Π­Ρ‚Π° доля составляСт 0,19 Ом. Π’ΠΎΡ‚ сопротивлСниС этого Π΄Π»ΠΈΠ½Π½ΠΎΠ³ΠΎ ΡƒΠ·ΠΊΠΎΠ³ΠΎ ΠΏΡ€ΠΎΠ²ΠΎΠ΄Π°.

    % PDF-1.7 % 6 0 obj > эндобдТ xref 6 74 0000000016 00000 Π½. 0000002085 00000 Π½. 0000002198 00000 Π½. 0000002735 00000 Π½. 0000002879 00000 ΠΏ. 0000003472 00000 Π½. 0000004007 00000 Π½. 0000004580 00000 Π½. 0000005063 00000 Π½. 0000005277 00000 Π½. 0000006081 00000 Π½. 0000006221 00000 Π½. 0000006553 00000 Π½. 0000006999 00000 Π½. 0000007767 00000 Π½. 0000008354 00000 Π½. 0000008669 00000 Π½. 0000008882 00000 Π½. 0000009291 00000 ΠΏ. 0000009781 00000 ΠΏ. 0000010448 00000 ΠΏ. 0000011141 00000 ΠΏ. 0000134013 00000 Π½. 0000134041 00000 Π½. 0000134114 00000 ΠΏ. 0000134230 00000 Π½. 0000134498 00000 Π½. 0000137641 00000 Π½. 0000137922 00000 Π½. 0000137991 00000 Π½. 0000138363 00000 Π½. 0000138388 00000 ΠΏ. 0000138885 00000 Π½. 0000139269 00000 Π½. 0000139531 00000 Π½. 0000139600 00000 Π½. QA @ 4L, X’0`

    Π—ΠΠšΠžΠ О ВОКАΠ₯ И ΠžΠ“Π ΠΠΠ˜Π§Π•ΠΠ˜Π―Π₯

    Π—ΠΠšΠžΠ О ВОКАΠ₯ И ΠžΠ“Π ΠΠΠ˜Π§Π•ΠΠ˜Π˜
    Рисунок 28.1. ЭлСктричСскоС ΠΏΠΎΠ»Π΅ Π² ΠΏΡ€ΠΎΠ²ΠΎΠ΄Π΅. Когда ΠΏΡ€ΠΎΠ²ΠΎΠ΄ ΠΏΠΎΠ΄ΠΊΠ»ΡŽΡ‡Π΅Π½ ΠΊ ΠΊΠ»Π΅ΠΌΠΌΠ°ΠΌ аккумулятора, элСктричСскоС ΠΏΠΎΠ»Π΅ создаСтся Π²Π½ΡƒΡ‚Ρ€ΠΈ ΠΏΡ€ΠΎΠ²ΠΎΠ΄Π° (см. рисунок 28.1). Π‘Π²ΠΎΠ±ΠΎΠ΄Π° элСктроны Π² ΠΏΡ€ΠΎΠ²ΠΎΠ΄Π΅ Π±ΡƒΠ΄ΡƒΡ‚ Π΄Π²ΠΈΠ³Π°Ρ‚ΡŒΡΡ Π² Π½Π°ΠΏΡ€Π°Π²Π»Π΅Π½ΠΈΠΈ, ΠΏΡ€ΠΎΡ‚ΠΈΠ²ΠΎΠΏΠΎΠ»ΠΎΠΆΠ½ΠΎΠΌ Π½Π°ΠΏΡ€Π°Π²Π»Π΅Π½ΠΈΡŽ поля Π»ΠΈΠ½ΠΈΠΉ. ЭлСктричСский заряд Π±ΡƒΠ΄Π΅Ρ‚ ΠΏΡ‹Ρ‚Π°Ρ‚ΡŒΡΡ ΠΏΠ΅Ρ€Π΅Ρ€Π°ΡΠΏΡ€Π΅Π΄Π΅Π»ΠΈΡ‚ΡŒΡΡ Ρ‚Π°ΠΊΠΈΠΌ ΠΎΠ±Ρ€Π°Π·ΠΎΠΌ, Ρ‡Ρ‚ΠΎΠ±Ρ‹ чистоС элСктричСскоС ΠΏΠΎΠ»Π΅ Π² ΠΏΡ€ΠΎΠ²ΠΎΠ΄Π΅ Ρ€Π°Π²Π½ΠΎ Π½ΡƒΠ»ΡŽ. Однако ΠΏΠΎΠ»ΠΎΠΆΠΈΡ‚Π΅Π»ΡŒΠ½Ρ‹ΠΉ ΠΊΠ»Π΅ΠΌΠΌΠ° Π±Π°Ρ‚Π°Ρ€Π΅ΠΈ дСйствуСт ΠΊΠ°ΠΊ сток для элСктронов, Π° ΠΎΡ‚Ρ€ΠΈΡ†Π°Ρ‚Π΅Π»ΡŒΠ½Π°Ρ ΠΊΠ»Π΅ΠΌΠΌΠ° дСйствуСт ΠΊΠ°ΠΊ источник элСктронов, ΠΈ Π½Π΅ΠΏΡ€Π΅Ρ€Ρ‹Π²Π½Ρ‹ΠΉ ΠΏΠΎΡ‚ΠΎΠΊ элСктронов Π±ΡƒΠ΄Π΅Ρ‚ созданный.Π­Ρ‚ΠΎΡ‚ Π½Π΅ΠΏΡ€Π΅Ρ€Ρ‹Π²Π½Ρ‹ΠΉ ΠΏΠΎΡ‚ΠΎΠΊ элСктронов называСтся элСктричСским Ρ‚ΠΎΠΊΠΎΠΌ . Ρ‚Π΅ΠΊΡƒΡ‰ΠΈΠΉ . Π‘ΠΈΠΌΠ²ΠΎΠ» Ρ‚ΠΎΠΊΠ° — I, Π° Π΅Π³ΠΎ Π΅Π΄ΠΈΠ½ΠΈΡ†Π° БИ — АмпСр (А). Π‘ΠΈΠ»Π° Ρ‚ΠΎΠΊΠ° ΠΎΠΏΡ€Π΅Π΄Π΅Π»Π΅Π½Π° ΠΊΠ°ΠΊ

    (28,1)

    Π³Π΄Π΅ dq — количСство заряда, ΠΊΠΎΡ‚ΠΎΡ€ΠΎΠ΅ ΠΏΡ€ΠΎΡ…ΠΎΠ΄ΠΈΡ‚ Ρ‡Π΅Ρ€Π΅Π· Π½Π΅ΠΊΠΎΡ‚ΠΎΡ€ΡƒΡŽ Π·Π°Π΄Π°Π½Π½ΡƒΡŽ Ρ‚ΠΎΡ‡ΠΊΡƒ Π½Π° ΠΏΡ€ΠΎΠ²ΠΎΠ΄Π΅. Π² Ρ‚Π΅Ρ‡Π΅Π½ΠΈΠ΅ ΠΏΠ΅Ρ€ΠΈΠΎΠ΄Π° Π²Ρ€Π΅ΠΌΠ΅Π½ΠΈ dt. Π’ΠΎΠΊ 1 А Ρ€Π°Π²Π΅Π½ 1 Кл / с. Π’ ΠΏΠ»ΠΎΡ‚Π½ΠΎΡΡ‚ΡŒ Ρ‚ΠΎΠΊΠ° Π”ΠΆ опрСдСляСтся ΠΊΠ°ΠΊ

    (28.2)

    Π³Π΄Π΅ I — Ρ‚ΠΎΠΊ, ΠΏΡ€ΠΎΡ‚Π΅ΠΊΠ°ΡŽΡ‰ΠΈΠΉ ΠΏΠΎ ΠΏΡ€ΠΎΠ²ΠΎΠ΄Π½ΠΈΠΊΡƒ, Π° A — ΠΏΠ»ΠΎΡ‰Π°Π΄ΡŒ ΠΏΠΎΠΏΠ΅Ρ€Π΅Ρ‡Π½ΠΎΠ³ΠΎ сСчСния ΠΏΡ€ΠΎΠ²ΠΎΠ΄Π½ΠΈΠΊΠ°. Π₯отя элСктроны Ρ‡ΡƒΠ²ΡΡ‚Π²ΡƒΡŽΡ‚ элСктричСскоС ΠΏΠΎΠ»Π΅ Π²Π½ΡƒΡ‚Ρ€ΠΈ ΠΏΡ€ΠΎΠ²ΠΎΠ΄Π½ΠΈΠΊΠ°, ΠΎΠ½ΠΈ Π½Π΅ Π±ΡƒΠ΄ΡƒΡ‚ ΡƒΡΠΊΠΎΡ€ΡΡ‚ΡŒΡΡ. Π­Π»Π΅ΠΊΡ‚Ρ€ΠΎΠ½Ρ‹ Π±ΡƒΠ΄Π΅Ρ‚ ΠΈΡΠΏΡ‹Ρ‚Ρ‹Π²Π°Ρ‚ΡŒ Π·Π½Π°Ρ‡ΠΈΡ‚Π΅Π»ΡŒΠ½ΠΎΠ΅ Ρ‚Ρ€Π΅Π½ΠΈΠ΅ Π² Ρ€Π΅Π·ΡƒΠ»ΡŒΡ‚Π°Ρ‚Π΅ столкновСний с ΠΏΠΎΠ»ΠΎΠΆΠΈΡ‚Π΅Π»ΡŒΠ½Ρ‹Π΅ ΠΈΠΎΠ½Ρ‹ Π² ΠΏΡ€ΠΎΠ²ΠΎΠ΄Π½ΠΈΠΊΠ΅. Π’ срСднСм элСктроны Π±ΡƒΠ΄ΡƒΡ‚ Π΄Π²ΠΈΠ³Π°Ρ‚ΡŒΡΡ с постоянная ΡΠΊΠΎΡ€ΠΎΡΡ‚ΡŒ ΠΎΡ‚ ΠΎΡ‚Ρ€ΠΈΡ†Π°Ρ‚Π΅Π»ΡŒΠ½ΠΎΠΉ ΠΊΠ»Π΅ΠΌΠΌΡ‹ аккумулятора ΠΊ ΠΏΠΎΠ»ΠΎΠΆΠΈΡ‚Π΅Π»ΡŒΠ½ΠΎΠΉ Π’Π΅Ρ€ΠΌΠΈΠ½Π°Π». Π˜Ρ… срСдняя ΡΠΊΠΎΡ€ΠΎΡΡ‚ΡŒ, Ρ‚Π°ΠΊΠΆΠ΅ называСмая Π΄Ρ€Π΅ΠΉΡ„ΠΎΠ²ΠΎΠΉ ΡΠΊΠΎΡ€ΠΎΡΡ‚ΡŒΡŽ v d , ΠΏΡ€ΠΎΠΏΠΎΡ€Ρ†ΠΈΠΎΠ½Π°Π»ΡŒΠ½ΠΎ элСктричСскому полю E

    (28.3)

    ΠŸΡ€ΠΈ Π·Π°Π΄Π°Π½Π½ΠΎΠΉ плотности элСктронов Π² ΠΏΡ€ΠΎΠ²ΠΎΠ΄Π½ΠΈΠΊΠ΅ ΡƒΠ²Π΅Π»ΠΈΡ‡Π΅Π½ΠΈΠ΅ Π΄Ρ€Π΅ΠΉΡ„Π° ΡΠΊΠΎΡ€ΠΎΡΡ‚ΡŒ ΠΊΠ°ΠΆΠ΄ΠΎΠ³ΠΎ ΠΈΠ· элСктронов ΡƒΠ²Π΅Π»ΠΈΡ‡ΠΈΡ‚ количСство проходящих элСктронов Π·Π°Π΄Π°Π½Π½ΠΎΠΉ Ρ‚ΠΎΡ‡ΠΊΠΎΠΉ Π½Π° ΠΏΡ€ΠΎΠ²ΠΎΠ΄Π½ΠΈΠΊΠ΅ Π² Π΅Π΄ΠΈΠ½ΠΈΡ†Ρƒ Π²Ρ€Π΅ΠΌΠ΅Π½ΠΈ. Π­Ρ‚ΠΎ ΠΏΠΎΠΊΠ°Π·Π°Π½ΠΎ Π² Рисунок 28.2. Π—Π° ΠΏΡ€ΠΎΠΌΠ΅ΠΆΡƒΡ‚ΠΎΠΊ Π²Ρ€Π΅ΠΌΠ΅Π½ΠΈ dt элСктроны ΠΏΡ€ΠΎΠΉΠ΄ΡƒΡ‚ Π² срСднСм ΠΎΠΊΠΎΠ»ΠΎ расстояниС, Ρ€Π°Π²Π½ΠΎΠ΅ dx, Π³Π΄Π΅

    (28.4)

    Рисунок 28.2. Π”Π²ΠΈΠΆΠ΅Π½ΠΈΠ΅ срСднСго элСктрона Π² ΠΏΡ€ΠΎΠ²ΠΎΠ΄Π½ΠΈΠΊΠ΅. ВсС элСктроны Π½Π° расстоянии dx ΠΎΡ‚ Ρ‚ΠΎΡ‡ΠΊΠΈ P Π±ΡƒΠ΄ΡƒΡ‚ поэтому ΠΏΡ€ΠΎΠΉΡ‚ΠΈ эту Ρ‚ΠΎΡ‡ΠΊΡƒ Π² Ρ‚Π΅Ρ‡Π΅Π½ΠΈΠ΅ Π²Ρ€Π΅ΠΌΠ΅Π½Π½ΠΎΠ³ΠΎ ΠΈΠ½Ρ‚Π΅Ρ€Π²Π°Π»Π° dt.ΠŸΡ€Π΅Π΄ΠΏΠΎΠ»ΠΎΠΆΠΈΠΌ, Ρ‡Ρ‚ΠΎ ΠΏΠ»ΠΎΡ‚Π½ΠΎΡΡ‚ΡŒ элСктронов Π² ΠΏΡ€ΠΎΠ²ΠΎΠ΄Π½ΠΈΠΊΠ΅ n элСктронов / ΠΌ 3 . ΠšΠΎΠ»ΠΈΡ‡Π΅ΡΡ‚Π²ΠΎ элСктронов dN, ΠΊΠΎΡ‚ΠΎΡ€Ρ‹Π΅ ΠΏΡ€ΠΎΠΉΠ΄ΡƒΡ‚ Ρ‡Π΅Ρ€Π΅Π· P Π·Π° ΠΏΡ€ΠΎΠΌΠ΅ΠΆΡƒΡ‚ΠΎΠΊ Π²Ρ€Π΅ΠΌΠ΅Π½ΠΈ dt, Ρ‚ΠΎΠ³Π΄Π° Π±ΡƒΠ΄Π΅Ρ‚ Ρ€Π°Π²Π½ΠΎ

    (28,5)

    ΠŸΠΎΡΠΊΠΎΠ»ΡŒΠΊΡƒ ΠΊΠ°ΠΆΠ΄Ρ‹ΠΉ элСктрон нСсСт заряд e, ΠΏΠΎΠ»Π½Ρ‹ΠΉ заряд dQ, ΠΊΠΎΡ‚ΠΎΡ€Ρ‹ΠΉ ΠΏΡ€ΠΎΠΉΠ΄Π΅Ρ‚ Ρ‚ΠΎΡ‡ΠΊΠ° P Π½Π° Π²Ρ€Π΅ΠΌΠ΅Π½Π½ΠΎΠΌ ΠΈΠ½Ρ‚Π΅Ρ€Π²Π°Π»Π΅ dt Ρ€Π°Π²Π½Π°

    (28,6)

    Π‘Π»Π΅Π΄ΠΎΠ²Π°Ρ‚Π΅Π»ΡŒΠ½ΠΎ, Ρ‚ΠΎΠΊ Ρ‡Π΅Ρ€Π΅Π· ΠΏΡ€ΠΎΠ²ΠΎΠ΄Π½ΠΈΠΊ Ρ€Π°Π²Π΅Π½

    . (28.7)

    Π£Ρ€Π°Π²Π½Π΅Π½ΠΈΠ΅ (28.7) ΠΏΠΎΠΊΠ°Π·Ρ‹Π²Π°Π΅Ρ‚, Ρ‡Ρ‚ΠΎ Ρ‚ΠΎΠΊ Π² ΠΏΡ€ΠΎΠ²ΠΎΠ΄Π½ΠΈΠΊΠ΅ ΠΏΡ€ΠΎΠΏΠΎΡ€Ρ†ΠΈΠΎΠ½Π°Π»Π΅Π½ силС Ρ‚ΠΎΠΊΠ°. ΠΏΠ»ΠΎΡ‰Π°Π΄ΡŒ ΠΏΠΎΠΏΠ΅Ρ€Π΅Ρ‡Π½ΠΎΠ³ΠΎ сСчСния ΠΏΡ€ΠΎΠ²ΠΎΠ΄Π½ΠΈΠΊΠ° ΠΈ ΠΏΡ€ΠΎΠΏΠΎΡ€Ρ†ΠΈΠΎΠ½Π°Π»ΡŒΠ½Π° скорости Π΄Ρ€Π΅ΠΉΡ„Π°. ΠŸΠΎΡΠΊΠΎΠ»ΡŒΠΊΡƒ ΡΠΊΠΎΡ€ΠΎΡΡ‚ΡŒ Π΄Ρ€Π΅ΠΉΡ„Π° ΠΏΡ€ΠΎΠΏΠΎΡ€Ρ†ΠΈΠΎΠ½Π°Π»ΡŒΠ½Π° элСктричСскому полю E, ΡΠ»Π΅Π΄ΡƒΡŽΡ‰Π΅Π΅ ΡΠΎΠΎΡ‚Π½ΠΎΡˆΠ΅Π½ΠΈΠ΅ выполняСтся для Ρ‚ΠΎΠΊΠ° Π² ΠΏΡ€ΠΎΠ²ΠΎΠ΄Π½ΠΈΠΊΠ΅:

    (28,8)

    ЭлСктричСскоС ΠΏΠΎΠ»Π΅ Π² ΠΏΡ€ΠΎΠ²ΠΎΠ΄Π½ΠΈΠΊΠ΅ опрСдСляСтся Π΅Π³ΠΎ Π΄Π»ΠΈΠ½ΠΎΠΉ L ΠΈ Π²Π΅Π»ΠΈΡ‡ΠΈΠ½ΠΎΠΉ Ρ€Π°Π·Π½ΠΎΡΡ‚ΡŒ ΠΏΠΎΡ‚Π΅Π½Ρ†ΠΈΠ°Π»ΠΎΠ² [Π”Π΅Π»ΡŒΡ‚Π°] V ΠΌΠ΅ΠΆΠ΄Ρƒ двумя Π΅Π³ΠΎ ΠΊΠΎΠ½Ρ†Π°ΠΌΠΈ (E = [Π”Π΅Π»ΡŒΡ‚Π°] V / L).Π’Π°ΠΊΠΈΠΌ ΠΎΠ±Ρ€Π°Π·ΠΎΠΌ, ΡƒΡ€Π°Π²Π½Π΅Π½ΠΈΠ΅ (28.8) ΠΌΠΎΠΆΠ½ΠΎ ΠΏΠ΅Ρ€Π΅ΠΏΠΈΡΠ°Ρ‚ΡŒ ΠΊΠ°ΠΊ

    (28,9)

    Π£Ρ€Π°Π²Π½Π΅Π½ΠΈΠ΅ (28.9) ΠΌΠΎΠΆΠ½ΠΎ ΠΏΠ΅Ρ€Π΅ΠΏΠΈΡΠ°Ρ‚ΡŒ ΠΊΠ°ΠΊ

    (28.10)

    ΠšΠΎΠ½ΡΡ‚Π°Π½Ρ‚Π° ΠΏΡ€ΠΎΠΏΠΎΡ€Ρ†ΠΈΠΎΠ½Π°Π»ΡŒΠ½ΠΎΡΡ‚ΠΈ [rho] называСтся ΡƒΠ΄Π΅Π»ΡŒΠ½Ρ‹ΠΌ сопротивлСниСм ΠΌΠ°Ρ‚Π΅Ρ€ΠΈΠ°Π». УдСльноС сопротивлСниС [rho] зависит ΠΎΡ‚ характСристик ΠΏΡ€ΠΎΠ²ΠΎΠ΄Π½ΠΈΠΊ ([rho] малСнький для Ρ…ΠΎΡ€ΠΎΡˆΠ΅Π³ΠΎ ΠΏΡ€ΠΎΠ²ΠΎΠ΄Π½ΠΈΠΊΠ°, ΠΈ [rho] ΠΎΡ‡Π΅Π½ΡŒ большой для Ρ…ΠΎΡ€ΠΎΡˆΠ΅Π³ΠΎ ΠΏΡ€ΠΎΠ²ΠΎΠ΄Π½ΠΈΠΊΠ° изолятор). Π‘ΠΎΠΏΡ€ΠΎΡ‚ΠΈΠ²Π»Π΅Π½ΠΈΠ΅ R ΠΏΡ€ΠΎΠ²ΠΎΠ΄Π½ΠΈΠΊΠ° опрСдСляСтся ΠΊΠ°ΠΊ

    . (28.11)

    Π•Π΄ΠΈΠ½ΠΈΡ†Π΅ΠΉ измСрСния сопротивлСния Π² систСмС БИ являСтся ΠΎΠΌ ([ОмСга]). Π˜ΡΠΏΠΎΠ»ΡŒΠ·ΡƒΡ сопротивлСниС R, ΠΌΡ‹ ΠΌΠΎΠΆΠ΅ΠΌ ΠΏΠ΅Ρ€Π΅ΠΏΠΈΡˆΠΈΡ‚Π΅ ΡƒΡ€Π°Π²Π½Π΅Π½ΠΈΠ΅ (28.10)

    (28.12)

    Π£Ρ€Π°Π²Π½Π΅Π½ΠΈΠ΅ (28.12) называСтся Π—Π°ΠΊΠΎΠ½ Ома . Π£Ρ€Π°Π²Π½Π΅Π½ΠΈΠ΅ (28.12) ΠΏΠΎΠΊΠ°Π·Ρ‹Π²Π°Π΅Ρ‚ Ρ‡Ρ‚ΠΎ Ρ‚ΠΎΠΊ Ρ‡Π΅Ρ€Π΅Π· ΠΏΡ€ΠΎΠ²ΠΎΠ΄Π½ΠΈΠΊ ΠΏΡ€ΠΎΠΏΠΎΡ€Ρ†ΠΈΠΎΠ½Π°Π»Π΅Π½ ΠΏΠΎΡ‚Π΅Π½Ρ†ΠΈΠ°Π»Ρƒ Ρ€Π°Π·Π½ΠΈΡ†Π° ΠΌΠ΅ΠΆΠ΄Ρƒ ΠΊΠΎΠ½Ρ†Π°ΠΌΠΈ ΠΏΡ€ΠΎΠ²ΠΎΠ΄Π½ΠΈΠΊΠ° ΠΈ ΠΎΠ±Ρ€Π°Ρ‚Π½ΠΎ ΠΏΡ€ΠΎΠΏΠΎΡ€Ρ†ΠΈΠΎΠ½Π°Π»ΡŒΠ½Π° Π΅Π³ΠΎ сопротивлСниС. Π£Ρ€Π°Π²Π½Π΅Π½ΠΈΠ΅ (28.12) Ρ‚Π°ΠΊΠΆΠ΅ ΠΏΠΎΠΊΠ°Π·Ρ‹Π²Π°Π΅Ρ‚, Ρ‡Ρ‚ΠΎ 1 [ОмСга] Ρ€Π°Π²Π½ΠΎ 1 Π’ / А.

    ΠŸΡ€ΠΈΠΌΠ΅Ρ€: Π—Π°Π΄Π°Ρ‡Π° 28.5

    ΠΠ»ΡŽΠΌΠΈΠ½ΠΈΠ΅Π²Ρ‹ΠΉ ΠΏΡ€ΠΎΠ²ΠΎΠ΄ ΠΈΠΌΠ΅Π΅Ρ‚ сопротивлСниС 0,10 Ом. Если Π²Ρ‹ нарисуСтС это ΠΏΡ€ΠΎΠ²ΠΎΠ΄ Ρ‡Π΅Ρ€Π΅Π· ΠΌΠ°Ρ‚Ρ€ΠΈΡ†Ρƒ, сдСлав Π΅Π΅ Ρ‚ΠΎΠ½ΡŒΡˆΠ΅ ΠΈ Π²Π΄Π²ΠΎΠ΅ Π΄Π»ΠΈΠ½Π½Π΅Π΅, Ρ‡Ρ‚ΠΎ Π±ΡƒΠ΄Π΅Ρ‚ Π΅Π΅ Π½ΠΎΠ²Ρ‹ΠΌ сопротивлСниС ?

    ΠΠ°Ρ‡Π°Π»ΡŒΠ½ΠΎΠ΅ сопротивлСниС R ΠΈ алюминиСвого ΠΏΡ€ΠΎΠ²ΠΎΠ΄Π° Π΄Π»ΠΈΠ½ΠΎΠΉ L ΠΈ ΠΏΠ»ΠΎΡ‰Π°Π΄ΡŒ ΠΏΠΎΠΏΠ΅Ρ€Π΅Ρ‡Π½ΠΎΠ³ΠΎ сСчСния A Ρ€Π°Π²Π½Π°

    (28,13)

    ΠΠ°Ρ‡Π°Π»ΡŒΠ½Ρ‹ΠΉ объСм ΠΏΡ€ΠΎΠ²ΠΎΠ»ΠΎΠΊΠΈ — Π». А.ПослС прохоТдСния ΠΏΡ€ΠΎΠ²ΠΎΠ΄Π° Ρ‡Π΅Ρ€Π΅Π· ΠΌΠ°Ρ‚Ρ€ΠΈΡ†Ρƒ Π΅Π΅ Π΄Π»ΠΈΠ½Π° измСнилась Π½Π° L ‘, Π° ΠΏΠ»ΠΎΡ‰Π°Π΄ΡŒ ΠΏΠΎΠΏΠ΅Ρ€Π΅Ρ‡Π½ΠΎΠ³ΠΎ сСчСния Ρ€Π°Π²Π½ΠΎ A ‘. Π’Π°ΠΊΠΈΠΌ ΠΎΠ±Ρ€Π°Π·ΠΎΠΌ, Π΅Π³ΠΎ ΠΊΠΎΠ½Π΅Ρ‡Π½Ρ‹ΠΉ объСм Ρ€Π°Π²Π΅Π½ L ‘A’. ΠŸΠΎΡΠΊΠΎΠ»ΡŒΠΊΡƒ ΠΏΠ»ΠΎΡ‚Π½ΠΎΡΡ‚ΡŒ алюминий Π½Π΅ мСняСтся, объСм ΠΏΡ€ΠΎΠ²ΠΎΠ΄Π° Π½Π΅ мСняСтся, ΠΈ поэтому Π½Π°Ρ‡Π°Π»ΡŒΠ½Ρ‹ΠΉ ΠΈ ΠΊΠΎΠ½Π΅Ρ‡Π½Ρ‹ΠΉ Ρ€Π°Π·ΠΌΠ΅Ρ€Ρ‹ ΠΏΡ€ΠΎΠ²ΠΎΠ»ΠΎΠΊΠΈ соотносятся:

    (28,14)

    ΠΈΠ»ΠΈ

    (28.15)

    ΠŸΡ€ΠΎΠ±Π»Π΅ΠΌΠ° Π·Π°ΠΊΠ»ΡŽΡ‡Π°Π΅Ρ‚ΡΡ Π² Ρ‚ΠΎΠΌ, Ρ‡Ρ‚ΠΎ Π΄Π»ΠΈΠ½Π° ΠΏΡ€ΠΎΠ²ΠΎΠ΄Π° ΡƒΠ²Π΅Π»ΠΈΡ‡Π΅Π½Π° Π²Π΄Π²ΠΎΠ΅ (L ‘= 2 L).Π’ поэтому конСчная ΠΏΠ»ΠΎΡ‰Π°Π΄ΡŒ ΠΏΠΎΠΏΠ΅Ρ€Π΅Ρ‡Π½ΠΎΠ³ΠΎ сСчСния A ‘связана с Π½Π°Ρ‡Π°Π»ΡŒΠ½Ρ‹ΠΌ ΠΏΠ»ΠΎΡ‰Π°Π΄ΡŒ ΠΏΠΎΠΏΠ΅Ρ€Π΅Ρ‡Π½ΠΎΠ³ΠΎ сСчСния A ΡΠ»Π΅Π΄ΡƒΡŽΡ‰ΠΈΠΌ ΠΎΠ±Ρ€Π°Π·ΠΎΠΌ:

    (28,16)

    ΠšΠΎΠ½Π΅Ρ‡Π½ΠΎΠ΅ сопротивлСниС ΠΏΡ€ΠΎΠ²ΠΎΠ΄Π° R f Ρ€Π°Π²Π½ΠΎ

    . (28,17)

    Π‘ΠΎΠΏΡ€ΠΎΡ‚ΠΈΠ²Π»Π΅Π½ΠΈΠ΅ ΠΏΡ€ΠΎΠ²ΠΎΠ΄Π° ΡƒΠ²Π΅Π»ΠΈΡ‡ΠΈΠ»ΠΎΡΡŒ Π² Ρ‡Π΅Ρ‚Ρ‹Ρ€Π΅ Ρ€Π°Π·Π° ΠΈ Ρ‚Π΅ΠΏΠ΅Ρ€ΡŒ составляСт 0,40. [ОмСга].

    Π•Π΄ΠΈΠ½ΠΈΡ†Ρ‹ измСрСния ΡƒΠ΄Π΅Π»ΡŒΠ½ΠΎΠ³ΠΎ сопротивлСния [rho] — ΠΎΠΌ-ΠΌΠ΅Ρ‚Ρ€ ([Omega] . ΠΌ). Π’ ΡƒΠ΄Π΅Π»ΡŒΠ½ΠΎΠ΅ сопротивлСниС Π±ΠΎΠ»ΡŒΡˆΠΈΠ½ΡΡ‚Π²Π° ΠΏΡ€ΠΎΠ²ΠΎΠ΄Π½ΠΈΠΊΠΎΠ² находится ΠΌΠ΅ΠΆΠ΄Ρƒ 10 -8 [ОмСга] . ΠΌ ΠΈ 10 -7 [Omega] . Π³. УдСльноС сопротивлСниС ΠΏΡ€ΠΎΠ²ΠΎΠ΄Π½ΠΈΠΊ зависит Π½Π΅ Ρ‚ΠΎΠ»ΡŒΠΊΠΎ ΠΎΡ‚ Ρ‚ΠΈΠΏΠ° ΠΌΠ°Ρ‚Π΅Ρ€ΠΈΠ°Π»Π°, Π½ΠΎ ΠΈ ΠΎΡ‚ Π΅Π³ΠΎ Ρ‚Π΅ΠΌΠΏΠ΅Ρ€Π°Ρ‚ΡƒΡ€Π°. УдСльноС сопротивлСниС изолятора колСблСтся Π² ΠΏΡ€Π΅Π΄Π΅Π»Π°Ρ… 10 11 [Omega] . m ΠΈ 10 17 [Omega] . ΠΌ. Π’ΠΎ всСх ΠΌΠ°Ρ‚Π΅Ρ€ΠΈΠ°Π»Π°Ρ… ΡƒΠ΄Π΅Π»ΡŒΠ½ΠΎΠ΅ сопротивлСниС ΡƒΠΌΠ΅Π½ΡŒΡˆΠ°Π΅Ρ‚ΡΡ с ΠΏΠΎΠ½ΠΈΠΆΠ΅Π½ΠΈΠ΅ΠΌ Ρ‚Π΅ΠΌΠΏΠ΅Ρ€Π°Ρ‚ΡƒΡ€Ρ‹.Π’ Ρƒ Π½Π΅ΠΊΠΎΡ‚ΠΎΡ€Ρ‹Ρ… ΠΌΠ°Ρ‚Π΅Ρ€ΠΈΠ°Π»ΠΎΠ², Ρ‚Π°ΠΊΠΈΡ… ΠΊΠ°ΠΊ свинСц, Ρ†ΠΈΠ½ΠΊ, ΠΎΠ»ΠΎΠ²ΠΎ ΠΈ Π½ΠΈΠΎΠ±ΠΈΠΉ, ΡƒΠ΄Π΅Π»ΡŒΠ½ΠΎΠ΅ сопротивлСниС исчСзаСт ΠΊΠΎΠ³Π΄Π° Ρ‚Π΅ΠΌΠΏΠ΅Ρ€Π°Ρ‚ΡƒΡ€Π° приблиТаСтся ΠΊ Π°Π±ΡΠΎΠ»ΡŽΡ‚Π½ΠΎΠΌΡƒ Π½ΡƒΠ»ΡŽ. ΠŸΡ€ΠΈ Ρ‚Π°ΠΊΠΈΡ… Π½ΠΈΠ·ΠΊΠΈΡ… Ρ‚Π΅ΠΌΠΏΠ΅Ρ€Π°Ρ‚ΡƒΡ€Π°Ρ… эти ΠΌΠ°Ρ‚Π΅Ρ€ΠΈΠ°Π»Ρ‹ Π΄Π΅ΠΌΠΎΠ½ΡΡ‚Ρ€ΠΈΡ€ΡƒΡŽΡ‚ ΡΠ²Π΅Ρ€Ρ…ΠΏΡ€ΠΎΠ²ΠΎΠ΄ΠΈΠΌΠΎΡΡ‚ΡŒ .

    ΠŸΡ€ΠΈΠΌΠ΅Ρ€: Π·Π°Π΄Π°Ρ‡Π° 28.17

    ΠšΠΎΠ½Π΄ΠΈΡ†ΠΈΠΎΠ½Π΅Ρ€ Π² Π΄ΠΎΠΌΠ΅ потрСбляСт Ρ‚ΠΎΠΊ 12 А. ΠŸΡ€Π΅Π΄ΠΏΠΎΠ»ΠΎΠΆΠΈΠΌ, Ρ‡Ρ‚ΠΎ ΠΏΠ°Ρ€Π° ΠΏΡ€ΠΎΠ²ΠΎΠ΄ΠΎΠ², ΡΠΎΠ΅Π΄ΠΈΠ½ΡΡŽΡ‰ΠΈΡ… ΠΊΠΎΠ½Π΄ΠΈΡ†ΠΈΠΎΠ½Π΅Ρ€ с Π±Π»ΠΎΠΊΠΎΠΌ ΠΏΡ€Π΅Π΄ΠΎΡ…Ρ€Π°Π½ΠΈΡ‚Π΅Π»Π΅ΠΉ, β„–10 ΠΌΠ΅Π΄Π½Ρ‹Π΅ ΠΏΡ€ΠΎΠ²ΠΎΠ΄Π° Π΄ΠΈΠ°ΠΌΠ΅Ρ‚Ρ€ΠΎΠΌ 0.259 см ΠΈ Π΄Π»ΠΈΠ½ΠΎΠΉ ΠΏΠΎ 25 ΠΌ.

    Π°) КакоС ΠΏΠ°Π΄Π΅Π½ΠΈΠ΅ ΠΏΠΎΡ‚Π΅Π½Ρ†ΠΈΠ°Π»Π° Π½Π° ΠΊΠ°ΠΆΠ΄ΠΎΠΌ ΠΏΡ€ΠΎΠ²ΠΎΠ΄Π΅? ΠŸΡ€Π΅Π΄ΠΏΠΎΠ»ΠΎΠΆΠΈΠΌ, Ρ‡Ρ‚ΠΎ напряТСниС доставлСно Π½Π° Π΄ΠΎΠΌ Ρ€ΠΎΠ²Π½ΠΎ 110 Π’ Π½Π° Π±Π»ΠΎΠΊ ΠΏΡ€Π΅Π΄ΠΎΡ…Ρ€Π°Π½ΠΈΡ‚Π΅Π»Π΅ΠΉ. КакоС напряТСниС поставили ΠΊΠΎΠ½Π΄ΠΈΡ†ΠΈΠΎΠ½Π΅Ρ€?

    Π±) Π’ Π½Π΅ΠΊΠΎΡ‚ΠΎΡ€Ρ‹Ρ… старых Π΄ΠΎΠΌΠ°Ρ… ΠΈΡΠΏΠΎΠ»ΡŒΠ·ΡƒΠ΅Ρ‚ΡΡ ΠΌΠ΅Π΄Π½Ρ‹ΠΉ ΠΏΡ€ΠΎΠ²ΠΎΠ΄ β„– 12 Π΄ΠΈΠ°ΠΌΠ΅Ρ‚Ρ€ΠΎΠΌ 0,205. см. ΠŸΠΎΠ²Ρ‚ΠΎΡ€ΠΈΡ‚Π΅ расчСт части (Π°) для этого ΠΏΡ€ΠΎΠ²ΠΎΠ΄Π°.

    Рисунок 28.3. ЭлСктросхСма ΠΊΠΎΠ½Π΄ΠΈΡ†ΠΈΠΎΠ½Π΅Ρ€Π° Π² ΠΏΡ€ΠΎΠ±Π»Π΅ΠΌΠ΅ 28.17.

    Π°) УдСльноС сопротивлСниС ΠΌΠ΅Π΄ΠΈ Ρ€Π°Π²Π½ΠΎ 1.7 x 10 -8 [Omega] . ΠΌ (см. Π’Π°Π±Π»ΠΈΡ†Ρƒ 28.1). Π‘ΠΎΠΏΡ€ΠΎΡ‚ΠΈΠ²Π»Π΅Π½ΠΈΠ΅ R Cu ΠΊΠ°ΠΆΠ΄ΠΎΠ³ΠΎ ΠΌΠ΅Π΄Π½ΠΎΠ³ΠΎ ΠΏΡ€ΠΎΠ²ΠΎΠ΄Π° Ρ€Π°Π²Π½ΠΎ Π½Π° Π½ΠΎΠΌΠ΅Ρ€

    (28.18)

    Π³Π΄Π΅ L — Π΄Π»ΠΈΠ½Π° ΠΏΡ€ΠΎΠ²ΠΎΠ»ΠΎΠΊΠΈ, Π° d — Π΅Π΅ Π΄ΠΈΠ°ΠΌΠ΅Ρ‚Ρ€. Π’ΠΎΠΊ I Π΅ΡΡ‚ΡŒ ΠΏΡ€ΠΎΡ‚Π΅ΠΊΠ°Π΅Ρ‚ ΠΏΠΎ ΠΏΡ€ΠΎΠ²ΠΎΠ΄Π°ΠΌ ΠΈ I = 12 A. ПадСниС напряТСния [Delta] V Π½Π° ΠΊΠ°ΠΆΠ΄ΠΎΠΌ ΠΏΡ€ΠΎΠ²ΠΎΠ΄ Ρ€Π°Π²Π΅Π½

    (28,19)

    На рисункС 28.3 схСматично ΠΏΠΎΠΊΠ°Π·Π°Π½Π° элСктричСская схСма ΠΊΠΎΠ½Π΄ΠΈΡ†ΠΈΠΎΠ½Π΅Ρ€Π°. схСма.НапряТСниС Π½Π° Π±Π»ΠΎΠΊΠ΅ ΠΊΠΎΠ½Π΄ΠΈΡ†ΠΈΠΎΠ½Π΅Ρ€Π° 110 — 2 . [Π”Π΅Π»ΡŒΡ‚Π°] V, Π³Π΄Π΅ [Π”Π΅Π»ΡŒΡ‚Π°] V опрСдСляСтся ΡƒΡ€Π°Π²Π½Π΅Π½ΠΈΠ΅ΠΌ (28.19). Π”Π»ΠΈΠ½Π° Π΄Π»ΠΈΠ½Π° ΠΊΠ°ΠΆΠ΄ΠΎΠ³ΠΎ ΠΌΠ΅Π΄Π½ΠΎΠ³ΠΎ кабСля составляСт 25 ΠΌ, Π° Π΅Π³ΠΎ Π΄ΠΈΠ°ΠΌΠ΅Ρ‚Ρ€ Ρ€Π°Π²Π΅Π½ 0,259 см. НапряТСниС ΠΏΠ°Π΄Π΅Π½ΠΈΠ΅ Π½Π° ΠΊΠ°ΠΆΠ΄ΠΎΠΌ ΠΏΡ€ΠΎΠ²ΠΎΠ΄Π΅, Ρ‚Π°ΠΊΠΈΠΌ ΠΎΠ±Ρ€Π°Π·ΠΎΠΌ, Ρ€Π°Π²Π½ΠΎ

    (28.20)

    Π’Π°ΠΊΠΈΠΌ ΠΎΠ±Ρ€Π°Π·ΠΎΠΌ, напряТСниС Π½Π° Π±Π»ΠΎΠΊΠ΅ ΠΏΠ΅Ρ€Π΅ΠΌΠ΅Π½Π½ΠΎΠ³ΠΎ Ρ‚ΠΎΠΊΠ° Ρ€Π°Π²Π½ΠΎ 108,1 Π’.

    Π±) ΠŸΡ€ΠΎΠ²ΠΎΠ»ΠΎΠΊΠ° β„– 12 ΠΈΠΌΠ΅Π΅Ρ‚ Π΄ΠΈΠ°ΠΌΠ΅Ρ‚Ρ€ 0,205 см. ПадСниС напряТСния Π½Π° этот ΠΏΡ€ΠΎΠ²ΠΎΠ΄ Ρ€Π°Π²Π΅Π½

    (28.21)

    Π° напряТСниС Π½Π° Π±Π»ΠΎΠΊΠ΅ ΠΏΠ΅Ρ€Π΅ΠΌΠ΅Π½Π½ΠΎΠ³ΠΎ Ρ‚ΠΎΠΊΠ° Ρ€Π°Π²Π½ΠΎ 106,9 Π’.

    ΠŸΡ€ΠΈΠΌΠ΅Ρ€: Π—Π°Π΄Π°Ρ‡Π° 28.12

    Π›Π­ΠŸ высокого напряТСния ΠΈΠΌΠ΅Π΅Ρ‚ Π°Π»ΡŽΠΌΠΈΠ½ΠΈΠ΅Π²Ρ‹ΠΉ кабСль Π΄ΠΈΠ°ΠΌΠ΅Ρ‚Ρ€ΠΎΠΌ 3,0 ΠΌΠΌ. см, ΠΏΡ€ΠΎΡ‚ΡΠΆΠ΅Π½Π½ΠΎΡΡ‚ΡŒΡŽ 200 ΠΊΠΌ. КакоС сопротивлСниС Ρƒ этого кабСля?

    УдСльноС сопротивлСниС алюминия составляСт 2,8 x 10 -8 Ом. Π΄Π»ΠΈΠ½Π° кабСль 200 ΠΊΠΌ ΠΈΠ»ΠΈ 2 Ρ… 10 5 ΠΌ. Π”ΠΈΠ°ΠΌΠ΅Ρ‚Ρ€ кабСля 3 см. Π° Π΅Π³ΠΎ ΠΏΠ»ΠΎΡ‰Π°Π΄ΡŒ ΠΏΠΎΠΏΠ΅Ρ€Π΅Ρ‡Π½ΠΎΠ³ΠΎ сСчСния Ρ€Π°Π²Π½Π° Ο€ (d / 2) 2 ΠΈΠ»ΠΈ 7.1 Ρ… 10 -4 ΠΌ 2 . ΠŸΠΎΠ΄ΡΡ‚Π°Π²Π»ΡΡ эти значСния Π² ΡƒΡ€Π°Π²Π½Π΅Π½ΠΈΠ΅ (28.11), ΠΏΠΎΠ»ΡƒΡ‡Π°Π΅ΠΌ сопротивлСниС кабСля ΠΌΠΎΠΆΠ½ΠΎ ΠΎΠΏΡ€Π΅Π΄Π΅Π»ΠΈΡ‚ΡŒ

    (28.22)

    Устройство, ΡΠΏΠ΅Ρ†ΠΈΠ°Π»ΡŒΠ½ΠΎ Ρ€Π°Π·Ρ€Π°Π±ΠΎΡ‚Π°Π½Π½ΠΎΠ΅ для обСспСчСния высокого сопротивлСния, называСтся рСзистор. ΠžΠ±ΠΎΠ·Π½Π°Ρ‡Π΅Π½ΠΈΠ΅ рСзистора Π½Π° ΠΏΡ€ΠΈΠ½Ρ†ΠΈΠΏΠΈΠ°Π»ΡŒΠ½ΠΎΠΉ схСмС — зигзагообразная линия (см. Рисунок 28.4).

    Рисунок 28.4. Π‘ΠΈΠΌΠ²ΠΎΠ» рСзистора. На рисункС 28.5 ΠΏΠΎΠΊΠ°Π·Π°Π½Ρ‹ Π΄Π²Π° рСзистора с сопротивлСниСм R 1 . ΠΈ R 2 соСдинСны ΠΏΠΎΡΠ»Π΅Π΄ΠΎΠ²Π°Ρ‚Π΅Π»ΡŒΠ½ΠΎ.ΠŸΡ€Π΅Π΄ΠΏΠΎΠ»ΠΎΠΆΠΈΠΌ, Ρ‡Ρ‚ΠΎ Ρ‚ΠΎΠΊ, ΠΏΡ€ΠΎΡ‚Π΅ΠΊΠ°ΡŽΡ‰ΠΈΠΉ Ρ‡Π΅Ρ€Π΅Π· Ρ†Π΅ΠΏΡŒ Ρ€Π°Π²Π½Π° I. ПадСниС напряТСния [Π”Π΅Π»ΡŒΡ‚Π°] Π’ 1 Π½Π° рСзисторС R 1 Ρ€Π°Π²Π½ΠΎ

    (28,23)

    ΠΈ ΠΏΠ°Π΄Π΅Π½ΠΈΠ΅ напряТСния [Delta] V 2 Π½Π° рСзисторС R 2 составляСт Ρ€Π°Π²Π½ΠΎ

    (28,24)

    Π Π°Π·Π½ΠΎΡΡ‚ΡŒ ΠΏΠΎΡ‚Π΅Π½Ρ†ΠΈΠ°Π»ΠΎΠ² [Delta] V Π² ΠΏΠΎΡΠ»Π΅Π΄ΠΎΠ²Π°Ρ‚Π΅Π»ΡŒΠ½ΠΎΠΉ Ρ†Π΅ΠΏΠΈ Ρ€Π°Π²Π½Π°

    . (28,25)

    Π£Ρ€Π°Π²Π½Π΅Π½ΠΈΠ΅ (28.25) ΠΏΠΎΠΊΠ°Π·Ρ‹Π²Π°Π΅Ρ‚, Ρ‡Ρ‚ΠΎ Π΄Π²Π° ΠΏΠΎΡΠ»Π΅Π΄ΠΎΠ²Π°Ρ‚Π΅Π»ΡŒΠ½ΠΎ Π²ΠΊΠ»ΡŽΡ‡Π΅Π½Π½Ρ‹Ρ… рСзистора Π΄Π΅ΠΉΡΡ‚Π²ΡƒΡŽΡ‚ ΠΊΠ°ΠΊ ΠΎΠ΄ΠΈΠ½ рСзистор с сопротивлСниСм, Ρ€Π°Π²Π½Ρ‹ΠΌ суммС сопротивлСний рСзистора 1 ΠΈ сопротивлСниС рСзистора 2

    (28.26)

    Рисунок 28.5. Π”Π²Π° рСзистора соСдинСны ΠΏΠΎΡΠ»Π΅Π΄ΠΎΠ²Π°Ρ‚Π΅Π»ΡŒΠ½ΠΎ. На рисункС 28.6 ΠΏΠΎΠΊΠ°Π·Π°Π½Ρ‹ Π΄Π²Π° ΠΏΠ°Ρ€Π°Π»Π»Π΅Π»ΡŒΠ½ΠΎ Π²ΠΊΠ»ΡŽΡ‡Π΅Π½Π½Ρ‹Ρ… рСзистора. Π’ Π’ этой схСмС Ρ‚ΠΎΠΊ Ρ‡Π΅Ρ€Π΅Π· ΠΊΠ°ΠΆΠ΄Ρ‹ΠΉ рСзистор Π±ΡƒΠ΄Π΅Ρ‚ Ρ€Π°Π·Π½Ρ‹ΠΌ, Π½ΠΎ ΠΏΠ°Π΄Π΅Π½ΠΈΠ΅ напряТСния [Delta] V Π½Π° ΠΊΠ°ΠΆΠ΄ΠΎΠΌ рСзисторС Π±ΡƒΠ΄Π΅Ρ‚ ΠΎΠ΄ΠΈΠ½Π°ΠΊΠΎΠ²Ρ‹ΠΌ. Π˜ΡΠΏΠΎΠ»ΡŒΠ·ΡƒΡ Π·Π°ΠΊΠΎΠ½ Ома Ρ‚ΠΎΠΊ I 1 , ΠΏΡ€ΠΎΡ‚Π΅ΠΊΠ°ΡŽΡ‰ΠΈΠΉ Ρ‡Π΅Ρ€Π΅Π· рСзистор R 1 , ΠΌΠΎΠΆΠ΅Ρ‚ Π±Ρ‹Ρ‚ΡŒ вычислСно

    (28.27)

    Π° Ρ‚ΠΎΠΊ I 2 , ΠΏΡ€ΠΎΡ‚Π΅ΠΊΠ°ΡŽΡ‰ΠΈΠΉ Ρ‡Π΅Ρ€Π΅Π· рСзистор R 2 , Ρ€Π°Π²Π΅Π½ Π½Π° Π½ΠΎΠΌΠ΅Ρ€

    (28.28)

    ΠŸΠΎΠ»Π½Ρ‹ΠΉ Ρ‚ΠΎΠΊ, ΠΏΡ€ΠΎΡ‚Π΅ΠΊΠ°ΡŽΡ‰ΠΈΠΉ ΠΏΠΎ Ρ†Π΅ΠΏΠΈ, Ρ€Π°Π²Π΅Π½ суммС Ρ‚ΠΎΠΊΠΈ Ρ‡Π΅Ρ€Π΅Π· ΠΊΠ°ΠΆΠ΄Ρ‹ΠΉ рСзистор

    (28.29)

    Π’Π°ΠΊΠΈΠΌ ΠΎΠ±Ρ€Π°Π·ΠΎΠΌ, схСма рСзисторов, показанная Π½Π° рисункС 28.6, эквивалСнтна ΠΎΠ΄ΠΈΠ½ΠΎΡ‡Π½ΠΎΠΉ рСзистор R, Π³Π΄Π΅ R ΠΌΠΎΠΆΠ½ΠΎ ΠΏΠΎΠ»ΡƒΡ‡ΠΈΡ‚ΡŒ ΠΈΠ· ΡΠ»Π΅Π΄ΡƒΡŽΡ‰Π΅Π³ΠΎ ΡΠΎΠΎΡ‚Π½ΠΎΡˆΠ΅Π½ΠΈΡ

    (28.30)

    Π£Ρ€Π°Π²Π½Π΅Π½ΠΈΠ΅ (28.30) ΠΏΠΎΠΊΠ°Π·Ρ‹Π²Π°Π΅Ρ‚, Ρ‡Ρ‚ΠΎ сопротивлСниС ΠΏΠ°Ρ€Π°Π»Π»Π΅Π»ΡŒΠ½ΠΎΠΉ ΠΊΠΎΠΌΠ±ΠΈΠ½Π°Ρ†ΠΈΠΈ рСзисторов всСгда мСньшС сопротивлСния ΠΊΠ°ΠΆΠ΄ΠΎΠ³ΠΎ Π² ΠΎΡ‚Π΄Π΅Π»ΡŒΠ½ΠΎΡΡ‚ΠΈ рСзисторы.

    Рисунок 28.6. Π”Π²Π° рСзистора ΠΏΠΎΠ΄ΠΊΠ»ΡŽΡ‡Π΅Π½Ρ‹ ΠΏΠ°Ρ€Π°Π»Π»Π΅Π»ΡŒΠ½ΠΎ.

    ΠŸΡ€ΠΈΠΌΠ΅Ρ€: Π·Π°Π΄Π°Ρ‡Π° 28.41

    ΠŸΡ€ΠΎΠΌΡ‹ΡˆΠ»Π΅Π½Π½Ρ‹Π΅ свСрхпроводящиС ΠΊΠ°Π±Π΅Π»ΠΈ состоят ΠΈΠ· Π½ΠΈΡ‚Π΅ΠΉ свСрхпроводящСго ΠΏΡ€ΠΎΠ²ΠΎΠ΄Π°, Π·Π°ΠΊΠ»ΡŽΡ‡Π΅Π½Π½ΠΎΠ³ΠΎ Π² ΠΌΠ°Ρ‚Ρ€ΠΈΡ†Ρƒ ΠΈΠ· ΠΌΠ΅Π΄ΠΈ. Пока Π½ΠΈΡ‚ΠΈ свСрхпроводящиС, Π² Π½ΠΈΡ… Ρ‚Π΅Ρ‡Π΅Ρ‚ вСсь Ρ‚ΠΎΠΊ, ΠΈ Π½Π΅Ρ‚ Ρ‚ΠΎΠΊΠ° Ρ‚Π΅Ρ‡Π΅Ρ‚ Π² мСдь.Но Ссли ΡΠ²Π΅Ρ€Ρ…ΠΏΡ€ΠΎΠ²ΠΎΠ΄ΠΈΠΌΠΎΡΡ‚ΡŒ Π²Π½Π΅Π·Π°ΠΏΠ½ΠΎ Π²Ρ‹ΠΉΠ΄Π΅Ρ‚ ΠΈΠ· строя ΠΈΠ·-Π·Π° ΠΏΠΎΠ²Ρ‹ΡˆΠ΅Π½ΠΈΠ΅ Ρ‚Π΅ΠΌΠΏΠ΅Ρ€Π°Ρ‚ΡƒΡ€Ρ‹, Ρ‚ΠΎΠΊ ΠΌΠΎΠΆΠ΅Ρ‚ ΠΏΡ€ΠΎΠ»ΠΈΡ‚ΡŒΡΡ Π½Π° мСдь; это ΠΏΡ€Π΅Π΄ΠΎΡ‚Π²Ρ€Π°Ρ‰Π°Π΅Ρ‚ ΠΏΠΎΠ²Ρ€Π΅ΠΆΠ΄Π΅Π½ΠΈΠ΅ Π½ΠΈΡ‚Π΅ΠΉ свСрхпроводника. РассчитайтС сопротивлСниС Π½Π° ΠΌΠ΅Ρ‚Ρ€ Π΄Π»ΠΈΠ½Ρ‹ ΠΌΠ΅Π΄Π½ΠΎΠΉ ΠΌΠ°Ρ‚Ρ€ΠΈΡ†Ρ‹. МСдная ΠΌΠ°Ρ‚Ρ€ΠΈΡ†Π° ΠΈΠΌΠ΅Π΅Ρ‚ Π΄ΠΈΠ°ΠΌΠ΅Ρ‚Ρ€ 0,7 ΠΌΠΌ, Π° каТдая ΠΈΠ· 2100 Π½ΠΈΡ‚Π΅ΠΉ ΠΈΠΌΠ΅Π΅Ρ‚ Π΄ΠΈΠ°ΠΌΠ΅Ρ‚Ρ€ 0,01 ΠΌΠΌ.

    Рассмотрим 1 ΠΌΠ΅Ρ‚Ρ€ кабСля. ΠŸΠ»ΠΎΡ‰Π°Π΄ΡŒ ΠΏΠΎΠΏΠ΅Ρ€Π΅Ρ‡Π½ΠΎΠ³ΠΎ сСчСния ΠΊΠ°ΠΆΠ΄ΠΎΠΉ Π½ΠΈΡ‚ΠΈ Π½Π°ΠΊΠ°Π»Π° [ΠΏΠΈ] . (Π³ / 2) 2 = 7.9 x 10 -11 ΠΌ 2 . Π’ ΠΏΠ»ΠΎΡ‰Π°Π΄ΡŒ ΠΏΠΎΠΏΠ΅Ρ€Π΅Ρ‡Π½ΠΎΠ³ΠΎ сСчСния 2100 Π½ΠΈΡ‚Π΅ΠΉ Ρ€Π°Π²Π½Π° 1,65 x 10 -7 ΠΌ 2 . Π”ΠΈΠ°ΠΌΠ΅Ρ‚Ρ€ ΠΌΠ΅Π΄Π½ΠΎΠΉ ΠΌΠ°Ρ‚Ρ€ΠΈΡ†Ρ‹ Ρ€Π°Π²Π΅Π½ 0,7 ΠΌΠΌ, Π° Π΅Π΅ Π΄ΠΈΠ°ΠΌΠ΅Ρ‚Ρ€ Ρ€Π°Π²Π΅Π½ 0,7 ΠΌΠΌ. ΠΏΠ»ΠΎΡ‰Π°Π΄ΡŒ ΠΏΠΎΠΏΠ΅Ρ€Π΅Ρ‡Π½ΠΎΠ³ΠΎ сСчСния Ρ€Π°Π²Π½Π° 1,54 Ρ… 10 -6 ΠΌ 2 . Π’ ΠΏΠ»ΠΎΡ‰Π°Π΄ΡŒ самой ΠΌΠ΅Π΄ΠΈ, Ρ‚Π°ΠΊΠΈΠΌ ΠΎΠ±Ρ€Π°Π·ΠΎΠΌ, Ρ€Π°Π²Π½Π° 1,37 x 10 -6 ΠΌ 2 . Π‘ΠΎΠΏΡ€ΠΎΡ‚ΠΈΠ²Π»Π΅Π½ΠΈΠ΅ ΠΌΠ΅Π΄Π½ΠΎΠΉ ΠΌΠ°Ρ‚Ρ€ΠΈΡ†Ρ‹ Π½Π° Π΅Π΄ΠΈΠ½ΠΈΡ†Ρƒ Π΄Π»ΠΈΠ½Ρ‹ Ρ€Π°Π²Π½ΠΎ Π½Π° Π½ΠΎΠΌΠ΅Ρ€

    (28.31)

    ΠŸΡ€Π΅Π΄ΠΏΠΎΠ»ΠΎΠΆΠΈΠΌ, Ρ‡Ρ‚ΠΎ ΡƒΠ΄Π΅Π»ΡŒΠ½ΠΎΠ΅ сопротивлСниС Π½ΠΈΡ‚ΠΈ ΠΏΡ€ΠΈ ΠΊΠΎΠΌΠ½Π°Ρ‚Π½ΠΎΠΉ Ρ‚Π΅ΠΌΠΏΠ΅Ρ€Π°Ρ‚ΡƒΡ€Π΅ Ρ‚Π°ΠΊΠΎΠ΅ ΠΆΠ΅, ΠΊΠ°ΠΊ Ρƒ ΡƒΠ΄Π΅Π»ΡŒΠ½ΠΎΠ΅ сопротивлСниС ΠΌΠ΅Π΄ΠΈ. Π‘ΠΎΠΏΡ€ΠΎΡ‚ΠΈΠ²Π»Π΅Π½ΠΈΠ΅ ΠΊΠ°ΠΆΠ΄ΠΎΠΉ свСрхпроводящСй Π½ΠΈΡ‚ΠΈ составляСт Ρ€Π°Π²Π½ΠΎ

    (28.32)

    ΠŸΡ€ΠΎΠ²ΠΎΠ΄ ΠΌΠΎΠΆΠ½ΠΎ Ρ€Π°ΡΡΠΌΠ°Ρ‚Ρ€ΠΈΠ²Π°Ρ‚ΡŒ ΠΊΠ°ΠΊ ΠΏΠ°Ρ€Π°Π»Π»Π΅Π»ΡŒΠ½ΡƒΡŽ Ρ†Π΅ΠΏΡŒ ΠΎΠ΄Π½ΠΎΠ³ΠΎ рСзистора, ΠΏΡ€Π΅Π΄ΡΡ‚Π°Π²Π»ΡΡŽΡ‰Π΅Π³ΠΎ сопротивлСниС ΠΌΠ΅Π΄Π½ΠΎΠΉ ΠΌΠ°Ρ‚Ρ€ΠΈΡ†Ρ‹ ΠΈ 2100 рСзисторов, ΠΏΡ€Π΅Π΄ΡΡ‚Π°Π²Π»ΡΡŽΡ‰ΠΈΡ… 2100 ΠΆΠΈΠ»Ρ‹ свСрхпроводящСго ΠΏΡ€ΠΎΠ²ΠΎΠ΄Π°. Доля Ρ‚ΠΎΠΊΠ°, ΠΏΡ€ΠΎΡ‚Π΅ΠΊΠ°ΡŽΡ‰Π΅Π³ΠΎ Ρ‡Π΅Ρ€Π΅Π· мСдная ΠΌΠ°Ρ‚Ρ€ΠΈΡ†Π° опрСдСляСтся Π»Π΅Π³ΠΊΠΎ.ΠŸΡ€Π΅Π΄ΠΏΠΎΠ»ΠΎΠΆΠΈΠΌ, Ρ‡Ρ‚ΠΎ ΠΏΠΎΡ‚Π΅Π½Ρ†ΠΈΠ°Π» Ρ€Π°Π·Π½ΠΎΡΡ‚ΡŒ ΠΏΠΎ ΠΏΡ€ΠΎΠ²ΠΎΠ΄Π½ΠΈΠΊΡƒ Ρ€Π°Π²Π½Π° [Delta] V. ЭлСктричСский Ρ‚ΠΎΠΊ I Cu , ΠΏΡ€ΠΎΡ‚Π΅ΠΊΠ°ΡŽΡ‰Π°Ρ Ρ‡Π΅Ρ€Π΅Π· ΠΌΠ΅Π΄Π½ΡƒΡŽ ΠΌΠ°Ρ‚Ρ€ΠΈΡ†Ρƒ, Ρ€Π°Π²Π½Π°

    (28,33)

    Π’ΠΎΠΊ I fil , ΠΏΡ€ΠΎΡ‚Π΅ΠΊΠ°ΡŽΡ‰ΠΈΠΉ Ρ‡Π΅Ρ€Π΅Π· 2100 Π½ΠΈΡ‚Π΅ΠΉ, Ρ€Π°Π²Π΅Π½

    . (28,34)

    Доля F ΠΏΠΎΠ»Π½ΠΎΠ³ΠΎ Ρ‚ΠΎΠΊΠ°, ΠΏΡ€ΠΎΡ‚Π΅ΠΊΠ°ΡŽΡ‰Π΅Π³ΠΎ Ρ‡Π΅Ρ€Π΅Π· ΠΌΠ΅Π΄Π½ΡƒΡŽ ΠΌΠ°Ρ‚Ρ€ΠΈΡ†Ρƒ, Ρ€Π°Π²Π½Π° Π½Π° Π½ΠΎΠΌΠ΅Ρ€

    (28.35)

    НСобходимо Ρ€Π°ΡΡΠΌΠΎΡ‚Ρ€Π΅Ρ‚ΡŒ Π΄Π²Π° особых случая.

    1. Π’Π΅ΠΌΠΏΠ΅Ρ€Π°Ρ‚ΡƒΡ€Π° Π½ΠΈΠΆΠ΅ критичСской. На ΡƒΡ€ΠΎΠ²Π½Π΅ ΠΈΠ»ΠΈ Π½ΠΈΠΆΠ΅ этого Ρ‚Π΅ΠΌΠΏΠ΅Ρ€Π°Ρ‚ΡƒΡ€Π° сопротивлСниС Π½ΠΈΡ‚Π΅ΠΉ исчСзаСт (R fil = 0 [ОмСга]). Π£Ρ€Π°Π²Π½Π΅Π½ΠΈΠ΅ (28.35) ΠΏΠΎΠΊΠ°Π·Ρ‹Π²Π°Π΅Ρ‚, Ρ‡Ρ‚ΠΎ Π² этом случаС Ρ‚ΠΎΠΊ Π½Π΅ Π±ΡƒΠ΄Π΅Ρ‚ Ρ‚Π΅Ρ‡ΡŒ. Ρ‡Π΅Ρ€Π΅Π· ΠΌΠ΅Π΄Π½ΡƒΡŽ ΠΌΠ°Ρ‚Ρ€ΠΈΡ†Ρƒ.

    2. Если Ρ‚Π΅ΠΌΠΏΠ΅Ρ€Π°Ρ‚ΡƒΡ€Π° ΠΏΡ€ΠΎΠ²ΠΎΠ΄Π° Π²Ρ‹ΡˆΠ΅ критичСской Ρ‚Π΅ΠΌΠΏΠ΅Ρ€Π°Ρ‚ΡƒΡ€Ρ‹, Ρ‚Π΅ΠΊΡƒΡ‰ΠΈΠΉ ΠΏΠΎΡ‚ΠΎΠΊ Ρ€Π΅Π·ΠΊΠΎ измСнится. Π’ этом случаС доля Ρ‚ΠΎΠΊ, ΠΏΡ€ΠΎΡ‚Π΅ΠΊΠ°ΡŽΡ‰ΠΈΠΉ Ρ‡Π΅Ρ€Π΅Π· мСдь, Ρ€Π°Π²Π΅Π½

    (28.36)

    МСдная ΠΌΠ°Ρ‚Ρ€ΠΈΡ†Π° Π±ΡƒΠ΄Π΅Ρ‚ ΠΏΡ€ΠΎΠΏΡƒΡΠΊΠ°Ρ‚ΡŒ 90% ΠΎΠ±Ρ‰Π΅Π³ΠΎ Ρ‚ΠΎΠΊΠ°.

    ΠŸΡ€ΠΈΠΌΠ΅Ρ€: Π·Π°Π΄Π°Ρ‡Π° 28.42

    КакоС сопротивлСниС Ρƒ ΠΊΠΎΠΌΠ±ΠΈΠ½Π°Ρ†ΠΈΠΈ ΠΈΠ· Ρ‡Π΅Ρ‚Ρ‹Ρ€Π΅Ρ… рСзисторов, ΠΏΠΎΠΊΠ°Π·Π°Π½Π½ΠΎΠΉ Π½Π° Рисунок 28.7. ΠšΠ°ΠΆΠ΄Ρ‹ΠΉ ΠΈΠ· рСзисторов ΠΈΠΌΠ΅Π΅Ρ‚ Π½ΠΎΠΌΠΈΠ½Π°Π» R.

    Рисунок 28.7. ΠŸΡ€ΠΎΠ±Π»Π΅ΠΌΠ° 28.42.

    Π§Ρ‚ΠΎΠ±Ρ‹ Π½Π°ΠΉΡ‚ΠΈ сопротивлСниС Ρ†Π΅ΠΏΠΈ, ΠΏΠΎΠΊΠ°Π·Π°Π½Π½ΠΎΠΉ Π½Π° рисункС 28.7, ΠΌΡ‹ Π½Π°Ρ‡Π½Π΅ΠΌ расчСт чистого сопротивлСния R 34 ΠΏΠ°Ρ€Π°Π»Π»Π΅Π»ΡŒΠ½ΠΎΠΉ Ρ†Π΅ΠΏΠΈ рСзисторы R 3 ΠΈ R 4 :

    (28.37)

    ΠΈΠ»ΠΈ

    (28,38)

    Π’Π°ΠΊΠΈΠΌ ΠΎΠ±Ρ€Π°Π·ΠΎΠΌ, схСма, показанная Π½Π° рисункС 28.7, эквивалСнтна ΠΏΠΎΠΊΠ°Π·Π°Π½Π½ΠΎΠΉ схСмС. Π½Π° рисункС 28.8. РСзисторы R 2 ΠΈ R 34 ΠΎΠ±Ρ€Π°Π·ΡƒΡŽΡ‚ ΡΠ΅Ρ€ΠΈΡŽ сСти ΠΈ ΠΌΠΎΠΆΠ΅Ρ‚ Π±Ρ‹Ρ‚ΡŒ Π·Π°ΠΌΠ΅Π½Π΅Π½ ΠΎΠ΄ΠΈΠ½ΠΎΡ‡Π½Ρ‹ΠΌ рСзистором с сопротивлСниСм R 234 Π³Π΄Π΅

    (28.39)

    Рисунок 28.8. ΠŸΡ€ΠΎΠ±Π»Π΅ΠΌΠ° 28.42. Рисунок 28.9. ΠŸΡ€ΠΎΠ±Π»Π΅ΠΌΠ° 28.42. Π‘Ρ…Π΅ΠΌΠ° ΠΏΠΎΠΊΠ°Π·Π°Π½Π° Π½Π° рисункС 28.8 Ρ‚Π΅ΠΏΠ΅Ρ€ΡŒ ΠΌΠΎΠΆΠ½ΠΎ Π·Π°ΠΌΠ΅Π½ΠΈΡ‚ΡŒ Π½Π° ЭквивалСнтная схСма ΠΏΠΎΠΊΠ°Π·Π°Π½Π° Π½Π° рисункС 28.9. Π‘ΠΎΠΏΡ€ΠΎΡ‚ΠΈΠ²Π»Π΅Π½ΠΈΠ΅ R Π΄ΠΎ ΠΎΡ‚ эта схСма ΠΌΠΎΠΆΠ΅Ρ‚ Π±Ρ‹Ρ‚ΡŒ ΠΏΠΎΠ»ΡƒΡ‡Π΅Π½Π° ΠΈΠ· ΡΠ»Π΅Π΄ΡƒΡŽΡ‰Π΅Π³ΠΎ ΡΠΎΠΎΡ‚Π½ΠΎΡˆΠ΅Π½ΠΈΡ

    (28,40)

    ΠΈΠ»ΠΈ

    (28.41)

    Π’ рассматриваСмом частном случаС R 1 = R 2 = R 3 = R 4 = R. Π’Π°ΠΊΠΈΠΌ ΠΎΠ±Ρ€Π°Π·ΠΎΠΌ,

    (28,42)

    (28.43)

    (28,44)

    Для R = 3 [ОмСга] ΠΏΠΎΠ»Π½ΠΎΠ΅ сопротивлСниС Ρ€Π°Π²Π½ΠΎ 1,8 [ОмСга].


    ΠžΡ‚ΠΏΡ€Π°Π²Π»ΡΠΉΡ‚Π΅ ΠΊΠΎΠΌΠΌΠ΅Π½Ρ‚Π°Ρ€ΠΈΠΈ, вопросы ΠΈ / ΠΈΠ»ΠΈ прСдлоТСния ΠΏΠΎ элСктронной ΠΏΠΎΡ‡Ρ‚Π΅ Π½Π° адрСс [email protected] ΠΈ / ΠΈΠ»ΠΈ посСтитС домашнюю страницу Ѐрэнка Π’ΠΎΠ»ΡŒΡ„ΡΠ°.

    Π‘ΠΎΠΏΡ€ΠΎΡ‚ΠΈΠ²Π»Π΅Π½ΠΈΠ΅ ΠΏΡ€ΠΎΠ²ΠΎΠ΄Π½ΠΈΠΊΠ°

    Π₯отя ΠΌΠΎΠΆΠ½ΠΎ ΠΈΡΠΏΠΎΠ»ΡŒΠ·ΠΎΠ²Π°Ρ‚ΡŒ ΠΏΡ€ΠΎΠ²ΠΎΠ΄ любого Ρ€Π°Π·ΠΌΠ΅Ρ€Π° ΠΈΠ»ΠΈ значСния сопротивлСния, слово Β«ΠΏΡ€ΠΎΠ²ΠΎΠ΄Π½ΠΈΠΊΒ» ΠΎΠ±Ρ‹Ρ‡Π½ΠΎ относится ΠΊ ΠΌΠ°Ρ‚Π΅Ρ€ΠΈΠ°Π»Π°ΠΌ, ΠΊΠΎΡ‚ΠΎΡ€Ρ‹Π΅ ΠΎΠ±Π»Π°Π΄Π°ΡŽΡ‚ Π½ΠΈΠ·ΠΊΠΈΠΌ сопротивлСниСм Ρ‚ΠΎΠΊΡƒ, Π° слово «изолятор» описываСт ΠΌΠ°Ρ‚Π΅Ρ€ΠΈΠ°Π»Ρ‹, ΠΊΠΎΡ‚ΠΎΡ€Ρ‹Π΅ ΠΎΠ±Π»Π°Π΄Π°ΡŽΡ‚ высоким сопротивлСниСм Ρ‚ΠΎΠΊΡƒ. .ΠœΠ΅ΠΆΠ΄Ρƒ ΠΏΡ€ΠΎΠ²ΠΎΠ΄Π½ΠΈΠΊΠ°ΠΌΠΈ ΠΈ изоляторами Π½Π΅Ρ‚ Ρ‡Π΅Ρ‚ΠΊΠΎΠΉ Ρ€Π°Π·Π΄Π΅Π»ΠΈΡ‚Π΅Π»ΡŒΠ½ΠΎΠΉ Π»ΠΈΠ½ΠΈΠΈ; ΠΏΡ€ΠΈ ΠΎΠΏΡ€Π΅Π΄Π΅Π»Π΅Π½Π½Ρ‹Ρ… условиях всС Ρ‚ΠΈΠΏΡ‹ ΠΌΠ°Ρ‚Π΅Ρ€ΠΈΠ°Π»ΠΎΠ² проводят Ρ‚ΠΎΠΊ. ΠœΠ°Ρ‚Π΅Ρ€ΠΈΠ°Π»Ρ‹, ΠΎΠ±Π΅ΡΠΏΠ΅Ρ‡ΠΈΠ²Π°ΡŽΡ‰ΠΈΠ΅ сопротивлСниС Ρ‚ΠΎΠΊΡƒ Π½Π° ΠΏΠΎΠ»ΠΏΡƒΡ‚ΠΈ ΠΌΠ΅ΠΆΠ΄Ρƒ Π»ΡƒΡ‡ΡˆΠΈΠΌΠΈ ΠΏΡ€ΠΎΠ²ΠΎΠ΄Π½ΠΈΠΊΠ°ΠΌΠΈ ΠΈ самыми ΠΏΠ»ΠΎΡ…ΠΈΠΌΠΈ ΠΏΡ€ΠΎΠ²ΠΎΠ΄Π½ΠΈΠΊΠ°ΠΌΠΈ (изоляторами), ΠΈΠ½ΠΎΠ³Π΄Π° Π½Π°Π·Ρ‹Π²Π°ΡŽΡ‚ Β«ΠΏΠΎΠ»ΡƒΠΏΡ€ΠΎΠ²ΠΎΠ΄Π½ΠΈΠΊΠ°ΠΌΠΈΒ» ΠΈ находят наибольшСС ΠΏΡ€ΠΈΠΌΠ΅Π½Π΅Π½ΠΈΠ΅ Π² области транзисторов.

    Π›ΡƒΡ‡ΡˆΠΈΠ΅ ΠΏΡ€ΠΎΠ²ΠΎΠ΄Π½ΠΈΠΊΠΈ — это ΠΌΠ°Ρ‚Π΅Ρ€ΠΈΠ°Π»Ρ‹, Π² основном ΠΌΠ΅Ρ‚Π°Π»Π»Ρ‹, ΠΊΠΎΡ‚ΠΎΡ€Ρ‹Π΅ ΠΎΠ±Π»Π°Π΄Π°ΡŽΡ‚ большим количСством свободных элСктронов; И Π½Π°ΠΎΠ±ΠΎΡ€ΠΎΡ‚, изоляторы — это ΠΌΠ°Ρ‚Π΅Ρ€ΠΈΠ°Π»Ρ‹ с нСбольшим количСством свободных элСктронов.Π›ΡƒΡ‡ΡˆΠΈΠ΅ ΠΏΡ€ΠΎΠ²ΠΎΠ΄Π½ΠΈΠΊΠΈ — сСрСбро, мСдь, Π·ΠΎΠ»ΠΎΡ‚ΠΎ ΠΈ алюминий; Π½ΠΎ Π½Π΅ΠΊΠΎΡ‚ΠΎΡ€Ρ‹Π΅ Π½Π΅ΠΌΠ΅Ρ‚Π°Π»Π»Ρ‹, Ρ‚Π°ΠΊΠΈΠ΅ ΠΊΠ°ΠΊ ΡƒΠ³Π»Π΅Ρ€ΠΎΠ΄ ΠΈ Π²ΠΎΠ΄Π°, ΠΌΠΎΠ³ΡƒΡ‚ ΠΈΡΠΏΠΎΠ»ΡŒΠ·ΠΎΠ²Π°Ρ‚ΡŒΡΡ Π² качСствС ΠΏΡ€ΠΎΠ²ΠΎΠ΄Π½ΠΈΠΊΠΎΠ². Π’Π°ΠΊΠΈΠ΅ ΠΌΠ°Ρ‚Π΅Ρ€ΠΈΠ°Π»Ρ‹, ΠΊΠ°ΠΊ Ρ€Π΅Π·ΠΈΠ½Π°, стСкло, ΠΊΠ΅Ρ€Π°ΠΌΠΈΠΊΠ° ΠΈ пластмассы, ΡΠ²Π»ΡΡŽΡ‚ΡΡ Π½Π°ΡΡ‚ΠΎΠ»ΡŒΠΊΠΎ ΠΏΠ»ΠΎΡ…ΠΈΠΌΠΈ ΠΏΡ€ΠΎΠ²ΠΎΠ΄Π½ΠΈΠΊΠ°ΠΌΠΈ, Ρ‡Ρ‚ΠΎ ΠΈΡ… ΠΎΠ±Ρ‹Ρ‡Π½ΠΎ ΠΈΡΠΏΠΎΠ»ΡŒΠ·ΡƒΡŽΡ‚ Π² качСствС изоляторов. Π’ΠΎΠΊ Π² Π½Π΅ΠΊΠΎΡ‚ΠΎΡ€Ρ‹Ρ… ΠΈΠ· этих ΠΌΠ°Ρ‚Π΅Ρ€ΠΈΠ°Π»ΠΎΠ² Π½Π°ΡΡ‚ΠΎΠ»ΡŒΠΊΠΎ ΠΌΠ°Π», Ρ‡Ρ‚ΠΎ ΠΎΠ±Ρ‹Ρ‡Π½ΠΎ считаСтся Π½ΡƒΠ»Π΅Π²Ρ‹ΠΌ. Π•Π΄ΠΈΠ½ΠΈΡ†Π° измСрСния сопротивлСния называСтся ΠΎΠΌ. Π‘ΠΈΠΌΠ²ΠΎΠ» ΠΎΠΌΠ° — грСчСская Π±ΡƒΠΊΠ²Π° ΠΎΠΌΠ΅Π³Π° (Ξ©). Π’ матСматичСских Ρ„ΠΎΡ€ΠΌΡƒΠ»Π°Ρ… заглавная Π±ΡƒΠΊΠ²Π° Β«RΒ» ΠΎΠ±ΠΎΠ·Π½Π°Ρ‡Π°Π΅Ρ‚ сопротивлСниС.Π‘ΠΎΠΏΡ€ΠΎΡ‚ΠΈΠ²Π»Π΅Π½ΠΈΠ΅ ΠΏΡ€ΠΎΠ²ΠΎΠ΄Π½ΠΈΠΊΠ° ΠΈ ΠΏΡ€ΠΈΠ»ΠΎΠΆΠ΅Π½Π½ΠΎΠ΅ ΠΊ Π½Π΅ΠΌΡƒ напряТСниС ΠΎΠΏΡ€Π΅Π΄Π΅Π»ΡΡŽΡ‚ количСство Π°ΠΌΠΏΠ΅Ρ€ Ρ‚ΠΎΠΊΠ°, ΠΏΡ€ΠΎΡ‚Π΅ΠΊΠ°ΡŽΡ‰Π΅Π³ΠΎ ΠΏΠΎ ΠΏΡ€ΠΎΠ²ΠΎΠ΄Π½ΠΈΠΊΡƒ. Π’Π°ΠΊΠΈΠΌ ΠΎΠ±Ρ€Π°Π·ΠΎΠΌ, сопротивлСниС 1 Ом ΠΎΠ³Ρ€Π°Π½ΠΈΡ‡ΠΈΠ²Π°Π΅Ρ‚ Ρ‚ΠΎΠΊ Π΄ΠΎ 1 Π°ΠΌΠΏΠ΅Ρ€Π° Π² ΠΏΡ€ΠΎΠ²ΠΎΠ΄Π½ΠΈΠΊΠ΅, ΠΊ ΠΊΠΎΡ‚ΠΎΡ€ΠΎΠΌΡƒ ΠΏΡ€ΠΈΠ»ΠΎΠΆΠ΅Π½ΠΎ напряТСниС 1 Π²ΠΎΠ»ΡŒΡ‚.

    Π€Π°ΠΊΡ‚ΠΎΡ€Ρ‹, Π²Π»ΠΈΡΡŽΡ‰ΠΈΠ΅ Π½Π° сопротивлСниС

    1. Π‘ΠΎΠΏΡ€ΠΎΡ‚ΠΈΠ²Π»Π΅Π½ΠΈΠ΅ мСталличСского ΠΏΡ€ΠΎΠ²ΠΎΠ΄Π½ΠΈΠΊΠ° зависит ΠΎΡ‚ Ρ‚ΠΈΠΏΠ° ΠΌΠ°Ρ‚Π΅Ρ€ΠΈΠ°Π»Π° ΠΏΡ€ΠΎΠ²ΠΎΠ΄Π½ΠΈΠΊΠ°. Π‘Ρ‹Π»ΠΎ ΡƒΠΊΠ°Π·Π°Π½ΠΎ, Ρ‡Ρ‚ΠΎ Π½Π΅ΠΊΠΎΡ‚ΠΎΡ€Ρ‹Π΅ ΠΌΠ΅Ρ‚Π°Π»Π»Ρ‹ ΠΎΠ±Ρ‹Ρ‡Π½ΠΎ ΠΈΡΠΏΠΎΠ»ΡŒΠ·ΡƒΡŽΡ‚ΡΡ Π² качСствС ΠΏΡ€ΠΎΠ²ΠΎΠ΄Π½ΠΈΠΊΠΎΠ² ΠΈΠ·-Π·Π° большого количСства свободных элСктронов Π½Π° ΠΈΡ… Π²Π½Π΅ΡˆΠ½ΠΈΡ… ΠΎΡ€Π±ΠΈΡ‚Π°Ρ….МСдь ΠΎΠ±Ρ‹Ρ‡Π½ΠΎ считаСтся Π»ΡƒΡ‡ΡˆΠΈΠΌ доступным ΠΌΠ°Ρ‚Π΅Ρ€ΠΈΠ°Π»ΠΎΠΌ для ΠΏΡ€ΠΎΠ²ΠΎΠ΄Π½ΠΈΠΊΠΎΠ², ΠΏΠΎΡΠΊΠΎΠ»ΡŒΠΊΡƒ мСдная ΠΏΡ€ΠΎΠ²ΠΎΠ»ΠΎΠΊΠ° ΠΎΠΏΡ€Π΅Π΄Π΅Π»Π΅Π½Π½ΠΎΠ³ΠΎ Π΄ΠΈΠ°ΠΌΠ΅Ρ‚Ρ€Π° обСспСчиваСт мСньшСС сопротивлСниС Ρ‚ΠΎΠΊΡƒ, Ρ‡Π΅ΠΌ алюминиСвая ΠΏΡ€ΠΎΠ²ΠΎΠ»ΠΎΠΊΠ° Ρ‚ΠΎΠ³ΠΎ ΠΆΠ΅ Π΄ΠΈΠ°ΠΌΠ΅Ρ‚Ρ€Π°. Однако алюминий Π½Π°ΠΌΠ½ΠΎΠ³ΠΎ Π»Π΅Π³Ρ‡Π΅ ΠΌΠ΅Π΄ΠΈ, ΠΈ ΠΏΠΎ этой ΠΏΡ€ΠΈΡ‡ΠΈΠ½Π΅, Π° Ρ‚Π°ΠΊΠΆΠ΅ ΠΏΠΎ сообраТСниям стоимости, алюминий часто ΠΈΡΠΏΠΎΠ»ΡŒΠ·ΡƒΠ΅Ρ‚ΡΡ, ΠΊΠΎΠ³Π΄Π° Π²Π°ΠΆΠ΅Π½ вСсовой коэффициСнт.
    2. Π‘ΠΎΠΏΡ€ΠΎΡ‚ΠΈΠ²Π»Π΅Π½ΠΈΠ΅ мСталличСского ΠΏΡ€ΠΎΠ²ΠΎΠ΄Π½ΠΈΠΊΠ° прямо ΠΏΡ€ΠΎΠΏΠΎΡ€Ρ†ΠΈΠΎΠ½Π°Π»ΡŒΠ½ΠΎ Π΅Π³ΠΎ Π΄Π»ΠΈΠ½Π΅. Π§Π΅ΠΌ большС Π΄Π»ΠΈΠ½Π° ΠΏΡ€ΠΎΠ²ΠΎΠ΄Π° Π΄Π°Π½Π½ΠΎΠ³ΠΎ сСчСния, Ρ‚Π΅ΠΌ большС сопротивлСниС.На рисункС 12-41 ΠΏΠΎΠΊΠ°Π·Π°Π½Ρ‹ Π΄Π²Π° ΠΏΡ€ΠΎΠ²ΠΎΠ΄Π½ΠΈΠΊΠ° Ρ€Π°Π·Π½ΠΎΠΉ Π΄Π»ΠΈΠ½Ρ‹. Если элСктричСскоС Π΄Π°Π²Π»Π΅Π½ΠΈΠ΅ 1 Π²ΠΎΠ»ΡŒΡ‚ ΠΏΡ€ΠΈΠ»ΠΎΠΆΠ΅Π½ΠΎ ΠΊ Π΄Π²ΡƒΠΌ ΠΊΠΎΠ½Ρ†Π°ΠΌ ΠΏΡ€ΠΎΠ²ΠΎΠ΄Π½ΠΈΠΊΠ° Π΄Π»ΠΈΠ½ΠΎΠΉ 1 Ρ„ΡƒΡ‚, Π° сопротивлСниС двиТСнию свободных элСктронов прСдполагаСтся Ρ€Π°Π²Π½Ρ‹ΠΌ 1 Ом, Ρ‚ΠΎΠΊ ограничиваСтся 1 Π°ΠΌΠΏΠ΅Ρ€. Если ΠΏΡ€ΠΎΠ²ΠΎΠ΄ Ρ‚ΠΎΠ³ΠΎ ΠΆΠ΅ Ρ€Π°Π·ΠΌΠ΅Ρ€Π° ΡƒΠ΄Π²ΠΎΠΈΡ‚ΡŒ Π² Π΄Π»ΠΈΠ½Ρƒ, Ρ‚Π΅ ΠΆΠ΅ элСктроны, ΠΏΡ€ΠΈΠ²Π΅Π΄Π΅Π½Π½Ρ‹Π΅ Π² Π΄Π²ΠΈΠΆΠ΅Π½ΠΈΠ΅ ΠΏΠΎΠ΄ дСйствиСм ΠΏΡ€ΠΈΠ»ΠΎΠΆΠ΅Π½Π½ΠΎΠ³ΠΎ 1 Π²ΠΎΠ»ΡŒΡ‚Π°, Ρ‚Π΅ΠΏΠ΅Ρ€ΡŒ ΠΎΠ±Π½Π°Ρ€ΡƒΠΆΠΈΠ²Π°ΡŽΡ‚ ΡƒΠ΄Π²ΠΎΠ΅Π½Π½ΠΎΠ΅ сопротивлСниС; ΡΠ»Π΅Π΄ΠΎΠ²Π°Ρ‚Π΅Π»ΡŒΠ½ΠΎ, Ρ‚ΠΎΠΊ ΡƒΠΌΠ΅Π½ΡŒΡˆΠ°Π΅Ρ‚ΡΡ Π²Π΄Π²ΠΎΠ΅. Рисунок 12-41. Π‘ΠΎΠΏΡ€ΠΎΡ‚ΠΈΠ²Π»Π΅Π½ΠΈΠ΅ зависит ΠΎΡ‚ Π΄Π»ΠΈΠ½Ρ‹ ΠΏΡ€ΠΎΠ²ΠΎΠ΄Π½ΠΈΠΊΠ°.
    3. Π‘ΠΎΠΏΡ€ΠΎΡ‚ΠΈΠ²Π»Π΅Π½ΠΈΠ΅ мСталличСского ΠΏΡ€ΠΎΠ²ΠΎΠ΄Π½ΠΈΠΊΠ° ΠΎΠ±Ρ€Π°Ρ‚Π½ΠΎ ΠΏΡ€ΠΎΠΏΠΎΡ€Ρ†ΠΈΠΎΠ½Π°Π»ΡŒΠ½ΠΎ ΠΏΠ»ΠΎΡ‰Π°Π΄ΠΈ ΠΏΠΎΠΏΠ΅Ρ€Π΅Ρ‡Π½ΠΎΠ³ΠΎ сСчСния. Π­Ρ‚Π° ΠΎΠ±Π»Π°ΡΡ‚ΡŒ ΠΌΠΎΠΆΠ΅Ρ‚ Π±Ρ‹Ρ‚ΡŒ Ρ‚Ρ€Π΅ΡƒΠ³ΠΎΠ»ΡŒΠ½ΠΎΠΉ ΠΈΠ»ΠΈ Π΄Π°ΠΆΠ΅ ΠΊΠ²Π°Π΄Ρ€Π°Ρ‚Π½ΠΎΠΉ, Π½ΠΎ ΠΎΠ±Ρ‹Ρ‡Π½ΠΎ ΠΊΡ€ΡƒΠ³Π»ΠΎΠΉ. Если ΠΏΠ»ΠΎΡ‰Π°Π΄ΡŒ ΠΏΠΎΠΏΠ΅Ρ€Π΅Ρ‡Π½ΠΎΠ³ΠΎ сСчСния ΠΏΡ€ΠΎΠ²ΠΎΠ΄Π½ΠΈΠΊΠ° увСличиваСтся Π²Π΄Π²ΠΎΠ΅, сопротивлСниС Ρ‚ΠΎΠΊΡƒ ΡƒΠΌΠ΅Π½ΡŒΡˆΠ°Π΅Ρ‚ΡΡ Π²Π΄Π²ΠΎΠ΅. Π­Ρ‚ΠΎ Π²Π΅Ρ€Π½ΠΎ ΠΈΠ·-Π·Π° ΡƒΠ²Π΅Π»ΠΈΡ‡Π΅Π½Π½ΠΎΠΉ ΠΏΠ»ΠΎΡ‰Π°Π΄ΠΈ, Π² ΠΊΠΎΡ‚ΠΎΡ€ΠΎΠΉ элСктрон ΠΌΠΎΠΆΠ΅Ρ‚ ΠΏΠ΅Ρ€Π΅ΠΌΠ΅Ρ‰Π°Ρ‚ΡŒΡΡ Π±Π΅Π· столкновСния ΠΈΠ»ΠΈ Π·Π°Ρ…Π²Π°Ρ‚Π° Π°Ρ‚ΠΎΠΌΠΎΠΌ. Π’Π°ΠΊΠΈΠΌ ΠΎΠ±Ρ€Π°Π·ΠΎΠΌ, сопротивлСниС измСняСтся ΠΎΠ±Ρ€Π°Ρ‚Π½ΠΎ ΠΏΡ€ΠΎΠΏΠΎΡ€Ρ†ΠΈΠΎΠ½Π°Π»ΡŒΠ½ΠΎ ΠΏΠ»ΠΎΡ‰Π°Π΄ΠΈ ΠΏΠΎΠΏΠ΅Ρ€Π΅Ρ‡Π½ΠΎΠ³ΠΎ сСчСния ΠΏΡ€ΠΎΠ²ΠΎΠ΄Π½ΠΈΠΊΠ°.
    4. Π§Π΅Ρ‚Π²Π΅Ρ€Ρ‚Ρ‹ΠΌ Π²Π°ΠΆΠ½Ρ‹ΠΌ Ρ„Π°ΠΊΡ‚ΠΎΡ€ΠΎΠΌ, Π²Π»ΠΈΡΡŽΡ‰ΠΈΠΌ Π½Π° сопротивлСниС ΠΏΡ€ΠΎΠ²ΠΎΠ΄Π½ΠΈΠΊΠ°, являСтся Ρ‚Π΅ΠΌΠΏΠ΅Ρ€Π°Ρ‚ΡƒΡ€Π°.Π₯отя Π½Π΅ΠΊΠΎΡ‚ΠΎΡ€Ρ‹Π΅ вСщСства, Ρ‚Π°ΠΊΠΈΠ΅ ΠΊΠ°ΠΊ ΡƒΠ³Π»Π΅Ρ€ΠΎΠ΄, Π΄Π΅ΠΌΠΎΠ½ΡΡ‚Ρ€ΠΈΡ€ΡƒΡŽΡ‚ сниТСниС сопротивлСния ΠΏΡ€ΠΈ ΠΏΠΎΠ²Ρ‹ΡˆΠ΅Π½ΠΈΠΈ Ρ‚Π΅ΠΌΠΏΠ΅Ρ€Π°Ρ‚ΡƒΡ€Ρ‹ ΠΎΠΊΡ€ΡƒΠΆΠ°ΡŽΡ‰Π΅ΠΉ срСды, Π±ΠΎΠ»ΡŒΡˆΠΈΠ½ΡΡ‚Π²ΠΎ ΠΌΠ°Ρ‚Π΅Ρ€ΠΈΠ°Π»ΠΎΠ², ΠΈΡΠΏΠΎΠ»ΡŒΠ·ΡƒΠ΅ΠΌΡ‹Ρ… Π² качСствС ΠΏΡ€ΠΎΠ²ΠΎΠ΄Π½ΠΈΠΊΠΎΠ², ΡƒΠ²Π΅Π»ΠΈΡ‡ΠΈΠ²Π°ΡŽΡ‚ сопротивлСниС ΠΏΡ€ΠΈ ΠΏΠΎΠ²Ρ‹ΡˆΠ΅Π½ΠΈΠΈ Ρ‚Π΅ΠΌΠΏΠ΅Ρ€Π°Ρ‚ΡƒΡ€Ρ‹. Π‘ΠΎΠΏΡ€ΠΎΡ‚ΠΈΠ²Π»Π΅Π½ΠΈΠ΅ Π½Π΅ΠΊΠΎΡ‚ΠΎΡ€Ρ‹Ρ… сплавов, Ρ‚Π°ΠΊΠΈΡ… ΠΊΠ°ΠΊ константан ΠΈ ΠΌΠ°Π½Π³Π°Π½ΠΈΠ½ β„’, ΠΎΡ‡Π΅Π½ΡŒ ΠΌΠ°Π»ΠΎ измСняСтся ΠΏΡ€ΠΈ ΠΈΠ·ΠΌΠ΅Π½Π΅Π½ΠΈΠΈ Ρ‚Π΅ΠΌΠΏΠ΅Ρ€Π°Ρ‚ΡƒΡ€Ρ‹. Π’Π΅Π»ΠΈΡ‡ΠΈΠ½Π° увСличСния сопротивлСния ΠΎΠ±Ρ€Π°Π·Ρ†Π° ΠΏΡ€ΠΎΠ²ΠΎΠ΄Π½ΠΈΠΊΠ° с сопротивлСниСм 1 Ом Π½Π° ΠΎΠ΄ΠΈΠ½ градус ΠΏΠΎΠ²Ρ‹ΡˆΠ΅Π½ΠΈΡ Ρ‚Π΅ΠΌΠΏΠ΅Ρ€Π°Ρ‚ΡƒΡ€Ρ‹ Π²Ρ‹ΡˆΠ΅ 0 Β° ΠΏΠΎ ЦСльсию (C), принятого стандарта, называСтся Ρ‚Π΅ΠΌΠΏΠ΅Ρ€Π°Ρ‚ΡƒΡ€Π½Ρ‹ΠΌ коэффициСнтом сопротивлСния.Для ΠΊΠ°ΠΆΠ΄ΠΎΠ³ΠΎ ΠΌΠ΅Ρ‚Π°Π»Π»Π° это Ρ€Π°Π·Π½Ρ‹Π΅ значСния. НапримСр, для ΠΌΠ΅Π΄ΠΈ это Π·Π½Π°Ρ‡Π΅Π½ΠΈΠ΅ составляСт ΠΏΡ€ΠΈΠΌΠ΅Ρ€Π½ΠΎ 0,00427 Ом. Π’Π°ΠΊΠΈΠΌ ΠΎΠ±Ρ€Π°Π·ΠΎΠΌ, ΠΌΠ΅Π΄Π½Ρ‹ΠΉ ΠΏΡ€ΠΎΠ²ΠΎΠ΄, ΠΈΠΌΠ΅ΡŽΡ‰ΠΈΠΉ сопротивлСниС 50 Ом ΠΏΡ€ΠΈ Ρ‚Π΅ΠΌΠΏΠ΅Ρ€Π°Ρ‚ΡƒΡ€Π΅ 0 Β° C, ΠΈΠΌΠ΅Π΅Ρ‚ ΡƒΠ²Π΅Π»ΠΈΡ‡Π΅Π½ΠΈΠ΅ сопротивлСния Π½Π° 50 Γ— 0,00427 ΠΈΠ»ΠΈ 0,214 Ом Π½Π° ΠΊΠ°ΠΆΠ΄Ρ‹ΠΉ градус ΠΏΠΎΠ²Ρ‹ΡˆΠ΅Π½ΠΈΡ Ρ‚Π΅ΠΌΠΏΠ΅Ρ€Π°Ρ‚ΡƒΡ€Ρ‹ Π²Ρ‹ΡˆΠ΅ 0 Β° C. Π’Π΅ΠΌΠΏΠ΅Ρ€Π°Ρ‚ΡƒΡ€Π½Ρ‹ΠΉ коэффициСнт сопротивлСния Π½Π΅ΠΎΠ±Ρ…ΠΎΠ΄ΠΈΠΌΠΎ ΡƒΡ‡ΠΈΡ‚Ρ‹Π²Π°Ρ‚ΡŒ Ρ‚Π°ΠΌ, Π³Π΄Π΅ Π½Π°Π±Π»ΡŽΠ΄Π°Π΅Ρ‚ΡΡ Π·Π°ΠΌΠ΅Ρ‚Π½ΠΎΠ΅ ΠΈΠ·ΠΌΠ΅Π½Π΅Π½ΠΈΠ΅ Ρ‚Π΅ΠΌΠΏΠ΅Ρ€Π°Ρ‚ΡƒΡ€Ρ‹ ΠΏΡ€ΠΎΠ²ΠΎΠ΄Π½ΠΈΠΊΠ° Π²ΠΎ врСмя Ρ€Π°Π±ΠΎΡ‚Ρ‹. Доступны Π³Ρ€Π°Ρ„ΠΈΠΊΠΈ с ΡƒΠΊΠ°Π·Π°Π½ΠΈΠ΅ΠΌ Ρ‚Π΅ΠΌΠΏΠ΅Ρ€Π°Ρ‚ΡƒΡ€Π½ΠΎΠ³ΠΎ коэффициСнта сопротивлСния для Ρ€Π°Π·Π»ΠΈΡ‡Π½Ρ‹Ρ… ΠΌΠ°Ρ‚Π΅Ρ€ΠΈΠ°Π»ΠΎΠ².На Рис. 12-42 ΠΏΠΎΠΊΠ°Π·Π°Π½Π° Ρ‚Π°Π±Π»ΠΈΡ†Π° Β«ΡƒΠ΄Π΅Π»ΡŒΠ½ΠΎΠ³ΠΎ сопротивлСния» Π½Π΅ΠΊΠΎΡ‚ΠΎΡ€Ρ‹Ρ… распространСнных элСктричСских ΠΏΡ€ΠΎΠ²ΠΎΠ΄Π½ΠΈΠΊΠΎΠ².
    Рисунок 12-42. Π’Π°Π±Π»ΠΈΡ†Π° ΡƒΠ΄Π΅Π»ΡŒΠ½ΠΎΠ³ΠΎ сопротивлСния.

    Π‘ΠΎΠΏΡ€ΠΎΡ‚ΠΈΠ²Π»Π΅Π½ΠΈΠ΅ ΠΌΠ°Ρ‚Π΅Ρ€ΠΈΠ°Π»Π° опрСдСляСтся Ρ‡Π΅Ρ‚Ρ‹Ρ€ΡŒΠΌΡ свойствами: ΠΌΠ°Ρ‚Π΅Ρ€ΠΈΠ°Π»ΠΎΠΌ, Π΄Π»ΠΈΠ½ΠΎΠΉ, ΠΏΠ»ΠΎΡ‰Π°Π΄ΡŒΡŽ ΠΈ Ρ‚Π΅ΠΌΠΏΠ΅Ρ€Π°Ρ‚ΡƒΡ€ΠΎΠΉ. ΠŸΠ΅Ρ€Π²Ρ‹Π΅ Ρ‚Ρ€ΠΈ свойства связаны ΡΠ»Π΅Π΄ΡƒΡŽΡ‰ΠΈΠΌ ΡƒΡ€Π°Π²Π½Π΅Π½ΠΈΠ΅ΠΌ ΠΏΡ€ΠΈ T = 20 Β° C (комнатная Ρ‚Π΅ΠΌΠΏΠ΅Ρ€Π°Ρ‚ΡƒΡ€Π°):

    Π‘ΠΎΠΏΡ€ΠΎΡ‚ΠΈΠ²Π»Π΅Π½ΠΈΠ΅ ΠΈ связь с Ρ€Π°Π·ΠΌΠ΅Ρ€ΠΎΠΌ ΠΏΡ€ΠΎΠ²ΠΎΠ΄Π°

    ΠšΡ€ΡƒΠ³Π»Ρ‹Π΅ ΠΏΡ€ΠΎΠ²ΠΎΠ΄Π½ΠΈΠΊΠΈ (ΠΏΡ€ΠΎΠ²ΠΎΠ΄Π° / ΠΊΠ°Π±Π΅Π»ΠΈ)

    ΠŸΠΎΡΠΊΠΎΠ»ΡŒΠΊΡƒ извСстно, Ρ‡Ρ‚ΠΎ Π‘ΠΎΠΏΡ€ΠΎΡ‚ΠΈΠ²Π»Π΅Π½ΠΈΠ΅ ΠΏΡ€ΠΎΠ²ΠΎΠ΄Π½ΠΈΠΊΠ° прямо ΠΏΡ€ΠΎΠΏΠΎΡ€Ρ†ΠΈΠΎΠ½Π°Π»ΡŒΠ½ΠΎ Π΅Π³ΠΎ Π΄Π»ΠΈΠ½Π΅, ΠΈ Ссли Π½Π°ΠΌ Π΄Π°Π½ΠΎ сопротивлСниС Π΅Π΄ΠΈΠ½ΠΈΡ‡Π½ΠΎΠΉ Π΄Π»ΠΈΠ½Ρ‹ ΠΏΡ€ΠΎΠ²ΠΎΠ΄Π°, ΠΌΡ‹ ΠΌΠΎΠΆΠ΅ΠΌ Π»Π΅Π³ΠΊΠΎ Π²Ρ‹Ρ‡ΠΈΡΠ»ΠΈΡ‚ΡŒ сопротивлСниС любой Π΄Π»ΠΈΠ½Ρ‹ ΠΏΡ€ΠΎΠ²ΠΎΠ΄Π° ΠΈΠ· Ρ‚ΠΎΠ³ΠΎ ΠΆΠ΅ ΠΌΠ°Ρ‚Π΅Ρ€ΠΈΠ°Π»Π°, ΠΈΠΌΠ΅ΡŽΡ‰Π΅Π³ΠΎ Ρ‚ΠΎΡ‚ ΠΆΠ΅ Π΄ΠΈΠ°ΠΌΠ΅Ρ‚Ρ€.ΠšΡ€ΠΎΠΌΠ΅ Ρ‚ΠΎΠ³ΠΎ, ΠΏΠΎΡΠΊΠΎΠ»ΡŒΠΊΡƒ извСстно, Ρ‡Ρ‚ΠΎ сопротивлСниС ΠΏΡ€ΠΎΠ²ΠΎΠ΄Π½ΠΈΠΊΠ° ΠΎΠ±Ρ€Π°Ρ‚Π½ΠΎ ΠΏΡ€ΠΎΠΏΠΎΡ€Ρ†ΠΈΠΎΠ½Π°Π»ΡŒΠ½ΠΎ Π΅Π³ΠΎ ΠΏΠ»ΠΎΡ‰Π°Π΄ΠΈ ΠΏΠΎΠΏΠ΅Ρ€Π΅Ρ‡Π½ΠΎΠ³ΠΎ сСчСния, ΠΈ Ссли Π½Π°ΠΌ Π΄Π°Π½ΠΎ сопротивлСниС ΠΎΡ‚Ρ€Π΅Π·ΠΊΠ° ΠΏΡ€ΠΎΠ²ΠΎΠ΄Π° с Π΅Π΄ΠΈΠ½ΠΈΡ‡Π½ΠΎΠΉ ΠΏΠ»ΠΎΡ‰Π°Π΄ΡŒΡŽ ΠΏΠΎΠΏΠ΅Ρ€Π΅Ρ‡Π½ΠΎΠ³ΠΎ сСчСния, ΠΌΡ‹ ΠΌΠΎΠΆΠ΅ΠΌ Π²Ρ‹Ρ‡ΠΈΡΠ»ΠΈΡ‚ΡŒ сопротивлСниС Ρ‚Π°ΠΊΠΎΠΉ ΠΆΠ΅ Π΄Π»ΠΈΠ½Ρ‹. ΠΈΠ· ΠΏΡ€ΠΎΠ²ΠΎΠ»ΠΎΠΊΠΈ ΠΈΠ· Ρ‚ΠΎΠ³ΠΎ ΠΆΠ΅ ΠΌΠ°Ρ‚Π΅Ρ€ΠΈΠ°Π»Π° любой ΠΏΠ»ΠΎΡ‰Π°Π΄ΠΈ сСчСния. Π‘Π»Π΅Π΄ΠΎΠ²Π°Ρ‚Π΅Π»ΡŒΠ½ΠΎ, Ссли ΠΌΡ‹ Π·Π½Π°Π΅ΠΌ сопротивлСниС Π΄Π°Π½Π½ΠΎΠ³ΠΎ ΠΏΡ€ΠΎΠ²ΠΎΠ΄Π½ΠΈΠΊΠ°, ΠΌΡ‹ ΠΌΠΎΠΆΠ΅ΠΌ Ρ€Π°ΡΡΡ‡ΠΈΡ‚Π°Ρ‚ΡŒ сопротивлСниС для любого ΠΏΡ€ΠΎΠ²ΠΎΠ΄Π½ΠΈΠΊΠ° ΠΈΠ· Ρ‚ΠΎΠ³ΠΎ ΠΆΠ΅ ΠΌΠ°Ρ‚Π΅Ρ€ΠΈΠ°Π»Π° ΠΏΡ€ΠΈ Ρ‚ΠΎΠΉ ΠΆΠ΅ Ρ‚Π΅ΠΌΠΏΠ΅Ρ€Π°Ρ‚ΡƒΡ€Π΅. Из ΠΎΡ‚Π½ΠΎΡˆΠ΅Π½ΠΈΡ:

    МоТно Ρ‚Π°ΠΊΠΆΠ΅ Π·Π°ΠΏΠΈΡΠ°Ρ‚ΡŒ:

    Если Ρƒ нас Π΅ΡΡ‚ΡŒ ΠΏΡ€ΠΎΠ²ΠΎΠ΄Π½ΠΈΠΊ Π΄Π»ΠΈΠ½ΠΎΠΉ 1 ΠΌΠ΅Ρ‚Ρ€ (ΠΌ) с ΠΏΠ»ΠΎΡ‰Π°Π΄ΡŒΡŽ ΠΏΠΎΠΏΠ΅Ρ€Π΅Ρ‡Π½ΠΎΠ³ΠΎ сСчСния 1 (ΠΌΠΈΠ»Π»ΠΈΠΌΠ΅Ρ‚Ρ€) ΠΌΠΌ 2 ΠΈ сопротивлСниСм 0 .017 Ом, ΠΊΠ°ΠΊΠΎΠ²ΠΎ сопротивлСниС 50 ΠΌ ΠΏΡ€ΠΎΠ²ΠΎΠ΄Π° ΠΈΠ· Ρ‚ΠΎΠ³ΠΎ ΠΆΠ΅ ΠΌΠ°Ρ‚Π΅Ρ€ΠΈΠ°Π»Π°, Π½ΠΎ с ΠΏΠ»ΠΎΡ‰Π°Π΄ΡŒΡŽ ΠΏΠΎΠΏΠ΅Ρ€Π΅Ρ‡Π½ΠΎΠ³ΠΎ сСчСния 0,25 ΠΌΠΌ 2 ?

    Π’ Ρ‚ΠΎ врСмя ΠΊΠ°ΠΊ Π΅Π΄ΠΈΠ½ΠΈΡ†Ρ‹ БИ ΠΎΠ±Ρ‹Ρ‡Π½ΠΎ ΠΈΡΠΏΠΎΠ»ΡŒΠ·ΡƒΡŽΡ‚ΡΡ ΠΏΡ€ΠΈ Π°Π½Π°Π»ΠΈΠ·Π΅ элСктричСских Ρ†Π΅ΠΏΠ΅ΠΉ, элСктричСскиС ΠΏΡ€ΠΎΠ²ΠΎΠ΄Π½ΠΈΠΊΠΈ Π² Π‘Π΅Π²Π΅Ρ€Π½ΠΎΠΉ АмСрикС всС Π΅Ρ‰Π΅ производятся с использованиСм стопы Π² качСствС Π΅Π΄ΠΈΠ½ΠΈΡ†Ρ‹ Π΄Π»ΠΈΠ½Ρ‹ ΠΈ ΠΌΠΈΠ» (ΠΎΠ΄Π½Π° тысячная дюйма) Π² качСствС Π΅Π΄ΠΈΠ½ΠΈΡ†Ρ‹ Π΄ΠΈΠ°ΠΌΠ΅Ρ‚Ρ€Π°. ΠŸΡ€Π΅ΠΆΠ΄Π΅ Ρ‡Π΅ΠΌ ΠΈΡΠΏΠΎΠ»ΡŒΠ·ΠΎΠ²Π°Ρ‚ΡŒ ΡƒΡ€Π°Π²Π½Π΅Π½ΠΈΠ΅ R = (ρ Γ— l) ⁄A для расчСта сопротивлСния ΠΏΡ€ΠΎΠ²ΠΎΠ΄Π½ΠΈΠΊΠ° Π΄Π°Π½Π½ΠΎΠ³ΠΎ амСриканского ΠΊΠ°Π»ΠΈΠ±Ρ€Π° ΠΏΡ€ΠΎΠ²ΠΎΠ΄ΠΎΠ² (AWG), ΠΏΠ»ΠΎΡ‰Π°Π΄ΡŒ ΠΏΠΎΠΏΠ΅Ρ€Π΅Ρ‡Π½ΠΎΠ³ΠΎ сСчСния Π² ΠΊΠ²Π°Π΄Ρ€Π°Ρ‚Π½Ρ‹Ρ… ΠΌΠ΅Ρ‚Ρ€Π°Ρ… Π΄ΠΎΠ»ΠΆΠ½Π° Π±Ρ‹Ρ‚ΡŒ ΠΎΠΏΡ€Π΅Π΄Π΅Π»Π΅Π½Π° с использованиСм коэффициСнта прСобразования 1 mil = 0. .0254 ΠΌΠΌ. Бамая удобная Π΅Π΄ΠΈΠ½ΠΈΡ†Π° Π΄Π»ΠΈΠ½Ρ‹ ΠΏΡ€ΠΎΠ²ΠΎΠ»ΠΎΠΊΠΈ — стопа. Π˜ΡΠΏΠΎΠ»ΡŒΠ·ΡƒΡ эти стандарты, Π΅Π΄ΠΈΠ½ΠΈΡ†Π΅ΠΉ измСрСния являСтся ΠΌΠΈΠ»-Ρ„ΡƒΡ‚. Π’Π°ΠΊΠΈΠΌ ΠΎΠ±Ρ€Π°Π·ΠΎΠΌ, ΠΏΡ€ΠΎΠ²ΠΎΠ»ΠΎΠΊΠ° ΠΈΠΌΠ΅Π΅Ρ‚ Π΅Π΄ΠΈΠ½ΠΈΡ‡Π½Ρ‹ΠΉ Ρ€Π°Π·ΠΌΠ΅Ρ€, Ссли ΠΎΠ½Π° ΠΈΠΌΠ΅Π΅Ρ‚ Π΄ΠΈΠ°ΠΌΠ΅Ρ‚Ρ€ 1 ΠΌΠΈΠ» ΠΈ Π΄Π»ΠΈΠ½Ρƒ 1 Ρ„ΡƒΡ‚.

    Π’ случаС использования ΠΌΠ΅Π΄Π½Ρ‹Ρ… ΠΏΡ€ΠΎΠ²ΠΎΠ΄Π½ΠΈΠΊΠΎΠ² ΠΌΡ‹ избавляСмся ΠΎΡ‚ ΡƒΡ‚ΠΎΠΌΠΈΡ‚Π΅Π»ΡŒΠ½Ρ‹Ρ… вычислСний с ΠΏΠΎΠΌΠΎΡ‰ΡŒΡŽ Ρ‚Π°Π±Π»ΠΈΡ†Ρ‹, ΠΏΠΎΠΊΠ°Π·Π°Π½Π½ΠΎΠΉ Π½Π° РисункС 12-43. ΠžΠ±Ρ€Π°Ρ‚ΠΈΡ‚Π΅ Π²Π½ΠΈΠΌΠ°Π½ΠΈΠ΅, Ρ‡Ρ‚ΠΎ Ρ€Π°Π·ΠΌΠ΅Ρ€Ρ‹ ΠΏΠΎΠΏΠ΅Ρ€Π΅Ρ‡Π½ΠΎΠ³ΠΎ сСчСния, ΡƒΠΊΠ°Π·Π°Π½Π½Ρ‹Π΅ Π² Ρ‚Π°Π±Π»ΠΈΡ†Π΅, Ρ‚Π°ΠΊΠΎΠ²Ρ‹, Ρ‡Ρ‚ΠΎ ΠΊΠ°ΠΆΠ΄ΠΎΠ΅ ΡƒΠΌΠ΅Π½ΡŒΡˆΠ΅Π½ΠΈΠ΅ Π½Π° ΠΎΠ΄ΠΈΠ½ Π½ΠΎΠΌΠ΅Ρ€ Π΄Π°Ρ‚Ρ‡ΠΈΠΊΠ° равняСтся 25-ΠΏΡ€ΠΎΡ†Π΅Π½Ρ‚Π½ΠΎΠΌΡƒ ΡƒΠ²Π΅Π»ΠΈΡ‡Π΅Π½ΠΈΡŽ ΠΏΠ»ΠΎΡ‰Π°Π΄ΠΈ ΠΏΠΎΠΏΠ΅Ρ€Π΅Ρ‡Π½ΠΎΠ³ΠΎ сСчСния.Из-Π·Π° этого ΡƒΠΌΠ΅Π½ΡŒΡˆΠ΅Π½ΠΈΠ΅ Ρ‚Ρ€Π΅Ρ… ΠΊΠ°Π»ΠΈΠ±Ρ€ΠΎΠ²ΠΎΡ‡Π½Ρ‹Ρ… чисСл ΠΎΠ·Π½Π°Ρ‡Π°Π΅Ρ‚ ΡƒΠ²Π΅Π»ΠΈΡ‡Π΅Π½ΠΈΠ΅ ΠΏΠ»ΠΎΡ‰Π°Π΄ΠΈ ΠΏΠΎΠΏΠ΅Ρ€Π΅Ρ‡Π½ΠΎΠ³ΠΎ сСчСния ΠΏΡ€ΠΈΠΌΠ΅Ρ€Π½ΠΎ Π½Π° 2: 1. Аналогичным ΠΎΠ±Ρ€Π°Π·ΠΎΠΌ, ΠΈΠ·ΠΌΠ΅Π½Π΅Π½ΠΈΠ΅ дСсяти ΠΊΠ°Π»ΠΈΠ±Ρ€ΠΎΠ²ΠΎΡ‡Π½Ρ‹Ρ… Π½ΠΎΠΌΠ΅Ρ€ΠΎΠ² ΠΏΡ€ΠΎΠ²ΠΎΠ΄ΠΎΠ² прСдставляСт собой ΠΈΠ·ΠΌΠ΅Π½Π΅Π½ΠΈΠ΅ ΠΏΠ»ΠΎΡ‰Π°Π΄ΠΈ ΠΏΠΎΠΏΠ΅Ρ€Π΅Ρ‡Π½ΠΎΠ³ΠΎ сСчСния 10: 1 — ΠΊΡ€ΠΎΠΌΠ΅ Ρ‚ΠΎΠ³ΠΎ, ΠΏΡ€ΠΈ ΡƒΠ΄Π²ΠΎΠ΅Π½ΠΈΠΈ ΠΏΠ»ΠΎΡ‰Π°Π΄ΠΈ ΠΏΠΎΠΏΠ΅Ρ€Π΅Ρ‡Π½ΠΎΠ³ΠΎ сСчСния ΠΏΡ€ΠΎΠ²ΠΎΠ΄Π½ΠΈΠΊΠ° сопротивлСниС ΡƒΠΌΠ΅Π½ΡŒΡˆΠ°Π΅Ρ‚ΡΡ Π²Π΄Π²ΠΎΠ΅. УмСньшСниС числа сСчСний ΠΏΡ€ΠΎΠ²ΠΎΠ΄ΠΎΠ² Π½Π° Ρ‚Ρ€ΠΈ ΡƒΠΌΠ΅Π½ΡŒΡˆΠ°Π΅Ρ‚ сопротивлСниС ΠΏΡ€ΠΎΠ²ΠΎΠ΄Π½ΠΈΠΊΠ° Π·Π°Π΄Π°Π½Π½ΠΎΠΉ Π΄Π»ΠΈΠ½Ρ‹ Π²Π΄Π²ΠΎΠ΅.

    Рисунок 12-43. Π’Π°Π±Π»ΠΈΡ†Π° прСобразования ΠΏΡ€ΠΈ использовании ΠΌΠ΅Π΄Π½Ρ‹Ρ… ΠΆΠΈΠ».

    ΠŸΡ€ΡΠΌΠΎΡƒΠ³ΠΎΠ»ΡŒΠ½Ρ‹Π΅ ΠΏΡ€ΠΎΠ²ΠΎΠ΄Π½ΠΈΠΊΠΈ (ΡˆΠΈΠ½Ρ‹)

    Для вычислСния ΠΏΠ»ΠΎΡ‰Π°Π΄ΠΈ ΠΏΠΎΠΏΠ΅Ρ€Π΅Ρ‡Π½ΠΎΠ³ΠΎ сСчСния ΠΏΡ€ΠΎΠ²ΠΎΠ΄Π½ΠΈΠΊΠ° Π² ΠΊΠ²Π°Π΄Ρ€Π°Ρ‚Π½Ρ‹Ρ… ΠΌΠΈΠ»Π°Ρ… Π΄Π»ΠΈΠ½Π° ΠΎΠ΄Π½ΠΎΠΉ стороны Π² ΠΌΠΈΠ»Π°Ρ… возводится Π² ΠΊΠ²Π°Π΄Ρ€Π°Ρ‚.Π’ случаС ΠΏΡ€ΡΠΌΠΎΡƒΠ³ΠΎΠ»ΡŒΠ½ΠΎΠ³ΠΎ ΠΏΡ€ΠΎΠ²ΠΎΠ΄Π½ΠΈΠΊΠ° Π΄Π»ΠΈΠ½Π° ΠΎΠ΄Π½ΠΎΠΉ стороны умноТаСтся Π½Π° Π΄Π»ΠΈΠ½Ρƒ Π΄Ρ€ΡƒΠ³ΠΎΠΉ. НапримСр, обычная ΠΏΡ€ΡΠΌΠΎΡƒΠ³ΠΎΠ»ΡŒΠ½Π°Ρ шина (большой, ΡΠΏΠ΅Ρ†ΠΈΠ°Π»ΡŒΠ½Ρ‹ΠΉ ΠΏΡ€ΠΎΠ²ΠΎΠ΄Π½ΠΈΠΊ) ΠΈΠΌΠ΅Π΅Ρ‚ Ρ‚ΠΎΠ»Ρ‰ΠΈΠ½Ρƒ 3⁄8 дюйма ΠΈ ΡˆΠΈΡ€ΠΈΠ½Ρƒ 4 дюйма. Π’ΠΎΠ»Ρ‰ΠΈΠ½Π° 3⁄8 дюйма ΠΌΠΎΠΆΠ΅Ρ‚ Π±Ρ‹Ρ‚ΡŒ Π²Ρ‹Ρ€Π°ΠΆΠ΅Π½Π° ΠΊΠ°ΠΊ 0,375 дюйма. ΠŸΠΎΡΠΊΠΎΠ»ΡŒΠΊΡƒ 1000 ΠΌΠΈΠ» равняСтся 1 Π΄ΡŽΠΉΠΌΡƒ, ΡˆΠΈΡ€ΠΈΠ½Ρƒ Π² Π΄ΡŽΠΉΠΌΠ°Ρ… ΠΌΠΎΠΆΠ½ΠΎ ΠΏΡ€Π΅ΠΎΠ±Ρ€Π°Π·ΠΎΠ²Π°Ρ‚ΡŒ Π² 4000 ΠΌΠΈΠ». ΠŸΠ»ΠΎΡ‰Π°Π΄ΡŒ ΠΏΠΎΠΏΠ΅Ρ€Π΅Ρ‡Π½ΠΎΠ³ΠΎ сСчСния ΠΏΡ€ΡΠΌΠΎΡƒΠ³ΠΎΠ»ΡŒΠ½ΠΎΠ³ΠΎ ΠΏΡ€ΠΎΠ²ΠΎΠ΄Π½ΠΈΠΊΠ° находится ΠΏΡƒΡ‚Π΅ΠΌ прСобразования 0,375 Π² ΠΌΠΈΠ» (375 ΠΌΠΈΠ» Γ— 4000 ΠΌΠΈΠ» = 1 500 000 ΠΊΠ²Π°Π΄Ρ€Π°Ρ‚Π½Ρ‹Ρ… ΠΌΠΈΠ»).

    Π›Π΅Ρ‚Π½Ρ‹ΠΉ ΠΌΠ΅Ρ…Π°Π½ΠΈΠΊ Ρ€Π΅ΠΊΠΎΠΌΠ΅Π½Π΄ΡƒΠ΅Ρ‚

    Π’ΠΎΠΊ, сопротивлСниС, напряТСниС ΠΈ ΠΌΠΎΡ‰Π½ΠΎΡΡ‚ΡŒ

    Π’Π΅ΠΊΡƒΡ‰ΠΈΠΉ
    Π’ΠΎΠΊ — это ΠΌΠ΅Ρ€Π° ΠΏΠΎΡ‚ΠΎΠΊΠ° элСктричСского заряда Ρ‡Π΅Ρ€Π΅Π· ΠΌΠ°Ρ‚Π΅Ρ€ΠΈΠ°Π».ΠœΠ°Ρ‚Π΅Ρ€ΠΈΠ°Π», ΠΊΠΎΡ‚ΠΎΡ€Ρ‹ΠΉ ΠΌΠΎΠΆΠ΅Ρ‚ ΠΏΠ΅Ρ€Π΅Π½ΠΎΡΠΈΡ‚ΡŒ ΠΏΠΎΡ‚ΠΎΠΊ заряда, называСтся ΠΏΡ€ΠΎΠ²ΠΎΠ΄Π½ΠΈΠΊΠΎΠΌ. Π’ΠΎΠΊ опрСдСляСтся ΠΊΠ°ΠΊ количСство заряда, ΠΊΠΎΡ‚ΠΎΡ€ΠΎΠ΅ ΠΏΡ€ΠΎΡ…ΠΎΠ΄ΠΈΡ‚ Ρ‡Π΅Ρ€Π΅Π· ΠΏΡ€ΠΎΠ²ΠΎΠ΄Π½ΠΈΠΊ Π·Π° ΠΎΠΏΡ€Π΅Π΄Π΅Π»Π΅Π½Π½ΠΎΠ΅ врСмя. Π•Π΄ΠΈΠ½ΠΈΡ†Π΅ΠΉ измСрСния Ρ‚ΠΎΠΊΠ° являСтся Π°ΠΌΠΏΠ΅Ρ€ (А), ΠΊΠΎΡ‚ΠΎΡ€Ρ‹ΠΉ Ρ€Π°Π²Π΅Π½ ΠΎΠ΄Π½ΠΎΠΌΡƒ ΠΊΡƒΠ»ΠΎΠ½Ρƒ Π² сСкунду (ΠΊΡƒΠ»ΠΎΠ½ — Π΅Π΄ΠΈΠ½ΠΈΡ†Π° заряда),

    Π‘ΠΈΠΌΠ²ΠΎΠ» I ΠΈΡΠΏΠΎΠ»ΡŒΠ·ΡƒΠ΅Ρ‚ΡΡ для обозначСния Ρ‚ΠΎΠΊΠ° (хотя J часто ΠΈΡΠΏΠΎΠ»ΡŒΠ·ΡƒΠ΅Ρ‚ΡΡ Π² ΠΈΠ½ΠΆΠ΅Π½Π΅Ρ€Π½Ρ‹Π΅ источники). Π’ΠΎΠΊ I Ρ‡Π΅Ρ€Π΅Π· ΠΏΡ€ΠΎΠ²ΠΎΠ΄Π½ΠΈΠΊ зависит ΠΎΡ‚ Π΅Π³ΠΎ ΠΏΠ»ΠΎΡ‰Π°Π΄ΠΈ A , ΠΊΠΎΠ½Ρ†Π΅Π½Ρ‚Ρ€Π°Ρ†ΠΈΠΈ n носитСлСй заряда, Π²Π΅Π»ΠΈΡ‡ΠΈΠ½Ρ‹ заряда q ΠΊΠ°ΠΆΠ΄ΠΎΠ³ΠΎ носитСля ΠΈ Π²Π΅Π»ΠΈΡ‡ΠΈΠ½Ρ‹ ΠΈΡ… срСднСй (ΠΈΠ»ΠΈ Β«Π΄Ρ€Π΅ΠΉΡ„ΡƒΡŽΡ‰Π΅ΠΉΒ») скорости. v d ,

    ΠŸΠ»ΠΎΡ‚Π½ΠΎΡΡ‚ΡŒ Ρ‚ΠΎΠΊΠ° — это количСство Ρ‚ΠΎΠΊΠ°, ΠΏΡ€ΠΎΡ‚Π΅ΠΊΠ°ΡŽΡ‰Π΅Π³ΠΎ Ρ‡Π΅Ρ€Π΅Π· ΠΏΡ€ΠΎΠ²ΠΎΠ΄Π½ΠΈΠΊ, Π΄Π΅Π»Π΅Π½Π½ΠΎΠ΅ Π½Π° Π΅Π³ΠΎ ΠΏΠ»ΠΎΡ‰Π°Π΄ΡŒ,

    НаправлСниС ΠΏΠΎΡ‚ΠΎΠΊΠ° Ρ‚ΠΎΠΊΠ° опрСдСляСтся Π² Ρ‚Π΅Ρ€ΠΌΠΈΠ½Π°Ρ… ΠΏΠΎΡ‚ΠΎΠΊΠ° ΠΏΠΎΠ»ΠΎΠΆΠΈΡ‚Π΅Π»ΡŒΠ½Ρ‹Ρ… зарядов (Π΄Π°ΠΆΠ΅ Ссли фактичСскиС носитСли заряда ΠΎΡ‚Ρ€ΠΈΡ†Π°Ρ‚Π΅Π»ΡŒΠ½Ρ‹).Π•Π΄ΠΈΠ½ΠΈΡ†Π° измСрСния плотности Ρ‚ΠΎΠΊΠ° — АмпСры Π½Π° ΠΊΠ²Π°Π΄Ρ€Π°Ρ‚Π½Ρ‹ΠΉ ΠΌΠ΅Ρ‚Ρ€ (А / ΠΌ 2 ).

    УдСльноС сопротивлСниС
    НСкоторыС ΠΏΡ€ΠΎΠ²ΠΎΠ΄Π½ΠΈΠΊΠΈ пСрСносят заряд Π»Π΅Π³Ρ‡Π΅, Ρ‡Π΅ΠΌ Π΄Ρ€ΡƒΠ³ΠΈΠ΅. УдСльноС сопротивлСниС ΠΌΠ°Ρ‚Π΅Ρ€ΠΈΠ°Π»Π° описываСт, насколько Π»Π΅Π³ΠΊΠΎ ΠΌΠΎΠΆΠ΅Ρ‚ Ρ‚Π΅Ρ‡ΡŒ заряд. Π₯ΠΎΡ€ΠΎΡˆΠΈΠ΅ ΠΏΡ€ΠΎΠ²ΠΎΠ΄Π½ΠΈΠΊΠΈ ΠΈΠΌΠ΅ΡŽΡ‚ нСбольшоС ΡƒΠ΄Π΅Π»ΡŒΠ½ΠΎΠ΅ сопротивлСниС, Π° Ρ…ΠΎΡ€ΠΎΡˆΠΈΠ΅ изоляторы — большоС. УдСльноС сопротивлСниС ρ (грСчСская Π±ΡƒΠΊΠ²Π° Β«Ρ€ΠΎΒ») Ρ€Π°Π²Π½ΠΎ Π²Π΅Π»ΠΈΡ‡ΠΈΠ½Π΅ элСктричСского поля Π² ΠΌΠ°Ρ‚Π΅Ρ€ΠΈΠ°Π»Π΅, Π΄Π΅Π»Π΅Π½Π½ΠΎΠΉ Π½Π° ΠΏΠ»ΠΎΡ‚Π½ΠΎΡΡ‚ΡŒ Ρ‚ΠΎΠΊΠ°,

    Π•Π΄ΠΈΠ½ΠΈΡ†Π° измСрСния Π²Π΅Π»ΠΈΡ‡ΠΈΠ½Ρ‹ элСктричСского поля — Π²ΠΎΠ»ΡŒΡ‚ Π½Π° ΠΌΠ΅Ρ‚Ρ€ (Π’ / ΠΌ). ), Π° Π΅Π΄ΠΈΠ½ΠΈΡ†Π΅ΠΉ измСрСния плотности Ρ‚ΠΎΠΊΠ° являСтся АмпСр Π½Π° ΠΊΠ²Π°Π΄Ρ€Π°Ρ‚Π½Ρ‹ΠΉ ΠΌΠ΅Ρ‚Ρ€ (А / ΠΌ 2 ), поэтому Π΅Π΄ΠΈΠ½ΠΈΡ†Π΅ΠΉ измСрСния ΡƒΠ΄Π΅Π»ΡŒΠ½ΠΎΠ³ΠΎ сопротивлСния являСтся Π’ΠΎΠ»ΡŒΡ‚-ΠΌΠ΅Ρ‚Ρ€ Π½Π° АмпСр,

    МногиС ΠΏΡ€ΠΎΠ²ΠΎΠ΄Π½ΠΈΠΊΠΈ ΠΏΠΎΠ΄Ρ‡ΠΈΠ½ΡΡŽΡ‚ΡΡ Π·Π°ΠΊΠΎΠ½Ρƒ Ома.ΠœΠ°Ρ‚Π΅Ρ€ΠΈΠ°Π»Ρ‹, ΠΊΠΎΡ‚ΠΎΡ€Ρ‹Π΅ ΠΏΠΎΠ΄Ρ‡ΠΈΠ½ΡΡŽΡ‚ΡΡ Π·Π°ΠΊΠΎΠ½Ρƒ Ома, ΠΈΠΌΠ΅ΡŽΡ‚ постоянноС ΡƒΠ΄Π΅Π»ΡŒΠ½ΠΎΠ΅ сопротивлСниС нСзависимо ΠΎΡ‚ Π·Π½Π°Ρ‡Π΅Π½ΠΈΠΉ элСктричСского поля E ΠΈ плотности Ρ‚ΠΎΠΊΠ° J. Π€ΠΎΡ€ΠΌΡƒΠ»Ρ‹, относящиСся ΠΊ цСпям, Π²Π΅Ρ€Π½Ρ‹ для «омичСских» ΠΌΠ°Ρ‚Π΅Ρ€ΠΈΠ°Π»ΠΎΠ², Π° «нСомичСскиС» ΠΌΠ°Ρ‚Π΅Ρ€ΠΈΠ°Π»Ρ‹ Π² этом курсС Π½Π΅ ΠΎΠ±ΡΡƒΠΆΠ΄Π°ΡŽΡ‚ΡΡ.

    УдСльноС сопротивлСниС омичСского ΠΏΡ€ΠΎΠ²ΠΎΠ΄Π½ΠΈΠΊΠ° зависит ΠΎΡ‚ Ρ‚Π΅ΠΌΠΏΠ΅Ρ€Π°Ρ‚ΡƒΡ€Ρ‹ ΠΌΠ°Ρ‚Π΅Ρ€ΠΈΠ°Π»Π°. ЗависящСС ΠΎΡ‚ Ρ‚Π΅ΠΌΠΏΠ΅Ρ€Π°Ρ‚ΡƒΡ€Ρ‹ ΡƒΠ΄Π΅Π»ΡŒΠ½ΠΎΠ΅ сопротивлСниС ρ (T) ΠΌΠΎΠΆΠ½ΠΎ Π½Π°ΠΉΡ‚ΠΈ ΠΏΠΎ Ρ„ΠΎΡ€ΠΌΡƒΠ»Π΅:

    Π­Ρ‚Π° Ρ„ΠΎΡ€ΠΌΡƒΠ»Π° Ρ‚Ρ€Π΅Π±ΡƒΠ΅Ρ‚ ρ 0 , ΡƒΠ΄Π΅Π»ΡŒΠ½ΠΎΠ³ΠΎ сопротивлСния ΠΏΡ€ΠΈ эталонной Ρ‚Π΅ΠΌΠΏΠ΅Ρ€Π°Ρ‚ΡƒΡ€Π΅ T 0 .Π’Π΅ΠΌΠΏΠ΅Ρ€Π°Ρ‚ΡƒΡ€Π½Ρ‹ΠΉ коэффициСнт ΡƒΠ΄Π΅Π»ΡŒΠ½ΠΎΠ³ΠΎ сопротивлСния Ξ± Ρ€Π°Π·Π»ΠΈΡ‡Π΅Π½ для ΠΊΠ°ΠΆΠ΄ΠΎΠ³ΠΎ ΠΌΠ°Ρ‚Π΅Ρ€ΠΈΠ°Π»Π°. Для Ρ‚Π΅ΠΌΠΏΠ΅Ρ€Π°Ρ‚ΡƒΡ€ Π² градусах ЦСльсия (℃) Ρ‚Π΅ΠΌΠΏΠ΅Ρ€Π°Ρ‚ΡƒΡ€Π½Ρ‹ΠΉ коэффициСнт ΡƒΠ΄Π΅Π»ΡŒΠ½ΠΎΠ³ΠΎ сопротивлСния ΠΈΠΌΠ΅Π΅Ρ‚ Π΅Π΄ΠΈΠ½ΠΈΡ†Ρ‹: 1 / ℃ = (℃) (-1)

    Π‘ΠΎΠΏΡ€ΠΎΡ‚ΠΈΠ²Π»Π΅Π½ΠΈΠ΅
    УдСльноС сопротивлСниС — это свойство ΠΌΠ°Ρ‚Π΅Ρ€ΠΈΠ°Π»Π°, Π° сопротивлСниС — это свойство ΠΎΠΏΡ€Π΅Π΄Π΅Π»Π΅Π½Π½ΠΎΠ³ΠΎ куска этого ΠΌΠ°Ρ‚Π΅Ρ€ΠΈΠ°Π»Π°. Π‘ΠΎΠΏΡ€ΠΎΡ‚ΠΈΠ²Π»Π΅Π½ΠΈΠ΅ куска ΠΏΡ€ΠΎΠ²ΠΎΠ΄Π½ΠΈΠΊΠ° зависит ΠΎΡ‚ Π΅Π³ΠΎ Π΄Π»ΠΈΠ½Ρ‹ L, ΠΏΠ»ΠΎΡ‰Π°Π΄ΠΈ A ΠΈ ΡƒΠ΄Π΅Π»ΡŒΠ½ΠΎΠ³ΠΎ сопротивлСния ρ,

    Π•Π΄ΠΈΠ½ΠΈΡ†Π΅ΠΉ измСрСния сопротивлСния являСтся Ом, ΠΊΠΎΡ‚ΠΎΡ€Ρ‹ΠΉ обозначаСтся грСчСской Π±ΡƒΠΊΠ²ΠΎΠΉ Ξ© (Β«ΠΎΠΌΠ΅Π³Π°Β»).Один Ом Ρ€Π°Π²Π΅Π½ ΠΎΠ΄Π½ΠΎΠΌΡƒ Π’ΠΎΠ»ΡŒΡ‚Ρƒ Π½Π° АмпСр,

    Π‘ΠΎΠΏΡ€ΠΎΡ‚ΠΈΠ²Π»Π΅Π½ΠΈΠ΅ зависит ΠΎΡ‚ Ρ‚Π΅ΠΌΠΏΠ΅Ρ€Π°Ρ‚ΡƒΡ€Ρ‹ Ρ‚Π°ΠΊ ΠΆΠ΅, ΠΊΠ°ΠΊ ΠΈ ΡƒΠ΄Π΅Π»ΡŒΠ½ΠΎΠ΅ сопротивлСниС,

    Для этой Ρ„ΠΎΡ€ΠΌΡƒΠ»Ρ‹ трСбуСтся R 0 , сопротивлСниС ΠΏΡ€ΠΈ эталонной Ρ‚Π΅ΠΌΠΏΠ΅Ρ€Π°Ρ‚ΡƒΡ€Π΅ T 0 . Π’Π΅ΠΌΠΏΠ΅Ρ€Π°Ρ‚ΡƒΡ€Π½Ρ‹ΠΉ коэффициСнт Ξ± отличаСтся для ΠΊΠ°ΠΆΠ΄ΠΎΠ³ΠΎ ΠΌΠ°Ρ‚Π΅Ρ€ΠΈΠ°Π»Π°, ΠΊΠ°ΠΊ описано Π² Ρ€Π°Π·Π΄Π΅Π»Π΅ Π‘ΠΎΠΏΡ€ΠΎΡ‚ΠΈΠ²Π»Π΅Π½ΠΈΠ΅ .

    РСзистор — это устройство, ΠΊΠΎΡ‚ΠΎΡ€ΠΎΠ΅ ΠΈΡΠΏΠΎΠ»ΡŒΠ·ΡƒΠ΅Ρ‚ΡΡ Π² элСктричСских цСпях ΠΈ ΠΈΠΌΠ΅Π΅Ρ‚ ΠΎΠΏΡ€Π΅Π΄Π΅Π»Π΅Π½Π½ΠΎΠ΅ фиксированноС сопротивлСниС. РСзисторы ΠΈΠ·Π³ΠΎΡ‚Π°Π²Π»ΠΈΠ²Π°ΡŽΡ‚ΡΡ ΠΏΡƒΡ‚Π΅ΠΌ Π²Ρ‹Π±ΠΎΡ€Π° куска ΠΌΠ°Ρ‚Π΅Ρ€ΠΈΠ°Π»Π° с ΠΎΠΏΡ€Π΅Π΄Π΅Π»Π΅Π½Π½Ρ‹ΠΌ ΡƒΠ΄Π΅Π»ΡŒΠ½Ρ‹ΠΌ сопротивлСниСм, Π΄Π»ΠΈΠ½ΠΎΠΉ ΠΈ ΠΏΠ»ΠΎΡ‰Π°Π΄ΡŒΡŽ ΠΈ обСртывания Π΅Π³ΠΎ изолятором с ΠΏΡ€ΠΎΠ²ΠΎΠ΄Π°ΠΌΠΈ, выходящими ΠΈΠ· ΠΊΠ°ΠΆΠ΄ΠΎΠ³ΠΎ ΠΊΠΎΠ½Ρ†Π°.На ΠΏΡ€ΠΈΠ½Ρ†ΠΈΠΏΠΈΠ°Π»ΡŒΠ½Ρ‹Ρ… схСмах ΠΎΠ½ прСдставлСн символом

    НапряТСниС
    НапряТСниС — это Ρ€Π°Π·Π½ΠΈΡ†Π° элСктричСского ΠΏΠΎΡ‚Π΅Π½Ρ†ΠΈΠ°Π»Π° ΠΌΠ΅ΠΆΠ΄Ρƒ двумя Ρ‚ΠΎΡ‡ΠΊΠ°ΠΌΠΈ. Если элСктричСскоС ΠΏΠΎΠ»Π΅ ΠΎΠ΄Π½ΠΎΡ€ΠΎΠ΄Π½ΠΎ Ρ‡Π΅Ρ€Π΅Π· ΠΏΡ€ΠΎΠ²ΠΎΠ΄Π½ΠΈΠΊ, Ρ€Π°Π·Π½ΠΎΡΡ‚ΡŒ ΠΏΠΎΡ‚Π΅Π½Ρ†ΠΈΠ°Π»ΠΎΠ² Π±ΡƒΠ΄Π΅Ρ‚ Ρ€Π°Π²Π½Π°,

    Π˜ΡΠΏΠΎΠ»ΡŒΠ·ΡƒΡ уравнСния Π² Π’ΠΎΠΊ, УдСльноС сопротивлСниС, ΠΈ Π‘ΠΎΠΏΡ€ΠΎΡ‚ΠΈΠ²Π»Π΅Π½ΠΈΠ΅ сСкций, ΠΌΠΎΠΆΠ½ΠΎ Π½Π°ΠΉΡ‚ΠΈ Π΄Ρ€ΡƒΠ³ΠΎΠ΅ ΡƒΡ€Π°Π²Π½Π΅Π½ΠΈΠ΅ для разности ΠΏΠΎΡ‚Π΅Π½Ρ†ΠΈΠ°Π»ΠΎΠ²,

    Π£Ρ€Π°Π²Π½Π΅Π½ΠΈΠ΅ V = IR ΠΎΠ·Π½Π°Ρ‡Π°Π΅Ρ‚, Ρ‡Ρ‚ΠΎ Ρ€Π°Π·Π½ΠΎΡΡ‚ΡŒ ΠΏΠΎΡ‚Π΅Π½Ρ†ΠΈΠ°Π»ΠΎΠ² ΠΈΠ»ΠΈ напряТСниС Π½Π° рСзисторС ΠΌΠΎΠΆΠ½ΠΎ Π½Π°ΠΉΡ‚ΠΈ, ΡƒΠΌΠ½ΠΎΠΆΠΈΠ² Π΅Π³ΠΎ сопротивлСниС Π½Π° Ρ‚ΠΎΠΊ, ΠΏΡ€ΠΎΡ‚Π΅ΠΊΠ°ΡŽΡ‰ΠΈΠΉ Ρ‡Π΅Ρ€Π΅Π· Π½Π΅Π³ΠΎ.Π•Π΄ΠΈΠ½ΠΈΡ†Π΅ΠΉ измСрСния разности ΠΏΠΎΡ‚Π΅Π½Ρ†ΠΈΠ°Π»ΠΎΠ² являСтся Π²ΠΎΠ»ΡŒΡ‚ (Π’), ΠΊΠΎΡ‚ΠΎΡ€Ρ‹ΠΉ Ρ€Π°Π²Π΅Π½ дТоуля Π½Π° ΠΊΡƒΠ»ΠΎΠ½ (Π”ΠΆ / Кл).

    Π˜ΡΡ‚ΠΎΡ‡Π½ΠΈΠΊ напряТСния — это устройство, ΠΈΡΠΏΠΎΠ»ΡŒΠ·ΡƒΠ΅ΠΌΠΎΠ΅ Π² элСктричСских цСпях, ΠΊΠΎΡ‚ΠΎΡ€ΠΎΠ΅ ΠΈΠΌΠ΅Π΅Ρ‚ Ρ„ΠΈΠΊΡΠΈΡ€ΠΎΠ²Π°Π½Π½ΡƒΡŽ Ρ€Π°Π·Π½ΠΎΡΡ‚ΡŒ ΠΏΠΎΡ‚Π΅Π½Ρ†ΠΈΠ°Π»ΠΎΠ² ΠΌΠ΅ΠΆΠ΄Ρƒ Π΅Π³ΠΎ ΠΊΠΎΠ½Ρ†Π°ΠΌΠΈ. Π˜ΡΡ‚ΠΎΡ‡Π½ΠΈΠΊΠΎΠΌ напряТСния ΠΌΠΎΠΆΠ΅Ρ‚ Π±Ρ‹Ρ‚ΡŒ батарСя ΠΈΠ»ΠΈ Π΄Ρ€ΡƒΠ³ΠΎΠΉ источник постоянного Ρ‚ΠΎΠΊΠ° с фиксированной Ρ€Π°Π·Π½ΠΎΡΡ‚ΡŒΡŽ ΠΏΠΎΡ‚Π΅Π½Ρ†ΠΈΠ°Π»ΠΎΠ². На ΠΏΡ€ΠΈΠ½Ρ†ΠΈΠΏΠΈΠ°Π»ΡŒΠ½Ρ‹Ρ… схСмах ΠΎΠ½ прСдставлСн символом

    Если ΠΊΠΎΠ½Ρ†Ρ‹ источника напряТСния соСдинСны Ρ‡Π΅Ρ€Π΅Π· Ρ†Π΅ΠΏΡŒ с Π»ΡŽΠ±Ρ‹ΠΌ количСством рСзисторов ΠΈΠ»ΠΈ Π΄Ρ€ΡƒΠ³ΠΈΡ… ΠΊΠΎΠΌΠΏΠΎΠ½Π΅Π½Ρ‚ΠΎΠ², образуСтся полная Ρ†Π΅ΠΏΡŒ, ΠΈ Ρ‚ΠΎΠΊ ΠΌΠΎΠΆΠ΅Ρ‚ Ρ‚Π΅Ρ‡ΡŒ ΠΎΡ‚ ΠΎΠ΄Π½ΠΎΠΉ ΠΊΠ»Π΅ΠΌΠΌΡ‹ ΠΊ Π΄Ρ€ΡƒΠ³ΠΎΠΉ. Π΄Ρ€ΡƒΠ³ΠΎΠΉ.Если Ρ‚ΠΎΠΊ Ρ‚Π΅Ρ‡Π΅Ρ‚, ΠΎΠ½ Π±ΡƒΠ΄Π΅Ρ‚ ΠΎΠ΄ΠΈΠ½Π°ΠΊΠΎΠ²Ρ‹ΠΌ Π½Π° ΠΎΠ±ΠΎΠΈΡ… Π²Ρ‹Π²ΠΎΠ΄Π°Ρ… источника напряТСния.

    Π˜ΡΡ‚ΠΎΡ‡Π½ΠΈΠΊ напряТСния, ΠΊΠΎΡ‚ΠΎΡ€Ρ‹ΠΉ являСтся Ρ‡Π°ΡΡ‚ΡŒΡŽ ΠΏΠΎΠ»Π½ΠΎΠΉ схСмы, ΠΌΠΎΠΆΠ΅Ρ‚ ΡΠΎΠ·Π΄Π°Π²Π°Ρ‚ΡŒ ΡΠ»Π΅ΠΊΡ‚Ρ€ΠΎΠ΄Π²ΠΈΠΆΡƒΡ‰ΡƒΡŽ силу, которая обозначаСтся символом Ξ΅ («скрипт eΒ»). Π•Π΄ΠΈΠ½ΠΈΡ†Π΅ΠΉ измСрСния элСктродвиТущСй силы являСтся Π²ΠΎΠ»ΡŒΡ‚ (Π’), ΠΊΠΎΡ‚ΠΎΡ€Ρ‹ΠΉ Ρ€Π°Π²Π΅Π½ дТоуля Π½Π° ΠΊΡƒΠ»ΠΎΠ½ (Π”ΠΆ / Кл). Для идСального источника элСктродвиТущая сила Ρ€Π°Π²Π½Π° Ρ€Π°Π·Π½ΠΈΡ†Π΅ напряТСний,

    НастоящиС источники, Ρ‚Π°ΠΊΠΈΠ΅ ΠΊΠ°ΠΊ Π±Π°Ρ‚Π°Ρ€Π΅ΠΈ, Π½Π΅ ΠΈΠ΄Π΅Π°Π»ΡŒΠ½Ρ‹, поэтому сущСствуСт Π½Π΅ΠΊΠΎΡ‚ΠΎΡ€ΠΎΠ΅ Π²Π½ΡƒΡ‚Ρ€Π΅Π½Π½Π΅Π΅ сопротивлСниС.Если Π²Π½ΡƒΡ‚Ρ€Π΅Π½Π½Π΅Π΅ сопротивлСниС Π±Π°Ρ‚Π°Ρ€Π΅ΠΈ Ρ€Π°Π²Π½ΠΎ r, Ρ‚ΠΎ Ρ€Π°Π·Π½ΠΈΡ†Π° напряТСний Π½Π° Π±Π°Ρ‚Π°Ρ€Π΅Π΅ Ρ€Π°Π²Π½Π°

    Π­Ρ‚ΠΎ Ρ‚Π°ΠΊΠΆΠ΅ называСтся напряТСниСм Π½Π° ΠΊΠ»Π΅ΠΌΠΌΠ°Ρ… Π±Π°Ρ‚Π°Ρ€Π΅ΠΈ. Если полная Ρ†Π΅ΠΏΡŒ сдСлана с использованиСм рСзистора с сопротивлСниСм R, Ρ‚ΠΎΠΊ, ΠΏΡ€ΠΎΡ‚Π΅ΠΊΠ°ΡŽΡ‰ΠΈΠΉ ΠΏΠΎ Ρ†Π΅ΠΏΠΈ, ΠΌΠΎΠΆΠ½ΠΎ Π½Π°ΠΉΡ‚ΠΈ с ΠΏΠΎΠΌΠΎΡ‰ΡŒΡŽ уравнСния V = IR,

    Π’ΠΎΠΊ Ρ€Π°Π²Π΅Π½ элСктродвиТущСму. сила источника, дСлСнная Π½Π° ΠΏΠΎΠ»Π½ΠΎΠ΅ сопротивлСниС Ρ†Π΅ΠΏΠΈ.

    ΠœΠΎΡ‰Π½ΠΎΡΡ‚ΡŒ
    ΠœΠΎΡ‰Π½ΠΎΡΡ‚ΡŒ (P) — это ΠΌΠ΅Ρ€Π° скорости, с ΠΊΠΎΡ‚ΠΎΡ€ΠΎΠΉ энСргия пСрСдаСтся ΠΈΠ»ΠΈ ΠΈΡΠΏΠΎΠ»ΡŒΠ·ΡƒΠ΅Ρ‚ΡΡ элСмСнтом схСмы. Π˜ΡΡ‚ΠΎΡ‡Π½ΠΈΠΊΠΈ напряТСния ΠΎΠ±Π΅ΡΠΏΠ΅Ρ‡ΠΈΠ²Π°ΡŽΡ‚ ΠΏΠΈΡ‚Π°Π½ΠΈΠ΅, Π° рСзисторы ΠΈΡΠΏΠΎΠ»ΡŒΠ·ΡƒΡŽΡ‚ ΠΌΠΎΡ‰Π½ΠΎΡΡ‚ΡŒ (рассСивая Π΅Π΅ Π² Π²ΠΈΠ΄Π΅ Ρ‚Π΅ΠΏΠ»Π°). ΠœΠΎΡ‰Π½ΠΎΡΡ‚ΡŒ Ρ€Π°Π²Π½Π° Π½Π°ΠΏΡ€ΡΠΆΠ΅Π½ΠΈΡŽ Π½Π° элСмСнтС схСмы, ΡƒΠΌΠ½ΠΎΠΆΠ΅Π½Π½ΠΎΠΌΡƒ Π½Π° Ρ‚ΠΎΠΊ, ΠΏΡ€ΠΎΡ‚Π΅ΠΊΠ°ΡŽΡ‰ΠΈΠΉ Ρ‡Π΅Ρ€Π΅Π· Π½Π΅Π³ΠΎ,

    Π•Π΄ΠΈΠ½ΠΈΡ†Π΅ΠΉ измСрСния мощности являСтся Π²Π°Ρ‚Ρ‚ (Π’Ρ‚), ΠΊΠΎΡ‚ΠΎΡ€Ρ‹ΠΉ Ρ€Π°Π²Π΅Π½ дТоулям Π² сСкунду,

    Π­Ρ‚ΠΎ ΡΠΎΠΎΡ‚Π½ΠΎΡˆΠ΅Π½ΠΈΠ΅ ΠΌΠΎΠΆΠ΅Ρ‚ Π±Ρ‹Ρ‚ΡŒ опрСдСляСтся ΠΏΠΎ Ρ„ΠΎΡ€ΠΌΡƒΠ»Π΅ для мощности:

    ΠœΠΎΡ‰Π½ΠΎΡΡ‚ΡŒ, потрСбляСмая ΠΈΠ»ΠΈ рассСиваСмая рСзистором, ΠΌΠΎΠΆΠ΅Ρ‚ Π±Ρ‹Ρ‚ΡŒ Π½Π°ΠΉΠ΄Π΅Π½Π° ΠΏΠΎ Ρ„ΠΎΡ€ΠΌΡƒΠ»Π΅ V = IR.Π­Ρ‚Π° Ρ„ΠΎΡ€ΠΌΡƒΠ»Π° ΠΌΠΎΠΆΠ΅Ρ‚ ΠΈΡΠΏΠΎΠ»ΡŒΠ·ΠΎΠ²Π°Ρ‚ΡŒΡΡ для Π·Π°ΠΌΠ΅Π½Ρ‹ напряТСния ΠΈΠ»ΠΈ Ρ‚ΠΎΠΊΠ° Π² Ρ„ΠΎΡ€ΠΌΡƒΠ»Π΅ мощности,

    ,

    ΠΈ

    Выходная ΠΌΠΎΡ‰Π½ΠΎΡΡ‚ΡŒ Π±Π°Ρ‚Π°Ρ€Π΅ΠΈ с Π²Π½ΡƒΡ‚Ρ€Π΅Π½Π½ΠΈΠΌ сопротивлСниСм ΠΌΠΎΠΆΠ΅Ρ‚ Π±Ρ‹Ρ‚ΡŒ Π½Π°ΠΉΠ΄Π΅Π½Π° ΠΏΠΎ Ρ„ΠΎΡ€ΠΌΡƒΠ»Π΅ V = Ξ΅-Ir ΠΈ Ρ„ΠΎΡ€ΠΌΡƒΠ»Π° мощности,

    Skin Depth / Skin Effect and Calculator

    ΠŸΠΎΡΠΊΠΎΠ»ΡŒΠΊΡƒ высокочастотныС сигналы ΠΏΠ»ΠΎΡ…ΠΎ ΠΏΡ€ΠΎΠ½ΠΈΠΊΠ°ΡŽΡ‚ Π² Ρ…ΠΎΡ€ΠΎΡˆΠΈΠ΅ ΠΏΡ€ΠΎΠ²ΠΎΠ΄Π½ΠΈΠΊΠΈ, сопротивлСниС, связанноС с ΠΏΡ€ΠΎΠ²ΠΎΠ΄Π½ΠΈΠΊΠΎΠΌ Π½Π° этих высоких частотах, Π±ΡƒΠ΄Π΅Ρ‚ Π²Ρ‹ΡˆΠ΅, Ρ‡Π΅ΠΌ сопротивлСниС постоянному Ρ‚ΠΎΠΊΡƒ.Π­Ρ‚ΠΎΡ‚ эффСкт извСстСн ΠΊΠ°ΠΊ скин-эффСкт, ΠΏΠΎΡΠΊΠΎΠ»ΡŒΠΊΡƒ высокочастотный Ρ‚ΠΎΠΊ Ρ‚Π΅Ρ‡Π΅Ρ‚ Π² Ρ‚ΠΎΠ½ΠΊΠΎΠΌ слоС Ρƒ повСрхности ΠΏΡ€ΠΎΠ²ΠΎΠ΄Π½ΠΈΠΊΠ°. Π€ΠΎΡ€ΠΌΡƒΠ»Π° для опрСдСлСния эффСктивной Π³Π»ΡƒΠ±ΠΈΠ½Ρ‹ скин-слоя для ΠΏΡ€ΠΎΠ²ΠΎΠ΄Π½ΠΈΠΊΠ° ΠΏΠΎΠΊΠ°Π·Π°Π½Π° Π½ΠΈΠΆΠ΅.

    Π£Ρ€Π°Π²Π½Π΅Π½ΠΈΠ΅ для расчСта Π³Π»ΡƒΠ±ΠΈΠ½Ρ‹ скин-слоя ΠΈΠ»ΠΈ скин-эффСкта ΠΌΠ΅Π΄Π½ΠΎΠ³ΠΎ ΠΏΡ€ΠΎΠ²ΠΎΠ΄Π½ΠΈΠΊΠ°

    для чистой ΠΌΠ΅Π΄ΠΈ Π’ этом случаС d — Π³Π»ΡƒΠ±ΠΈΠ½Π° скин-слоя (Π² ΠΌ), f — ΠΈΠ½Ρ‚Π΅Ρ€Π΅ΡΡƒΡŽΡ‰Π°Ρ частота (Π² Π“Ρ†), m — ΠΏΡ€ΠΎΠ½ΠΈΡ†Π°Π΅ΠΌΠΎΡΡ‚ΡŒ ΠΌΠ°Ρ‚Π΅Ρ€ΠΈΠ°Π»Π° (ΠΌ o ΠΈΠ»ΠΈ 1,2566E-6 H / ΠΌ для Π±ΠΎΠ»ΡŒΡˆΠΈΠ½ΡΡ‚Π²Π° ΠΌΠ°Ρ‚Π΅Ρ€ΠΈΠ°Π»ΠΎΠ²), s — ΠΏΡ€ΠΎΠ²ΠΎΠ΄ΠΈΠΌΠΎΡΡ‚ΡŒ ΠΌΠ°Ρ‚Π΅Ρ€ΠΈΠ°Π»Π° (Π² БимСнсах / ΠΌ ΠΈΠ»ΠΈ 1 / r, Π³Π΄Π΅ r — ΡƒΠ΄Π΅Π»ΡŒΠ½ΠΎΠ΅ сопротивлСниС Π² Ом-ΠΌ).

    Π”ΠΈΠ°Π³Ρ€Π°ΠΌΠΌΠ°, ΠΏΠΎΠΊΠ°Π·Ρ‹Π²Π°ΡŽΡ‰Π°Ρ эффСктивноС сопротивлСниС ΠΊΡ€ΡƒΠ³Π»ΠΎΠΉ ΠΏΡ€ΠΎΠ²ΠΎΠ»ΠΎΠΊΠΈ Π·Π° счСт скин-эффСкта

    Если ΠΈΡΠΏΠΎΠ»ΡŒΠ·ΡƒΠ΅Ρ‚ΡΡ ΠΊΡ€ΡƒΠ³Π»Ρ‹ΠΉ ΠΏΡ€ΠΎΠ²ΠΎΠ΄ с радиусом a, эффСктивноС сопротивлСниС ΠΏΡ€ΠΎΠ²ΠΎΠ΄Π° ΠΌΠΎΠΆΠ½ΠΎ Ρ€Π°ΡΡΡ‡ΠΈΡ‚Π°Ρ‚ΡŒ, ΠΊΠ°ΠΊ ΠΏΠΎΠΊΠ°Π·Π°Π½ΠΎ Π½ΠΈΠΆΠ΅, Π³Π΄Π΅ l — Π΄Π»ΠΈΠ½Π° ΠΏΡ€ΠΎΠ²ΠΎΠ΄Π°, Π° Π΄Ρ€ΡƒΠ³ΠΈΠ΅ ΠΏΠ΅Ρ€Π΅ΠΌΠ΅Π½Π½Ρ‹Π΅ ΠΎΠΏΡ€Π΅Π΄Π΅Π»Π΅Π½Ρ‹, ΠΊΠ°ΠΊ ΡƒΠΊΠ°Π·Π°Π½ΠΎ Π²Ρ‹ΡˆΠ΅.

    Π£Ρ€Π°Π²Π½Π΅Π½ΠΈΠ΅ для расчСта эффСктивного сопротивлСния ΠΏΡ€ΠΎΠ²ΠΎΠ΄Π° Π² зависимости ΠΎΡ‚ Π³Π»ΡƒΠ±ΠΈΠ½Ρ‹ скин-слоя

    ΠΏΡ€ΠΈ a >> d

    ΠŸΡ€ΠΈΠ²Π΅Π΄Π΅Π½Π½ΠΎΠ΅ Π²Ρ‹ΡˆΠ΅ ΡƒΡ€Π°Π²Π½Π΅Π½ΠΈΠ΅ ΠΏΡ€ΠΈΠΌΠ΅Π½ΠΈΠΌΠΎ для Ρ‚Π΅Ρ… случаСв, ΠΊΠΎΠ³Π΄Π° Π³Π»ΡƒΠ±ΠΈΠ½Π° скин-слоя находится ΠΌΠ΅ΠΆΠ΄Ρƒ 0 ΠΈ радиусом ΠΏΡ€ΠΎΠ²ΠΎΠ»ΠΎΠΊΠΈ a.Если Π³Π»ΡƒΠ±ΠΈΠ½Π° скин-слоя большС, Ρ‡Π΅ΠΌ радиус ΠΏΡ€ΠΎΠ²ΠΎΠ΄Π°, Ρ‚ΠΎ эквивалСнтноС сопротивлСниС ΠΏΡ€ΠΎΠ²ΠΎΠ΄Π° ΠΏΠ΅Ρ€Π΅ΠΌΠ΅Π½Π½ΠΎΠΌΡƒ Ρ‚ΠΎΠΊΡƒ Π½Π΅ отличаСтся ΠΎΡ‚ сопротивлСния постоянному Ρ‚ΠΎΠΊΡƒ ΠΈ просто опрСдСляСтся ΠΏΠΎ стандартной Ρ„ΠΎΡ€ΠΌΡƒΠ»Π΅ с использованиСм всСй ΠΏΠ»ΠΎΡ‰Π°Π΄ΠΈ ΠΏΠΎΠΏΠ΅Ρ€Π΅Ρ‡Π½ΠΎΠ³ΠΎ сСчСния ΠΏΡ€ΠΎΠ²ΠΎΠ΄Π°. Когда частота приблиТаСтся ΠΊ Π½ΡƒΠ»ΡŽ (dc), Π³Π»ΡƒΠ±ΠΈΠ½Π° скин-слоя становится бСсконСчной, Π° с ΡƒΠ²Π΅Π»ΠΈΡ‡Π΅Π½ΠΈΠ΅ΠΌ частоты Π³Π»ΡƒΠ±ΠΈΠ½Π° скин-слоя становится всС мСньшС ΠΈ мСньшС.

    Π’ ΡΠ»Π΅Π΄ΡƒΡŽΡ‰Π΅ΠΉ Ρ‚Π°Π±Π»ΠΈΡ†Π΅ ΠΏΠΎΠΊΠ°Π·Π°Π½ΠΎ, ΠΊΠ°ΠΊ Ρ‚ΠΎΠ»Ρ‰ΠΈΠ½Π° скин-слоя измСняСтся Π² зависимости ΠΎΡ‚ ΠΌΠ°Ρ‚Π΅Ρ€ΠΈΠ°Π»Π° ΠΏΡ€ΠΎΠ²ΠΎΠ΄Π½ΠΈΠΊΠ° Π² качСствС ΠΏΡ€ΠΈΠΌΠ΅Ρ€Π° (чистая мСдь ΠΈ чистый алюминий) для Ρ‚ΠΈΠΏΠΈΡ‡Π½Ρ‹Ρ… частот ΠΈΠΌΠΏΡƒΠ»ΡŒΡΠ½ΠΎΠΉ мощности ΠΈ согласования мощности Π² Π΄ΠΈΠ°ΠΏΠ°Π·ΠΎΠ½Π΅ ΠΎΡ‚ 1 ΠΊΠ“Ρ† Π΄ΠΎ 1 Π“Π“Ρ†.

    Частота (Π“Ρ†) Π“Π»ΡƒΠ±ΠΈΠ½Π° ΠΊΠΎΠΆΠΈ Π² ΠΌΠ΅Π΄ΠΈ (см) Π“Π»ΡƒΠ±ΠΈΠ½Π° ΠΊΠΎΠΆΠΈ Π² алюминии (см)
    1000 2.09E-1 2,68E-1
    10 000 6.61E-2 8,46E-2
    100 000 2.09E-2 2,68E-2
    1 000 000 6.61E-3 8.46E-3
    10 000 000 2.09E-3 2,68E-3
    100000000 6.61E-4 8,46E-4
    1 000 000 000 2.09E-4 2,68E-4

    ΠšΠ°Π»ΡŒΠΊΡƒΠ»ΡΡ‚ΠΎΡ€ Π½ΠΈΠΆΠ΅ ΠΌΠΎΠΆΠ½ΠΎ ΠΈΡΠΏΠΎΠ»ΡŒΠ·ΠΎΠ²Π°Ρ‚ΡŒ для опрСдСлСния Π³Π»ΡƒΠ±ΠΈΠ½Ρ‹ скин-слоя для Π΄Π°Π½Π½ΠΎΠ³ΠΎ сплава ΠΌΠ°Ρ‚Π΅Ρ€ΠΈΠ°Π»Π° ΠΈ Ρ€Π°Π±ΠΎΡ‡Π΅ΠΉ частоты. Он Ρ‚Π°ΠΊΠΆΠ΅ Π±ΡƒΠ΄Π΅Ρ‚ ΠΎΡ‚ΠΎΠ±Ρ€Π°ΠΆΠ°Ρ‚ΡŒ ΡƒΠ΄Π΅Π»ΡŒΠ½ΠΎΠ΅ сопротивлСниС, ΠΏΡ€ΠΎΠ²ΠΎΠ΄ΠΈΠΌΠΎΡΡ‚ΡŒ ΠΈ ΠΏΡ€ΠΎΠ½ΠΈΡ†Π°Π΅ΠΌΠΎΡΡ‚ΡŒ, принятыС для Π²Ρ‹Π±Ρ€Π°Π½Π½ΠΎΠ³ΠΎ ΠΌΠ°Ρ‚Π΅Ρ€ΠΈΠ°Π»Π°.РСкомСндуСтся, Ρ‡Ρ‚ΠΎΠ±Ρ‹ ΠΏΠΎΠ»ΡŒΠ·ΠΎΠ²Π°Ρ‚Π΅Π»ΠΈ Π΄Π²Π°ΠΆΠ΄Ρ‹ провСряли эту ΠΈΠ½Ρ„ΠΎΡ€ΠΌΠ°Ρ†ΠΈΡŽ, ΠΏΠΎΡΠΊΠΎΠ»ΡŒΠΊΡƒ ΡƒΠ΄Π΅Π»ΡŒΠ½ΠΎΠ΅ сопротивлСниС ΠΌΠ°Ρ‚Π΅Ρ€ΠΈΠ°Π»Π° ΠΌΠΎΠΆΠ΅Ρ‚ Π²Π°Ρ€ΡŒΠΈΡ€ΠΎΠ²Π°Ρ‚ΡŒΡΡ Π² зависимости ΠΎΡ‚ Ρ‚ΠΎΡ‡Π½ΠΎΠ³ΠΎ состава, отпуска ΠΈ Ρ‚. Π”. Π’Ρ‹Π±Ρ€Π°Π½Π½ΠΎΠ³ΠΎ сплава ΠΌΠ°Ρ‚Π΅Ρ€ΠΈΠ°Π»Π°. ΠšΡ€Π΅Π΄ΠΈΡ‚ Π·Π° исходный ΠΊΠΎΠ΄ Javascript, ΠΈΡΠΏΠΎΠ»ΡŒΠ·ΡƒΠ΅ΠΌΡ‹ΠΉ Π² ΠΊΠ°Π»ΡŒΠΊΡƒΠ»ΡΡ‚ΠΎΡ€Π΅, Π΄Π°Π½ Рэю АллСну, Ρƒ ΠΊΠΎΡ‚ΠΎΡ€ΠΎΠ³ΠΎ Π΅ΡΡ‚ΡŒ нСсколько ΠΏΠΎΠ΄ΠΎΠ±Π½Ρ‹Ρ… ΠΏΠΎΠ»Π΅Π·Π½Ρ‹Ρ… ΠΊΠ°Π»ΡŒΠΊΡƒΠ»ΡΡ‚ΠΎΡ€ΠΎΠ² Π½Π° своСм Π²Π΅Π±-сайтС Pulsed Power Portal.


    ΠšΠΎΠ½ΡΡƒΠ»ΡŒΡ‚Π°Ρ†ΠΈΠΈ, ΠΊΠΎΠΌΠΌΠ΅Π½Ρ‚Π°Ρ€ΠΈΠΈ ΠΈ прСдлоТСния направляйтС ΠΏΠΎ адрСсу [email protected]

    УдСльноС сопротивлСниС — AP Physics 1

    Если Π²Ρ‹ считаСтС, Ρ‡Ρ‚ΠΎ ΠΊΠΎΠ½Ρ‚Π΅Π½Ρ‚, доступный Ρ‡Π΅Ρ€Π΅Π· Π’Π΅Π±-сайт (ΠΊΠ°ΠΊ ΠΎΠΏΡ€Π΅Π΄Π΅Π»Π΅Π½ΠΎ Π² Π½Π°ΡˆΠΈΡ… Условиях обслуТивания), Π½Π°Ρ€ΡƒΡˆΠ°Π΅Ρ‚ ΠΈΠ»ΠΈ Π΄Ρ€ΡƒΠ³ΠΈΠ΅ ваши авторскиС ΠΏΡ€Π°Π²Π°, сообщитС Π½Π°ΠΌ, ΠΎΡ‚ΠΏΡ€Π°Π²ΠΈΠ² письмСнноС ΡƒΠ²Π΅Π΄ΠΎΠΌΠ»Π΅Π½ΠΈΠ΅ (Β«Π£Π²Π΅Π΄ΠΎΠΌΠ»Π΅Π½ΠΈΠ΅ ΠΎ Π½Π°Ρ€ΡƒΡˆΠ΅Π½ΠΈΠΈΒ»), содСрТащСС Π² ΠΈΠ½Ρ„ΠΎΡ€ΠΌΠ°Ρ†ΠΈΡŽ, ΠΎΠΏΠΈΡΠ°Π½Π½ΡƒΡŽ Π½ΠΈΠΆΠ΅, Π½Π°Π·Π½Π°Ρ‡Π΅Π½Π½ΠΎΠΌΡƒ Π½ΠΈΠΆΠ΅ Π°Π³Π΅Π½Ρ‚Ρƒ.Если Ρ€Π΅ΠΏΠ΅Ρ‚ΠΈΡ‚ΠΎΡ€Ρ‹ унивСрситСта ΠΏΡ€Π΅Π΄ΠΏΡ€ΠΈΠΌΡƒΡ‚ дСйствия Π² ΠΎΡ‚Π²Π΅Ρ‚ Π½Π° Π°Π½ Π£Π²Π΅Π΄ΠΎΠΌΠ»Π΅Π½ΠΈΠ΅ ΠΎ Π½Π°Ρ€ΡƒΡˆΠ΅Π½ΠΈΠΈ, ΠΎΠ½ΠΎ ΠΏΡ€Π΅Π΄ΠΏΡ€ΠΈΠΌΠ΅Ρ‚ Π΄ΠΎΠ±Ρ€ΠΎΡΠΎΠ²Π΅ΡΡ‚Π½ΡƒΡŽ ΠΏΠΎΠΏΡ‹Ρ‚ΠΊΡƒ ΡΠ²ΡΠ·Π°Ρ‚ΡŒΡΡ со стороной, которая прСдоставила Ρ‚Π°ΠΊΠΎΠΉ ΠΊΠΎΠ½Ρ‚Π΅Π½Ρ‚ срСдствами самого послСднСго адрСса элСктронной ΠΏΠΎΡ‡Ρ‚Ρ‹, Ссли Ρ‚Π°ΠΊΠΎΠ²ΠΎΠΉ имССтся, прСдоставлСнного Ρ‚Π°ΠΊΠΎΠΉ стороной Varsity Tutors.

    Π’Π°ΡˆΠ΅ Π£Π²Π΅Π΄ΠΎΠΌΠ»Π΅Π½ΠΈΠ΅ ΠΎ Π½Π°Ρ€ΡƒΡˆΠ΅Π½ΠΈΠΈ ΠΏΡ€Π°Π² ΠΌΠΎΠΆΠ΅Ρ‚ Π±Ρ‹Ρ‚ΡŒ ΠΎΡ‚ΠΏΡ€Π°Π²Π»Π΅Π½ΠΎ сторонС, ΠΏΡ€Π΅Π΄ΠΎΡΡ‚Π°Π²ΠΈΠ²ΡˆΠ΅ΠΉ доступ ΠΊ ΠΊΠΎΠ½Ρ‚Π΅Π½Ρ‚Ρƒ, ΠΈΠ»ΠΈ Ρ‚Ρ€Π΅Ρ‚ΡŒΠΈΠΌ Π»ΠΈΡ†Π°ΠΌ, Ρ‚Π°ΠΊΠΈΠΌ ΠΊΠ°ΠΊ Π² качСствС ChillingEffects.org.

    ΠžΠ±Ρ€Π°Ρ‚ΠΈΡ‚Π΅ Π²Π½ΠΈΠΌΠ°Π½ΠΈΠ΅, Ρ‡Ρ‚ΠΎ Π²Ρ‹ Π±ΡƒΠ΄Π΅Ρ‚Π΅ нСсти ΠΎΡ‚Π²Π΅Ρ‚ΡΡ‚Π²Π΅Π½Π½ΠΎΡΡ‚ΡŒ Π·Π° ΡƒΡ‰Π΅Ρ€Π± (Π²ΠΊΠ»ΡŽΡ‡Π°Ρ расходы ΠΈ Π³ΠΎΠ½ΠΎΡ€Π°Ρ€Ρ‹ Π°Π΄Π²ΠΎΠΊΠ°Ρ‚Π°ΠΌ), Ссли Π²Ρ‹ сущСствСнно ΠΈΡΠΊΠ°ΠΆΠ°Ρ‚ΡŒ ΠΈΠ½Ρ„ΠΎΡ€ΠΌΠ°Ρ†ΠΈΡŽ ΠΎ Ρ‚ΠΎΠΌ, Ρ‡Ρ‚ΠΎ ΠΏΡ€ΠΎΠ΄ΡƒΠΊΡ‚ ΠΈΠ»ΠΈ дСйствиС Π½Π°Ρ€ΡƒΡˆΠ°Π΅Ρ‚ ваши авторскиС ΠΏΡ€Π°Π²Π°.Π’Π°ΠΊΠΈΠΌ ΠΎΠ±Ρ€Π°Π·ΠΎΠΌ, Ссли Π²Ρ‹ Π½Π΅ ΡƒΠ²Π΅Ρ€Π΅Π½Ρ‹, Ρ‡Ρ‚ΠΎ ΠΊΠΎΠ½Ρ‚Π΅Π½Ρ‚ находится Π½Π° Π’Π΅Π±-сайтС ΠΈΠ»ΠΈ ΠΏΠΎ ссылкС с Π½Π΅Π³ΠΎ Π½Π°Ρ€ΡƒΡˆΠ°Π΅Ρ‚ ваши авторскиС ΠΏΡ€Π°Π²Π°, Π²Π°ΠΌ слСдуСт сначала ΠΎΠ±Ρ€Π°Ρ‚ΠΈΡ‚ΡŒΡΡ ΠΊ ΡŽΡ€ΠΈΡΡ‚Ρƒ.

    Π§Ρ‚ΠΎΠ±Ρ‹ ΠΎΡ‚ΠΏΡ€Π°Π²ΠΈΡ‚ΡŒ ΡƒΠ²Π΅Π΄ΠΎΠΌΠ»Π΅Π½ΠΈΠ΅, Π²Ρ‹ΠΏΠΎΠ»Π½ΠΈΡ‚Π΅ ΡΠ»Π΅Π΄ΡƒΡŽΡ‰ΠΈΠ΅ дСйствия:

    Π’Ρ‹ Π΄ΠΎΠ»ΠΆΠ½Ρ‹ Π²ΠΊΠ»ΡŽΡ‡ΠΈΡ‚ΡŒ ΡΠ»Π΅Π΄ΡƒΡŽΡ‰Π΅Π΅:

    ЀизичСская ΠΈΠ»ΠΈ элСктронная подпись правообладатСля ΠΈΠ»ΠΈ Π»ΠΈΡ†Π°, ΡƒΠΏΠΎΠ»Π½ΠΎΠΌΠΎΡ‡Π΅Π½Π½ΠΎΠ³ΠΎ Π΄Π΅ΠΉΡΡ‚Π²ΠΎΠ²Π°Ρ‚ΡŒ ΠΎΡ‚ ΠΈΡ… ΠΈΠΌΠ΅Π½ΠΈ; Π˜Π΄Π΅Π½Ρ‚ΠΈΡ„ΠΈΠΊΠ°Ρ†ΠΈΡ авторских ΠΏΡ€Π°Π², ΠΊΠΎΡ‚ΠΎΡ€Ρ‹Π΅, ΠΊΠ°ΠΊ утвСрТдаСтся, Π±Ρ‹Π»ΠΈ Π½Π°Ρ€ΡƒΡˆΠ΅Π½Ρ‹; ОписаниС Ρ…Π°Ρ€Π°ΠΊΡ‚Π΅Ρ€Π° ΠΈ Ρ‚ΠΎΡ‡Π½ΠΎΠ³ΠΎ мСстонахоТдСния ΠΊΠΎΠ½Ρ‚Π΅Π½Ρ‚Π°, ΠΊΠΎΡ‚ΠΎΡ€Ρ‹ΠΉ, ΠΏΠΎ Π²Π°ΡˆΠ΅ΠΌΡƒ мнСнию, Π½Π°Ρ€ΡƒΡˆΠ°Π΅Ρ‚ ваши авторскиС ΠΏΡ€Π°Π²Π°, Π² \ достаточно подробностСй, Ρ‡Ρ‚ΠΎΠ±Ρ‹ ΠΏΠΎΠ·Π²ΠΎΠ»ΠΈΡ‚ΡŒ Ρ€Π΅ΠΏΠ΅Ρ‚ΠΈΡ‚ΠΎΡ€Π°ΠΌ унивСрситСтских школ Π½Π°ΠΉΡ‚ΠΈ ΠΈ Ρ‚ΠΎΡ‡Π½ΠΎ ΠΈΠ΄Π΅Π½Ρ‚ΠΈΡ„ΠΈΡ†ΠΈΡ€ΠΎΠ²Π°Ρ‚ΡŒ этот ΠΊΠΎΠ½Ρ‚Π΅Π½Ρ‚; Π½Π°ΠΏΡ€ΠΈΠΌΠ΅Ρ€, ΠΌΡ‹ Ρ‚Ρ€Π΅Π±ΡƒΠ΅ΠΌ Π° ссылка Π½Π° ΠΊΠΎΠ½ΠΊΡ€Π΅Ρ‚Π½Ρ‹ΠΉ вопрос (Π° Π½Π΅ Ρ‚ΠΎΠ»ΡŒΠΊΠΎ Π½Π° Π½Π°Π·Π²Π°Π½ΠΈΠ΅ вопроса), ΠΊΠΎΡ‚ΠΎΡ€Ρ‹ΠΉ содСрТит содСрТаниС ΠΈ описаниС ΠΊ ΠΊΠ°ΠΊΠΎΠΉ ΠΊΠΎΠ½ΠΊΡ€Π΅Ρ‚Π½ΠΎΠΉ части вопроса — ΠΈΠ·ΠΎΠ±Ρ€Π°ΠΆΠ΅Π½ΠΈΡŽ, ссылкС, тСксту ΠΈ Ρ‚. Π΄. — относится ваша ΠΆΠ°Π»ΠΎΠ±Π°; Π’Π°ΡˆΠ΅ имя, адрСс, Π½ΠΎΠΌΠ΅Ρ€ Ρ‚Π΅Π»Π΅Ρ„ΠΎΠ½Π° ΠΈ адрСс элСктронной ΠΏΠΎΡ‡Ρ‚Ρ‹; ΠΈ Π’Π°ΡˆΠ΅ заявлСниС: (Π°) Π²Ρ‹ добросовСстно ΠΏΠΎΠ»Π°Π³Π°Π΅Ρ‚Π΅, Ρ‡Ρ‚ΠΎ использованиС ΠΊΠΎΠ½Ρ‚Π΅Π½Ρ‚Π°, ΠΊΠΎΡ‚ΠΎΡ€Ρ‹ΠΉ, ΠΏΠΎ Π²Π°ΡˆΠ΅ΠΌΡƒ мнСнию, Π½Π°Ρ€ΡƒΡˆΠ°Π΅Ρ‚ ваши авторскиС ΠΏΡ€Π°Π²Π° Π½Π΅ Ρ€Π°Π·Ρ€Π΅ΡˆΠ΅Π½Ρ‹ Π·Π°ΠΊΠΎΠ½ΠΎΠΌ, Π²Π»Π°Π΄Π΅Π»ΡŒΡ†Π΅ΠΌ авторских ΠΏΡ€Π°Π² ΠΈΠ»ΠΈ Π΅Π³ΠΎ Π°Π³Π΅Π½Ρ‚ΠΎΠΌ; (Π±) Ρ‡Ρ‚ΠΎ всС информация, содСрТащаяся Π² вашСм Π£Π²Π΅Π΄ΠΎΠΌΠ»Π΅Π½ΠΈΠΈ ΠΎ Π½Π°Ρ€ΡƒΡˆΠ΅Π½ΠΈΠΈ, являСтся Ρ‚ΠΎΡ‡Π½ΠΎΠΉ, ΠΈ (c) ΠΏΠΎΠ΄ страхом наказания Π·Π° Π»ΠΆΠ΅ΡΠ²ΠΈΠ΄Π΅Ρ‚Π΅Π»ΡŒΡΡ‚Π²ΠΎ, Ρ‡Ρ‚ΠΎ Π²Ρ‹ Π»ΠΈΠ±ΠΎ Π²Π»Π°Π΄Π΅Π»Π΅Ρ† авторских ΠΏΡ€Π°Π², Π»ΠΈΠ±ΠΎ Π»ΠΈΡ†ΠΎ, ΡƒΠΏΠΎΠ»Π½ΠΎΠΌΠΎΡ‡Π΅Π½Π½ΠΎΠ΅ Π΄Π΅ΠΉΡΡ‚Π²ΠΎΠ²Π°Ρ‚ΡŒ ΠΎΡ‚ ΠΈΡ… ΠΈΠΌΠ΅Π½ΠΈ.

    ΠŸΡ€ΠΎΠ²ΠΎΠ΄

    Π”ΠΎΠ±Π°Π²ΠΈΡ‚ΡŒ ΠΊΠΎΠΌΠΌΠ΅Π½Ρ‚Π°Ρ€ΠΈΠΉ

    Π’Π°Ρˆ адрСс email Π½Π΅ Π±ΡƒΠ΄Π΅Ρ‚ ΠΎΠΏΡƒΠ±Π»ΠΈΠΊΠΎΠ²Π°Π½. ΠžΠ±ΡΠ·Π°Ρ‚Π΅Π»ΡŒΠ½Ρ‹Π΅ поля ΠΏΠΎΠΌΠ΅Ρ‡Π΅Π½Ρ‹ *